Vous êtes sur la page 1sur 263

PG-DIAMS ANATOMY 1

ANATOMY
(Human Anatomy Made Simple)

Dr Rajesh K Kaushal
PG-DIAMS ANATOMY 2

CONTENTS
PG-DIAMS ANATOMY 3

INTRODUCTION
1. Pars intermedia in pituitary develops from
a) Roof of stomodeum
b) Neural crest
c) Alar plate of diencephalon
d) Endoderm

 Pituitary gland is ectodermal in origin and develops from Rathke’s pouch (a surface ectodermal
diverticulum of the primitive oral cavity-stomodeum). Surface ectoderm forms anterior lobe
(adenohypophysis), pars tuberalis, pars intermedia, whereas posterior pituitary
(neurohypophysis/pars nervosa) develops from neuroectoderm (neural plate ectoderm) of
hypothalamus diverticulum (infundibulum).
 Pituitary gland is present at the floor of middle cranial fossa in the sella turcica (Turkish saddle) in
sphenoid bone.
 Cranio-pharyngioma tumour (CPT) is a rare, suprasellar neoplasm, developing from Rathke's pouch
epithelium. Patient may present with bitemporal inferior quadrantanopia leading to bitemporal
hemianopia, as the tumor may compress the optic chiasm.

 Tongue develops from all the three germ layers, connective tissue from pharyngeal arch mesoderm,
muscles from occipital somites and epithelium from ectoderm and endoderm. Sulcus terminalis lies at
the endoderm and ectoderm junction. Tonsillar epithelium develops from endoderm.
PG-DIAMS ANATOMY 4

Section 1. GENERAL EMBRYOLOGY

Gametogenesis

 Gametogenesis is formation of gamete from primordial germ cells and involves cell division mitosis
and meiosis.
 Primordial germ cells (PGCs) are derived from the epiblast, they migrate to the endodermal wall of
the yolk sac (fourth week) and then reach the indeterminate gonad by the end of the fifth week, to
differentiate into gametes (gametogenesis). Aberrant migration may lead to germ cell tumours (for
e.g., teratoma).

1. Abnormal persistence of which of the following cells from primitive streak result in
sacrococcygeal teratoma
a) Primordial germ cells
b) Endodermal cells
c) Mesodermal cells
d) Ectodermal cells
 Teratomas may arise from PGCs (or from epiblast cells), which are pluripotent cells. Therefore, within
teratomas are present derivatives of all three germ layers and may include skin, bone, teeth, gut
tissue.
PG-DIAMS ANATOMY 5

2. After entering first meiotic division, primary oocyte remains arrested in which stage
a) Diplotene
b) Pacyhtene
c) Metaphase
d) Telophase

Oogenesis
 Meiosis consists of two cell divisions (meiosis I and meiosis II) and results in the formation of gametes
containing 23 chromosomes and 1N amount of DNA (1n, 1N), where n is the number of chromosomes
and N is the unit (amount) of DNA. Meiosis I is a reduction division and reduces the chromosome
number to half in gamete.
 During oogenesis, primordial germ cells differentiate into oogonia (46,2N), which enter meiosis I and
undergo DNA replication to form primary oocytes (46,4N). All primary oocytes are formed by the fifth
month of fetal life (about 7 million), and remain dormant in prophase (diplotene/dictyotene stage) of
meiosis I until the beginning of LH surge (puberty).
 Primary oocyte completes meiosis I to form a secondary oocyte (23,2N) and a first polar body (which
later degenerates). The secondary oocyte enters meiosis II, remains arrested in metaphase of
meiosis II until fertilization occurs.
 At fertilization, the secondary oocyte completes meiosis II to form a mature oocyte (23,1N) and a
second polar body.
3. Which cell undergoes fertilization
a) Primary oocyte in prophase arrest
b) Primary oocyte in metaphase arrest
c) Secondary oocyte in prophase arrest
d) Secondary oocyte in metaphase arrest
PG-DIAMS ANATOMY 6

4. Secondary oocyte is
a) Haploid (n) and N
b) Haploid (n) and 2N
c) Diploid (2n) and N
d) Diploid (2n) and 2N
 Fertilization occurs within 24 hours of ovulation, and there is a narrow opportunity window for
conception (2 days preceding or on the day of ovulation).

5. After how many hours of LH surge does ovulation occur


a) 12- 24
b) 24- 48
c) 24- 36
d) 36- 48
 LH surge occurs 34 – 36 hours before ovulation, peaks at 12 hours before ovulation and is
accompanied by release of first polar body.
PG-DIAMS ANATOMY 7

6. First polar body is released


a) 12 hours before ovulation
b) At ovulation
c) 24 hours after ovulation
d) At fertilization

Infertility Clinic

7. Conceptus reaches uterine cavity at which stage


a) 1 cell
b) 2 cell
c) 16 cell
d) 32 cell
PG-DIAMS ANATOMY 8

8. Choose the correct sequence of the following embryonic events


a) Cortical reaction→ Zona reaction → Acrosome reaction
b) Zona reaction → Acrosome reaction → Cortical reaction
c) Acrosome reaction → Cortical reaction → Zona reaction
d) Acrosome reaction → Zona reaction → Cortical reaction

9. Trans-vaginal insertion of conceptus is done at which stage


a) 1 cell
b) 8 cell
c) 16 cell
d) 32 cell
10. After how many days of ovulation embryo implantation occurs
a) 3-5 days
b) 5-7 days
c) 7-9 days
d) 10-12 days
PG-DIAMS ANATOMY 9

Gastrulation

11. Which of the following is NOT true regarding gastrulation


a) Occurs at 3rd week
b) Epiblast cells on inner cell mass form all germ layers
c) Establishes all the three germ layers
d) Occurs at the caudal end of the embryo prior to its cephalic end
12. Germ cells are derivative of
a) Epiblast
b) Endodermal sinus
c) Mesoderm
d) Ectoderm
PG-DIAMS ANATOMY 10

Pre & Post Fertilization Events


13. Heart beat begins at week
a) 4
b) 5
c) 6
d) 8

14. Feto-placental circulation begins at day


a) 12
b) 17
c) 22
d) 30

 Placenta: The fetal portion of the placenta forms from the trophoblast. Syncytiotrophoblast cells are in
direct contact with maternal tissue, whereas the embryo proper is separated from the cytotrophoblast by
extraembryonic mesoderm (together, the chorion).
 Chorion: Extra-embryonic connective tissue + cytotrophoblast + syncytiotrophoblast
 The presumptive umbilical blood vessels form in the wall of the allantois, an endodermal out pocketing
of the urogenital sinus. The amnionic membrane develops from epiblast and is continuous with
embryonic ectoderm. The lining of the yolk sac develops from hypoblast and is continuous with
embryonic endoderm. The yolk sac gives rise to the first blood islands that will form the vitelline vessels.
PG-DIAMS ANATOMY 11

 Primary villus: Syncytiotrophoblast with a cytotrophoblast core. Secondary villus: Cytotrophoblast


core invaded by extraembryonic mesoderm. Tertiary villus: Fetal blood vessels invade the mesoderm
(week 3).
15. Tertiary villi develop by the end of week
a) 1
b) 2
c) 3
d) 4
Germ Layer Derivatives
Ectoderm
16. All are derivatives of ectoderm EXCEPT
a) Epidermis
b) Parotid gland
c) Neuro-hypophysis
d) Arrector pilorum
PG-DIAMS ANATOMY 12

 DiGeorge syndrome : Presents with chromosome 22q11 deletion. Neural crest cell migration is
affected and patients lack mature T cells (due to absence of thymus). There is defective development
of pharyngeal pouch three and four. Most common cause of death is cardiovascular defects, though
severe bacterial infections, hypocalcaemic tetany may also lead to grave consequences.
 Presentation: Positive Chvostek sign (and Trousseau sign); recurrent infections (viral, fungal, and
protozoal); characteristic facies (micrognathia, broad nasal bridge, long face, narrow palpebral fissures,
, asymmetric crying face)
 Diagnostics: Hypocalcemia, lymphopenia, absent thymic silhouette on neonatal imaging
 Treatment: Patients is supported by calcium supplementation and prophylactic antibiotics. Surgical
correction of cardiac and vascular defects may be carried out.

Mesoderm derivatives
PG-DIAMS ANATOMY 13

17. All of the following muscles are derivatives of para-axial mesoderm EXCEPT
a) Masseter
b) Diaphragm
c) Biceps femoris
d) Detrusor
18. Muscle derived from visceral splanchnic lateral plate mesoderm is
a) Myo-epitheliocytes of skin glands
b) Iris muscles
c) Smooth muscles of gut tube
d) Detrusor

 Vertebra develops from the sclerotome (somite), which covers nucleus pulposus and the spinal cord.
Non fusion of sclerotome halves on the posterior aspect results in spina bifida.
PG-DIAMS ANATOMY 14

19. Which of the following develop from paraxial mesoderm


a) Adrenal cortex
b) Humerus
c) Biceps brachii
d) Masseter

 Biceps and Masseter are skeletal muscles developing from para-axial mesoderm. Humerus
(appendicular skeleton) develops from dorsal somatic portion of lateral p;late mesoderm. Adrenal
cortex (and kidney) develops from intermediate mesoderm. Adrenal medulla (contains sympathetic
neurons) is derived from neural crest cells.
 The muscles and bones of the trunk are derived from the somites. Each somite forms two distinct
zones: a sclerotome and a dermomyotome. The former gives rise to the bones of the axial skeleton.
The latter gives rise to the dermatome and myotome, in which dermatome forms the dermis of the
back skin of the trunk and neck and the myotome forms the muscles of the trunk, limbs and tongue.
The dermis and bones of the limbs develop from lateral plate mesoderm. The bones of the head and
neck arise from neural crest cells, as does most of the dermis of the head, whereas the pharyngeal
arch muscles like masticatory, facial, laryngeal muscles etc. arise from unsegmented paraxial
mesoderm in head.
 The smooth muscles of the arterial tree are from various embryological origins.. Upper-body arterial
smooth-muscle cells (like aortic arch) derive from the neural crest, whereas lower-body arteries
derive smooth-muscle cells from neighboring mesodermal structures (for e.g., smooth muscle of
dorsal aorta arise from para-axial mesoderm). Derivatives of the proepicardial organ, which gives rise
to the epicardial layer of the heart, contribute to the vascular smooth-muscle cells of the coronary
arteries. Recent evidence suggests that bone marrow may give rise to both vascular endothelial cells
and smooth-muscle cells, particularly under conditions of injury repair or vascular lesion formation.

Endoderm derivatives
20. Vaginal epithelium is derived from
a) Endoderm of genital ridge
b) Endoderm of urogenital sinus
c) Mesoderm of genital ridge
d) Mesoderm of urogenital sinus

 The most common form of Tracheo-Oesophageal Fistula (TOF), the esophagus ends in a blind
pouch (esophageal atresia) and air enters the stomach (gastric bubble on radiograph). Patient may
present with gastric acid aspiration pneumonitis.
PG-DIAMS ANATOMY 15

21. Smooth muscles of vagina are derived from


a) Para-axial mesoderm
b) Intermediate mesoderm
c) Somatopleuric mesoderm
d) Splanchnopleuric mesoderm

Questions: General Embryology


22. Which of the following pair is INCORRECTLY matched in terms of the conceptus and timeline
in the following diagram displaying female reproductive tract

a) A – Primary oocyte : Ovulation


b) B – Zygote : Day 1 post - ovulation
c) D – Blastocyst (free) : Day 4 post - ovulation
d) E–Blastocyst (attached) : Day 6 post – ovulation

23. How many oocytes are present at fifth month of intrauterine life
a) 1 million
b) 5 million
c) 7 million
d) 400
24. At the end of 5th week of gestation, how many number of somites can be seen
a) 24
b) 26
c) 38
d) 44
25. Which muscle does NOT develop from hypaxial mesoderm
a) Erector spinae
b) Quadratus lumborum
c) Infrahyoid muscles
d) Scalenus muscles
PG-DIAMS ANATOMY 16

Additional Questions
26. Spermatogenesis is completed in
a) 60 days
b) 64 days
c) 70 days
d) 74 days
Ans. d) 74 days >b) 64 days.
Explanation: Spermatogenesis takes 74 days to complete. Earlier editions of standard textbooks used to
mention it as 64 days (2 months).

27. All is true regarding events related to oogenesis EXCEPT


a) Primary oocyte is arrested at prophase – I at birth
b) LH surge occurs 24 – 48 hrs. prior to ovulation
c) First polar body is released before ovulation
d) Meiosis - II is a reduction division
Ans. d) Meiosis II is a reduction division.
Explanation: Chromosome number gets reduced to half during meiosis- I (reduction division). Maternal
and paternal chromosomes separate during meiosis- I.

28. Morula is a cell stage


a) Four
b) Eight
c) Sixteen
d) Thirty two
Ans. c) Sixteen >d) Thirty two.
Explanation: Morula is a mulberry shaped collection of cells after the stage of compaction. It may be 16
cells stage, 32 cells, and even more than 32, till blastocyst is formed.

29. All of the following statements are true concerning the early embryological development
EXCEPT
a) Zona pellucida is a glycoprotein membrane preventing implantation
b) Blastocyst attaches to endometrium on day 6
c) Primordial germ cells are derivative of epiblast
d) The first germ layer to form is ectoderm
Ans. d) The first germ layer to form is ectoderm.
Explanation: Endoderm is the first germ layer to develop from the epiblast, followed by mesoderm and
then ectoderm.
30. Trophoblast differentiates into cyto and syncytio-trophoblast at day
a) 4
b) 6
c) 8
d) 10
Ans. b) 6.
Explanation: Trophoblast in the outer cell mass differentiate into cyto-trophoblast and syncytio-trophoblast
at the 6th day of development and form placenta. Syncytio-trophoblast attaches the conceptus to the
endometrium wall and secrete HCG (Human Chorionic Gonadotropin).
PG-DIAMS ANATOMY 17

31. Which of the following system becomes functional earliest


a) Nervous
b) Respiratory
c) Cardiovascular
d) Gastrointestinal
Ans. c) Cardiovascular.
Explanation: Uteroplacental circulation is established as early as 12th day of life, embryoplacental
circulation at day 17. Heart beat begins as early as day 22 of life.
32. Which of the following is present at the beginning of third week
a) Notochord
b) Primitive streak
c) Mesoderm
d) Neural crest cells
Ans. b) Primitive streak.
Explanation: Primitive streak appears at the end of second week and beginning of third week. Epiblast
cells in the primitive streak forms the notochord, mesoderm and neural crest cells later in the third week.
33. Dilator pupillae muscle is derived from
a) Neural crest cells
b) Neural plate ectoderm
c) Surface ectoderm
d) Mesoderm
Ans. b) Neural plate ectoderm.
Explanation: Smooth muscles of iris (sphincter and dilator pupillae) develop in the neural plate ectoderm.
Neural plate ectoderm forms the CNS (Central Nervous System) and some other derivatives as well.
34. Endoderm gives all EXCEPT
a) Urethra
b) Endocardium
c) Lungs
d) Vagina
Ans. b) Endocardium.
Explanation: Endoderm of the urogenital sinus forms the urethra and vagina epithelium. Lung develops
from the endoderm of anterior part of foregut. Endocardium is present in the heart tube (mesodermal
origin).
35. All is true about notochord EXCEPT
a) Endodermal
b) Appears at week 3
c) Becomes nucleus pulposus
d) Embryonic notochordal remnant may result in chordoma
Ans. a) Endodermal.
Explanation: Notochord is the axial mesoderm forming the axis of the embryo. It appears at week 3 and
later becomes nucleus pulposus of the intervertebral disc. Occasionally it may form a tumour- chordoma.
36. Structures derived from neural crest cells are all EXCEPT (PGIC)
a) Ganglia
b) Mesenchyme of brain
c) Astrocyte and oligodendrocyte
d) AP septum of heart
e) Enamel
Ans. c) Astrocytes and oligodendrocyte; e) Enamel.
Explanation: Astrocyte and oligodendrocyte develop from the neural plate ectoderm. Enamel develops
from ameloblasts (surface ectoderm).
*Most of the ganglia, head & neck mesenchyme and AP septum of heart develop from neural crest cells.
*Teeth develop from neural crest cells (odontoblast forms dentine) and are covered by surface ectoderm
(en-ameloblast forms enamel).
PG-DIAMS ANATOMY 18
PG-DIAMS ANATOMY 19

Neural plate ectoderm (Neuroectoderm) Neural crest cells


-Neural tube(CNS) – All neurons in the brain & spinal cord -Neurons in the ganglia*
-Astrocytes, oligodendrocytes, ependymocytes, tanycytes -Schwann cells
-Neuro-hypophysis (post pituitary) -Head and neck mesenchyme
-Pineal gland -Forebrain meninges (pia and
-Retina and optic nerve arachnoid)
-Epithelium of iris and ciliary body -Skull bones*
-Iris muscles (sphincter and dilator pupillae) -Eyeball*
-Dentine (odontoblast) of teeth
-Pharyngeal arches*
-Parafollicular C cells of thyroid
-Carotid body
-Adrenal medulla
-Melanocyte & Merkel cells
-Aorta-pulmonary(cono-truncal)
septum
-Smooth muscles of head & neck
blood vessels*
-Dermis in the face and neck
*with few exceptions
PG-DIAMS ANATOMY 20

High Yield Facts


 Microglia cells are derivative of monocyte-phagocyte system (mesoderm) and become the
resident macrophages of the brain and spinal cord (CNS), and thus act as the first and main
form of active immune defense.
 Otic placode is a thickening on ectoderm which appears at day 20 – 25, and develops into
components of ear.
 Cell division :
PG-DIAMS ANATOMY 21

Section 2. Histology, Osteology & Arthrology

Epithelium

1. Epithelium lining of lingual surface of epiglottis is


a) Simple Columnar
b) Pseudostratified ciliated columnar
c) Simple Cuboidal
d) Stratified squamous epithelium

 Larynx is lined by respiratory epithelium (pseudo-stratified ciliated columnar epithelium with goblet
cells) except at the vocal cords, which are lined by stratified squamous epithelium.
 Cornea has five layers, in which the anterior most is surface (stratified squamous) epithelium.
 The auricle (pinna) is made up of elastic cartilage and is covered by skin (stratified squamous
epithelium). External auditory canal is covered by skin with sebaceous glands and ceruminous
glands (modified apocrine sweat glands that produce wax).Tympanic membrane is lined by skin
(stratified squamous epithelium) on its external surface and simple cuboidal epithelium on its inner
surface.
 Nasal vestibule is lined by skin with appendages like hair follicles and associated glands.
 Oral cavity has non-keratinized stratified squamous epithelium. Para-keratinization: persistence of
the nuclei of the keratinocytes into the stratum corneum; this is normal only in the epithelium of true
mucous membranes of the mouth and vagina.
PG-DIAMS ANATOMY 22

2. Neuroepithelial type of receptors are present in the following systems EXCEPT


a) Visual
b) Olfactory
c) Gustatory
d) Auditory
PG-DIAMS ANATOMY 23

 Wet surface openings like eyes, oral cavity, urethra, vagina, anal canal should not have keratin
(dryness).

3. Lining epithelium of anal canal below pectinate line is


a) Columnar epithelium
b) Transitional epithelium
c) Non-keratinized stratified squamous epithelium
d) Keratinized stratified squamous epithelium

4. Barrett’s esophagus is diagnosed by which of the following


a) Columnar metaplasia
b) Columnar dysplasia
c) Gastric metaplasia
d) Intestinal metaplasia

Barrett esophagus: Peptic ulcer of the lower esophagus (often with stricture). Biopsy shows replacement
(metaplasia) of normal stratified squamous epithelium by columnar (non - absorptive) epithelium and
goblet cells. Sometimes functional mucous cells, parietal cells, or chief cells may also be present. It is a
pre-malignant leading to esophageal adenocarcinoma occasionally.
PG-DIAMS ANATOMY 24

5. Identify the organ

a) Anal canal
b) Rectum
c) Cervix
d) Oesophagus

 Squamo-columnar junction is present at the endocervix and ectocervix junction. Uterus is lined by
ciliated columnar epithelium and vagina has stratified squamous epithelium.

6. All is true about Brunner’s gland EXCEPT


a) Sub-mucosal glands
b) Secrete urogastrone, which inhibit gastric HCl production
c) Secrete human epidermal growth factor
d) Present in the lower duodenum

Gastric glands
7. All are correct about stomach EXCEPT
a) Pylorus has more acid secreting cells
b) Lots of mucous secreting cells in pylorus
c) Chief cells secrete pepsinogen
d) Parietal cells secrete intrinsic factor
PG-DIAMS ANATOMY 25

Intestinal Epithelium
8. Paneth cells are most distinguished by
a) High zinc content
b) Numerous lysozyme granules
c) Rich rough endoplasmic reticulum
d) Foamy appearance

 Paneth cells is distinguished by the apical eosinophilia in H&E staining. The apical region has large
number of lysozymes, which takes eosin, making the paneth cell appear dark pink at the apex. Paneth
cells are rich in zinc and have large amount of endoplasmic reticulum as well, but are not the
answers of first preference. Mucus in the Goblet cells give the foamy appearance and not a feature of
Paneth cells.
PG-DIAMS ANATOMY 26

 At least six types of cells are found in intestinal mucosal epithelium. 1. Columnar cell (Enterocytes):
For absorption 2. Goblet cell: Mucus production, 3. Paneth cell: Maintain intestinal flora by secreting
antimicrobial substances. 4. Enteroendocrine cell: Secrete paracrine and endocrine hormones 5. M
cells (microfold cells), modified enterocytes that cover enlarged lymphatic nodules in the lamina
propria. 6. Stem cell: for repair of epithelium.

 Paneth cells are columnar epithelial cells migrating towards the base of the intestinal glands,
occurring in the fundus of the crypts of Lieberkuhn; they contain large granules that secrete
antimicrobial substances such as defensins (like TNF - ).

Connective Tissue

Collagen fibres
Type Distribution
I Bone, Fibro-cartilage, skin
II Cartilage (hyaline & elastic)
III Lymphoid tissue, blood vessels (e.g., RA)
IV Basement membrane

Lymphoid Tissue
9. GALT (Gut Associated Lymphoid tissue) is present in
a) Submucosa
b) Lamina propria
c) Muscularis mucosa
d) Adventitia/Serosa
 GALT is present in the lamina propria (mucosa) of the body tubes, though it may also be found in the
submucosa occasionally (for e.g., vermiform appendix). In this case the smooth muscle layer of
muscularis mucosa is interrupted at places.
PG-DIAMS ANATOMY 27

 Primary lymphoid organs: 1. Bone marrow – Has the stem cells that develop into B lymphocytes,
which form plasma cells to secrete antibodies (humoral immunity). Most other leukocytes migrate to
peripheral sites to fully mature. 2. Thymus – Receive stem cells from bone marrow and train them as T-
lymphocyte (cell mediated immunity).
 Secondary lymphoid organs: Tonsils, lymph nodes, spleen, MALT etc. Lymphoid organs have type III
collagen fibres.
 Mucosa-associated lymphoid tissue (MALT) is unencapsulated lymphoid tissue that lines the
digestive tract (GALT), respiratory tract (BALT), and genitourinary tract. Peyer patches are found in the
lamina propria of the ileum and are separated from the intestinal lumen by a layer of flattened epithelial
cells known as microfold cells (M cells). M cells transcytose antigens and present to the underlying
Peyer patches, where APCs phagocytose the antigens and present them to resident T cells and B cells.

Skin
PG-DIAMS ANATOMY 28

 The epidermis of thick skin consists of five layers of cells (keratinocytes): stratum corneum
(characterized by dead and dying cells with compacted keratin), stratum lucidum (a translucent layer
not obvious in thin skin), stratum granulosum (characterized by kerato-hyalin granules), stratum
spinosum (characterized by tonofibrils and associated desmosomes) and stratum basale (proliferative
layer. The epidermis regenerates approximately every 30 days, is carried out by the mitotic activity of
stem cells at the basal layer.
 Non-keratinocyte epidermal cells include melanocytes (derived from neural crest), Langerhans cells
(antigen-presenting cells derived from monocytes), and Merkel cells (sensory mechanoreceptors;
derived from neural crest).
 Langerhans cells are dendritic cells derived from monocyte-phagocyte series in the bone marrow; lack
tonofilaments, desmosomes, and melanosomes. These cells are identified by the presence of tennis
racket–shaped organelles known as Birbeck granules. They are found principally in the stratum
spinosum of the epidermis, but also in lymph nodes, spleen, and thymus. Their surface markers are
characteristic of macrophages, and are antigen-presenting cells involved in contact allergic responses
and other cell-mediated immune reactions in the skin (delayed hypersensitivity).
 Langerhans cell histiocytosis is a disease characterized by the excessive proliferation of Langerhans
cells, which can manifest as skin or bone lesions.
Sensory Receptors

 Sensory receptors are three types :


1. Exteroceptors- receive information from the outside environment.
2. Proprioceptors- receive information from muscles, tendons, and joint structures.
3. Interoceptors- receive information from within the internal environment.
 Merkel cells are neural crest cell derivatives located at the basal layers of epidermis. They are slowly
adapting receptors to detect light touch.
 Meissner corpuscles are rapidly adapting, encapsulated receptors in the dermal papillae (dermo-
epidermo junction). They are distributed in eyelids, lips, nipples, finger tips. They carry fine touch
perception, which is essential for tactile discrimination, and reading Braille.
 Pacinian corpuscles are rapidly adapting encapsulated receptors in the deep dermis and in the
connective tissue of the mesenteries and joints. They carry the information of pressure, touch, and
vibration .
 Ruffini receptors are slowly adapting encapsulated structure in the dermis and joints.They perceive
dermal stretch and pressure.
 Free nerve endings are un-encapsulated, non-myelinated terminations in the skin to carry pain,
temperature etc.
PG-DIAMS ANATOMY 29

 Golgi tendon organs are encapsulated mechanoreceptors sensitive to stretch and tension in tendons
and carry proprioceptive information.
 Muscle spindle receptors are also encapsulated and carry proprioception. They have intrafusal
muscle fibers called flower spray endings and annulospiral endings that sense differences in muscle
length and tension.

Cell Junctions
10. 20 nm of intercellular gap is found in the following cell junction
a) Zona occludens
b) Zona adherence
c) Macula adherence
d) Gap junctions

 Pemphigus vulgaris the most common and severe form of pemphigus, seen usually in persons 40
to 60 years old, characterized by chronic, flaccid, easily ruptured blisters on the skin and mucous
membranes. It begins focally but then becomes generalized, leaving large, weeping, denuded
surfaces that partially crust over but do not heal and enlarge by confluence. Autoantibodies are
detected against the cadherins of desmosomes. Nikolsky sign becomes positive.
 Bullous pemphigoid is a usually mild, self-limited, sub-epidermal blistering skin disease,
sometimes with oral involvement, predominantly affecting the elderly; characteristics include large,
tense bullae that rupture to leave denuded areas and have a tendency to heal spontaneously, and
cleft formation and deposition of complement, usually with the IgG class of immunoglobulins, at the
dermo-epidermal junction. Autoantibodies are detected against the hemi-desmosomes.
PG-DIAMS ANATOMY 30

11. In the electron micrograph below, the structure labelled ‘D’ primarily does which of the
following

a) Forms a spot weld between cells


b) Facilitates communication between adjacent cells
c) Seals membranes between cells
d) Moves microvilli

Cartilage and Bone


12. Fibro cartilage is found in
a) Costal cartilage
b) Nasal septum
c) Intervertebral disc
d) Auditory tube
PG-DIAMS ANATOMY 31

 Osteoblasts synthesize type I collagen and bone matrix proteins to form an unmineralized osteoid.
Calcium and phosphate are deposited on the cartilaginous matrix to form mineralized bone. Blood
supply within the haversian canals supply osteoblasts. Later osteoblasts become surrounded by
bone matrix to become osteocytes.
 Osteocytes are present in the space called lacuna and communicate with other osteocytes via
cytoplasmic extensions called canaliculi . They are not directly involved in bone resorption but under
the influence of parathyroid hormone (PTH) they stimulate osteoclastic bone resorption, which
allows Calcium to be transferred rapidly into the blood.
 Osteoclasts are multinucleated cells (formed from monocytes), contain acid phosphatase and
under influence of PTH cause bone resorption.

 Bone formation occurs in two ways. During endochondral ossification, a cartilage model first forms
and is eventually replaced with bone, except at epiphyseal plates and articular cartilages. This type of
ossification underlie formation of the axial (vertebral column and ribs) and appendicular (limb)
skeletons, with the exception of part of the clavicles.
 During intramembranous ossification, bone forms directly from mesenchymal cells without the prior
formation of cartilage. This type of ossification underlies formation of the majority of bones of the face
and skull.
 Primary ossification centre appears in the developing bone (at 6 – 12 weeks of intra-uterine life) and
forms diaphysis. Secondary ossification centres appear (usually after birth) at the ends of long
bones and develop into epiphysis.
 Nutrient artery is directed away from the growing ends (towards the elbow I go, from the knee I flee).
For e.g., Nutrient foramen in tibia is immediately below the popliteal line and directed obliquely
downward, away from knee joint. It also suggests that the ends of the bones at knee joint and elbow
joints are growing ends.

Joints
13. Inferior tibio-fibular joint is
a) Synchondrosis
b) Syndesmosis
c) Symphysis
d) Schindylesis
PG-DIAMS ANATOMY 32

14. Spheno-vomerine joint is


a) Synchondrosis
b) Syndesmosis
c) Symphysis
d) Schindylesis
PG-DIAMS ANATOMY 33

Questions: Histology, Osteology & Arthrology


15. Strongest layer of intestine is

a) B
b) C
c) D
d) E
 The submucosa consists of a layer of fibroelastic connective tissue containing blood vessels and
nerves. It is the strongest component of the oesophagus and intestinal wall and therefore must be
included in anastomotic sutures.
16. Intestinal epithelium has which cell type (AIIMS)
a) T lymphocytes
b) B lymphocytes
c) Macrophages
d) Neutrophils
17. All of the following are the components of the white pulp of spleen EXCEPT
a) Periarteriolar lymphoid sheath
b) B cells
c) Antigen presenting cells
d) Vascular sinus

 The spleen is composed of white pulp (25%) having large number of white blood cells arranged in
diffuse and nodular lymphoid tissue for immune function. Red pulp (75%) has large number of red
blood cells and consists of venous sinusoids and splenic cords.
 White pulp has lymphoid follicles with eccentric arterioles surrounded by T lymphocytes (PALS – Peri
Arteriolar Lymphatic Sheath). B lymphocytes are distributed at the germinal centres of lymphoid
follicles.
PG-DIAMS ANATOMY 34

18. Hyaline cartilage of respiratory tube extends till


a) Bronchus
b) Terminal bronchiole
c) Respiratory bronchiole
d) Alveolar duct

 Bronchioles have a diameter smaller than 1 mm and lack cartilage and glands within their walls.
Goblet cells (and cilia) decrease in number and almost negligible at the levels of bronchioles (small
lumen). Hyaline cartilage also is almost non-existent at the levels of bronchioles. Epithelium gradually
changes from pseudo -stratified columnar to simple columnar to cuboidal to squamous.
 Alveolus is lined by type-I pneumocyte (simple squamous epithelium) for respiratory gas exchange.
Type- II pneumocyte is a cuboidal cell for surfactant secretion.
 Bronchial arteries (branches of descending thoracic aorta) supply till the level of respiratory
bronchiole. They perfuse the proximal air conducting pathways including tertiary and terminal
bronchioles and reach till the beginning of respiratory unit. Pulmonary arteries alone vascularize the
further distal pathways, including alveolar ducts and the alveoli.

19. All of the following cells are found in lung EXCEPT


a) Kulchitsky cells
b) Clara cells
c) Brush cells
d) Langerhans cells
 Kulchitsky cells are the neuro-endocrine cells secreting hormones; the function of brush cell (with
microvilli) is not known clearly; Clara cells function as stem cells for repair of epithelium. They also
secrete surfactant lipoproteins. Langerhans cells are antigen presenting cells located in the skin and
migrate towards lymphoid tissue.
PG-DIAMS ANATOMY 35

20. Dense and regular arrangement of collagen fibres is seen in all EXCEPT (AIIMS)
a) Tendon
b) Ligament
c) Aponeurosis
d) Periosteum
 Dense connective tissue contains more fibers and fewer cells and is classified by the orientation of
its fiber bundles into two types:
1. Dense and irregular connective tissue (most common), which contains fiber bundles that have no
definite orientation. For e.g., dermis and organ capsules.
2. Dense, regular connective tissue, which contains fiber bundles are arranged in a uniform parallel
fashion with few fibroblasts. E.g., Tendons, ligaments.

21. Serous demilunes are present in large number in which gland


a) Parotid
b) Submandibular
c) Sublingual
d) Pituitary
 Parotid gland is predominantly a serous gland and sublingual gland is mucous. Submandibular
gland is mixed type with a cap of serous gland appearing on mucus gland (serous demilune) in
histological preparations.

Additional Questions
22. Thyroid follicles are lined by (PGIC)
a) Simple squamous
b) Simple cuboidal
c) Stratified cuboidal
d) Simple columnar
e) Stratified columnar
Ans. a) Simple squamous; b) Simple cuboidal; d) Simple columnar.
Explanation: Thyroid follicles are lined by simple cuboidal epithelium. The epithelium changes to simple
columnar in hyper-secretion (amount of endoplasmic reticulum increases), and to simple squamous
epithelium in hypo-secretion (resting follicle).
PG-DIAMS ANATOMY 36

23. Simple squamous cells line


a) Gall bladder
b) Blood vessels
c) Pleura
d) Ependyma
e) Male urethra
Ans. b) Blood vessel; c) Pleura.
Explanation: Endothelium (blood vessels) and mesothelium (pleura, pericardium, peritoneum) are
lined by simple squamous epithelium. Gall bladder is lined by simple columnar epithelium with microvilli
(brush border). Ependyma lines the ventricles of brain and has simple cuboidal (to columnar) cells with
cilia and microvilli. Male urethra is lined by stratified columnar epithelium in entire length except the
beginning (transitional epithelium) and the tip (stratified squamous epithelium).

24. The ducts of all the following glands consist of stratified cuboidal epithelium EXCEPT
a) Sweat glands
b) Sebaceous glands
c) Salivary glands
d) Pancreas
Ans. b) Sebaceous glands.
Explanation: The ducts of all exocrine glands are generally lined by stratified cuboidal/columnar
epithelium, with few exceptions (sebaceous duct) which carries the same lining as that of skin (stratified
squamous epithelium).

25. Chief cells are found at which part of the gastric gland
a) Neck
b) Isthmus
c) Body
d) Fundus
Ans. d) Fundus.
Explanation: Chief cells are chiefly found at fundus of stomach (gross anatomy) and fundus of gastric
gland (histology). Stem cells are predominantly located at the isthmus.

26. Goblet cells are present in all EXCEPT


a) Small intestine
b) Large intestine
c) Esophagus
d) Stomach
Ans. d) Stomach > c) Oesophagus.
Explanation: Goblet cells are absent in the stomach and oesophagus, though oesophagus may have
goblet cells in Barrett’s metaplasia (pathology).

27. All of these cells are found in small intestine EXCEPT


a) Stem cells
b) Neuro - endocrine
c) Neck cells
d) Paneth cells
e) Goblet cell

Ans. c) Neck cells.


Explanation: Mucous neck cells are present in the gastric glands. Small intestinal glands have columnar
(absorptive) cells with goblet cells, along with the neuro-endocrine and stem cells. Paneth cells are
present in the small intestine to maintain the intestinal flora.

28. Which of the following is lined by an epithelium containing ciliated cells and Clara cells
a) Nasopharynx
b) Trachea
c) Respiratory bronchiole
d) Intrapulmonary bronchi
Ans. c) Respiratory bronchiole.
PG-DIAMS ANATOMY 37

Explanation: Clara (club) cells are predominantly present in the terminal bronchiole and also in respiratory
bronchiole. They function as stem cells for the repair of respiratory epithelium. They are also involved in de-
toxification of the inhaled air. They also secrete surfactant lipoproteins.

29. All of the following are categorized as secondary lymphoid organs EXCEPT
a) Lymph nodes
b) Spleen
c) Thymus
d) Subepithelial collections of lymphocytes
e) Bone marrow

Ans. c) Thymus; e) Bone marrow.


Explanation: Primary (central) lymphoid organs have stem cells for lymphopoiesis (bone marrow and
thymus). Secondary (peripheral) lymphoid organs are in the periphery for e.g., lymph node, tonsil, spleen,
MALT (sub-epithelial collection of lymphocytes) etc.

30. Following are given the collagen types and the sites of location. Choose the INCORRECT pair
a) Skin : Type – I
b) Lens Capsule : Type – I
c) Blood vessel : Type – III
d) Spleen : Type – III
e) Hyaline cartilage : Type – I
Ans. b) Lens capsule; e) Hyaline cartilage.
Explanation: Generally capsules have type – I collagen fibres; lens capsule/ filtration membrane have type
IV collagen fibres. Hyaline cartilage has type II collagen fibres.
Collagen fibres
Type Distribution
I Bone, Fibro-cartilage, skin, capsule(joint, organ)
II Cartilage (hyaline & elastic)
III Lymphoid tissue, blood vessels
IV Basement membrane, lens capsule

31. Slowly adapting dermal stretch receptor


a) Merkel’s tactile disc
b) Meissner’s corpuscle
c) Ruffini’s endings
d) Paccinian corpuscle
Ans. c) Ruffini’s endings.
Explanation: Ruffini receptor is present in the deep dermis, detects dermal stretch sensations. Ruffini and
Merkel cells are slowly adapting whereas, Meissner and Paccinian are rapidly adapting receptors.

32. TRUE about Merkel cell


a) Neural crest cell derivative
b) Rapidly adapting receptor
c) Dermal stretch receptor
d) Neural basis for reading Braille text
e) Detect pain and temperature
Ans. a) Neural crest cell derivative.
Explanation: Merkel cell is a slowly adapting receptor at the basal layer of epidermis for detection of light
touch sensations. It is a neural crest cell derivative. Neural basis for reading Braille is Meissner’s
corpuscle mediated reception. Free nerve endings detect pain and temperature form the skin. Ruffini
receptor perceives dermal stretch.

33. Which of the following functions in metabolic coupling between adjacent cells
a) Tight junction
b) Desmosome
c) Gap junction
d) Zonula adherens
PG-DIAMS ANATOMY 38

Ans. c) Gap junction.


Explanation: Metabolic coupling (gap junction) allows free exchange of small molecules across the cells
membranes. It also works as electrical synapse to conduct impulses across the smooth muscle, cardiac
muscle cells.

34. In pemphigus vulgaris autoantibodies are formed against which cell adhesion molecule
a) Selectin
b) Cadherin
c) Integrin
d) IgSF
Ans. b) Cadherin.
Explanation: Autoantibodies are directed against the cadherins (of desmosomes) in pemphigus vulgaris,
which leads to intercellular separation (Nikolsky sign present).

35. Intra-epidermal blistering of skin is observed in


a) Erythema
b) Bullous pemphigoid
c) Pemphigus vulgaris
d) SLE
Ans. c) Pemphigus vulgaris.
Explanation: Intra-epidermal blistering is a sign of intra-epithelial separation as observed in pemphigus
vulgaris. Autoantibodies are directed against the hemi-desmosomes in bullous pemphigoid, leading to
separation of epithelium from the basement membrane (sub-epidermal lesion).

36. What is the type of joint seen in the growth plate


a) Fibrous
b) Primary cartilaginous
c) Secondary cartilaginous
d) Plane synovial
Ans. b) Primary cartilaginous.
Explanation: Primary Cartilaginous Joints (synchondroses) are united by hyaline cartilage and permit no
movement but growth in the length. It Includes epiphyseal cartilage plates (the union between the epiphysis
and the diaphysis of a growing bone) and spheno-occipital and manubrio-sternal synchondroses.

37. All are fibrous joints EXCEPT


a) Gomphosis
b) Syndesmosis
c) Symphysis
d) Schindylesis
Ans. c) Symphysis.
Explanation: Fibrous joints are three types; symphysis is a secondary cartilaginous joint. Schindylesis is
a special suture, seen at the roof of the nasal cavity (spheno-vomerine) joint.

38. Traction epiphysis is/are (PGIC)


a) Head of humerus
b) Lesser tubercle
c) Deltoid tuberosity
d) Coracoid process
e) Greater trochanter
Ans. b) Lesser tubercle; e) Greater trochanter.
Explanation: Traction epiphysis are present at the ends of long bones and develop due to traction by the
attached muscles (and are therefore extracapsular). E.g., Greater (and lesser) tubercles in humerus and
greater (and lesser) trochanter in femur.
PG-DIAMS ANATOMY 39

 Pressure epiphysis are involved in weight transmission (and are intracapsular) for e.g., head of humerus
& femur and condyles of humerus, femur, tibia etc.
 Coracoid process in scapula is an example of atavistic epiphysis.
 Deltoid tuberosity is not an epiphysis (it is present on the shaft/diaphysis).

High Yield Facts


 Vagina and ectocervix is lined by non-keratinized stratified squamous epithelium. Vagina has no
glands though cervix has glandular epithelium.
 Hassal’s corpuscles are present in the medulla of thymus.
 Hilton’s law: The nerve supplying a joint also supplies the muscles that move the joint and the skin
covering the insertion of such muscles.
 Ventricles of brain are lined by ependyma (ciliated columnar epithelium).
 Sebaceous gland is a holocrine gland.
PG-DIAMS ANATOMY 40

Section 3. NeuroAnatomy

Embryology
 Neurulation begins in the third week of development. As the primitive streak regresses caudally, the
notochord develops in the axial line of the embryo (between the buccopharyngeal membrane and
cloacal membrane).
 Notochord induces the overlying ectoderm to form the neural plate. By the end of the third week, the
lateral margins of the neural plate thicken and become elevated to form the neural folds with the
neural groove located centrally between the two folds. The neural folds then grow over the midline
and begin to fuse to form the neural tube. Closure of the neural tube begins in the cervical region
and continues cranially and caudally (The recent literature mentions multiple levels of fusion).
 The anterior (cranial) neuropore closes earlier than the posterior (caudal) neuropore. Failure of closure
of the neuropores results in open neural tube defects (anencephaly and rchischisis) and present with
elevated levels of alpha-fetoprotein levels (and acetylcholine-esterase).
 Neural crest cells are the fourth germ layer cells, which appear at the margins of the neural folds
during closure of the neural tube. (Earlier neural crest cells were considered to be derived from
nneuro-ectoderm).
 Neural crest cells contribute to the peripheral nervous system and most of the ganglia are derived from
these cells.
1. Neural tube begin to close from which region
a) Cranial
b) Cervical
c) Thoracic
d) Lumbar
PG-DIAMS ANATOMY 41

2. The third ventricle of brain belongs to


a) Telencephalon
b) Diencephalon
c) Metencephalon
d) Myelencephalon

 The first CSF (cerebro spinal fluid) is formed of amniotic fluid, and is later secreted by the choroid
(capillary) plexus in the lateral ventricles (chiefly) and partly in third and fourth ventricles. It escapes
the ventricular space at the roof of fourth ventricles at three foramina (midline Magendie and two
lateral Luschka) into the subarachnoid space. (Sub – under).
 CSF circulating in the sub-arachnoid space is absorbed into the dural venous sinus (for e.g., superior
sagittal sinus) via arachnoid granulations. Dural venous sinus is the intradural space (between two
layers of duramater), contains venous blood from several tributaries including veins of brain.
 Non-communicating hydrocephalus may result from obstruction within the ventricles (e.g., congenital
aqueductal stenosis). Communicating hydrocephalus results from blockage within the subarachnoid
space (e.g., adhesions after tuberculous meningitis).
PG-DIAMS ANATOMY 42
PG-DIAMS ANATOMY 43

 Cranial nerve 1, 2 and 8 are pure sensory nerves.

 Hippocampus is concerened with recent memory traces and is related to the inferior (temporal) horn
of lateral ventricle.
PG-DIAMS ANATOMY 44

 Choroid plexus is a capillary plexus projecting into the ventricles to secrete CSF (ultrafiltrate of blood).

3. All is seen in the floor of the third ventricle EXCEPT


a) Mammillary body
b) Occulomotor nerve
c) Optic chiasma
d) Tuber cinereum

 The anterior part of the floor of the third ventricle is formed mainly by hypothalamic structures.
Immediately behind the optic chiasma lies the thin infundibular recess, which extends into the
pituitary stalk. Behind this recess, the tuber cinereum and the mammillary bodies form the floor of
the ventricle.
 Pineal gland is at the posterior wall of third ventricle.
 Thalamus and hypothalamus are at the lateral wall of third ventricle.
 Lamina terminalis is at the anterior wall of third ventricle.

4. All is seen in the floor of the fourth ventricle EXCEPT


a) Vagal triangle
b) Hypoglossal triangle
c) Vestibular area
d) Stria terminalis
PG-DIAMS ANATOMY 45

 Abducent(6) , vestibular(8), vagus(10) , hypoglossal(12) nuclei are at the floor of 4th ventricle.
5. Visceral efferent column in the lateral horns of spinal cord arises from which plate of the neural
tube
a) Alar
b) Basal
c) Roof
d) Floor

 Cranial nerve 3,4,6 and 12 are pure mototr nerves.


PG-DIAMS ANATOMY 46

 Mantle zone of the spinal cord (and brain stem) gets organized into a pair of anterior (basal) plates
and posterior (alar) plates. Laterally, sulcus limitans is present between the two plates; dorsally
and ventrally, they are connected by nonneurogenic structures(roof plate and floor plate). Sensory/
association neurons form in the dorsal plates, the somatic motor column and the visceral motor
column form in the ventral plates.

 Special Somatic Afferent (SSA) fibers convey special sensory impulses of smell, vision, hearing &
balance to the CNS.
 Special Visceral Afferent (SVA) fibers transmit taste sensations to the CNS.
 General Somatic Afferent (GSA) fibers transmit general sensations like touch, pain, temperature,
proprioception from the body to the CNS.
 General Visceral Afferent (GVA) fibers carry sensory impulses from visceral organs to the CNS. For
e.g., carotid sinus pressure sensation.
 General Visceral Efferent (GVE) fibers transmit motor impulses to smooth muscle, cardiac muscle,
and glandular tissues (Autonomic Nervous system).
 General Somatic Efferent (GSE) fibers conduct motor impulses to the skeletal (somatic) muscles of
the body.
 Special Visceral Efferent (SVE) fibers convey motor impulses to the muscles of the head and neck,
which develop from pharyngeal arches such as muscles of mastication, muscles of facial
expression, and muscles of palate, pharynx and larynx (speech & swallowing).

6. Taste pathway comes under the neural column


a) SSA
b) GSA
c) SVA
d) GVA

7. All of the following nuclei belong to GSE (General Somatic Efferent) EXCEPT
a) Occulomotor
b) Trochlear
c) Trigeminal
d) Abducent

 Alar plates give rise to the superior and inferior colliculi (rounded protuberances on the dorsal
surface of the midbrain).The superior colliculi control ocular reflexes; the inferior colliculi serve as
relays in the auditory pathway.
PG-DIAMS ANATOMY 47

Basic NeuroAnatomy

Cerebrum
8. While doing surgery for meningioma on cerebral hemisphere, there occurred injury to left
paracentral lobule, it will lead to paresis of
a) Left face
b) Right neck and scapular region
c) Right leg and perineum
d) Right shoulder and trunk
PG-DIAMS ANATOMY 48
PG-DIAMS ANATOMY 49

 Para-central lobule: Paracentral lobule is on the medial surface of the hemisphere and is the
continuation of the precentral and postcentral gyri. The paracentral lobule is supplied blood by the
anterior cerebral artery. The paracentral lobule controls motor and sensory innervations of the
contralateral lower extremity. It is also responsible for control of defectation and urination.
9. All of the following pairs for Brodmann area are correct EXCEPT
a) Superior temporal gyrus: Auditory cortex (41,42)
b) Superior temporal gyrus: Wernicke’s sensory speech area (22)
c) Inferior frontal gyrus: Broca’s motor speech area (44)
d) Superior frontal gyrus: Frontal eye field (8)
 Frontal eye field (8) is present in the middle frontal gyrus. Frontal eye field is the center for
contralateral horizontal gaze. A lesion results in an inability to make voluntary eye movements
toward the contralateral side. Since the activity of the intact frontal eye field in the opposite cortex is
unopposed in such a lesion, the result is conjugate slow deviation of the eyes toward the side of the
lesion.
 Auditory cortex is located at the anterior part of the superior temporal gyrus (transverse temporal
gyri of Heschl). Wernicke’s sensory speech area is present at the posterior part of the superior
temporal gyrus. Broca’s motor speech area is present on the inferior frontal gyrus.

Basal Ganglia, Thalamus and Internal Capsule


 The basal ganglia is involved in starting (initiation), and stopping of the voluntary motor activity and
inhibiting unwanted movements. It consists of 3 structural nuclei deep in the cerebrum (caudate
nucleus, putamen, and globus pallidus) and two functional nuclei: Substantia nigra (midbrain) and the
subthalamic nucleus (diencephalon).
PG-DIAMS ANATOMY 50

10. Relations of internal capsule are


a) Thalamus medially, caudate and lentiform nuclei laterally
b) Thalamus laterally, caudate and lentiform nuclei medially
c) Thalamus and caudate nucleus medially and lentiform nucleus laterally
d) Thalamus and caudate nucleus laterally and lentiform nucleus medially

11. Which of the following fibres DON’T pass through the posterior limb of internal capsule
a) Sublentiform
b) Retrolentiform
c) Corticonuclear
d) Dorsal column

 Diencephalon includes thalamus and all the related thalami including epithalamus, hypothalamus,
metathalamus, subthalamus etc. Both medial geniculate body and lateral geniculate body are part of
metathalamus.
PG-DIAMS ANATOMY 51

Cranial nerve and Brainstem Nuclei


 Brain stem has three subdivisions: Mid-brain, Pons and Medulla oblongata and has cranial nerve 3-12
nuclei. Nuclei for CN 3 and 4 (midbrain); CN 5-8 (pons); and 9-12 (medulla). Motor nuclei are located
medially (and sensory nuclei are lateral).
 Motor nuclei of cranial nerve are lower motor neurons that innervate the skeletal muscles of the head.
These lower motor neurons are under influence of upper motor neurons by corticobulbar fibers. The
neurone bodies of corticobulbar fibers are located in the cerebrum (frontal motor cortex).
 Corticobulbar (cortico-nuclear) innervation of lower motor neurons is primarily bilateral from both the
right and left cerebral cortex, except for the innervation of the lower muscles of facial expression
(around the mouth); lateral pterygoid (jaw) and genioglossus (tongue), which receive fibres only from
contralateral cerebral cortex.
12. Nucleus tractus solitarius receive fibres from all of the following cranial nerves EXCEPT
a) Facial
b) Glossopharyngeal
c) Vagus
d) Accessory

 Trochlear nerve arises from lower midbrain and Abducent arises from pons.
PG-DIAMS ANATOMY 52

 The NTS (Nucleus Tractus Solitarius) in the medulla receives taste sensation from three nerves: 1.
The anterior two-thirds of the tongue via the chorda tympani nerve of the facial nerve (CN VII) 2.
The posterior third of the tongue via the glossopharyngeal nerve (CN IX) 3. The posteriormost
tongue (and epiglottic region of the pharynx) via the vagus nerve (CN X).
 Neurons carrying taste sensations ascend in the ventral tegmental tract to the VPM nucleus of the
thalamus, which further project the fibres to fibers to the parietal lobe.
13. All of the following pairs regarding neural columns and associated nuclei are correct EXCEPT
a) Hypoglossal nucleus: GSE
b) Nucleus ambigus: SVE
c) Dorsal nucleus of vagus: GVA
d) Nucleus tractus solitarius: SVA
 Nucleus tractus solitarius has both SVA and GVA neural columns. Taste sensations reach the
upper part of the nucleus, whereas, other sensations like carotid sinus pressure sensations reach the
lower part of the nucleus.

*SVE: Special(S) muscles (E) which develop around the pharynx viscera (V) – pharyngeal arch muscles:
Arch – I (Muscles of mastication, 5th nerve)
Arch – II (Muscles of facial expression, 7th nerve)
Arch – III, IV and VI (Palate, pharynx and larynx muscles), Nucleus Ambiguus (9, 10, 11 nerves).
* GVE & GVA are under ANS (Autonomic nervous system).
PG-DIAMS ANATOMY 53

Cerebellum
 Cerebellum is concerned with coordination of voluntary motor activity, controls posture, euillibrium and
muscle tone, and is involved learning of repeated motor functions, Cerebellar lesion leads to abnormal
gait, disturbed balance, and in-coordination of voluntary motor activity (no paralysis or inability to start
or stop movement).
 Mossy fibres (and climbing fibres) are the afferent fibres reaching the cerebellum via the cerebellar
peduncles. These are excitatory in nature and project directly (or indirectly via granule cells) to the
Purkinje cells of the cerebellar cortex.
 The axons of the Purkinje cells are inhibitory (GABA – ergic) and are the only efferent (outflow) from
the cerebellar cortex. They project to and inhibit the deep cerebellar nuclei (dentate, interposed, and
fastigii) in the medulla.
 From the deep nuclei, efferents project through the superior cerebellar peduncle to the contralateral
ventral lateral (and ventral anterior) nuclei of the thalamus, to reach the contralateral cerebrum
(precentral gyrus). The upper motor neurons of the cerebrum thence influence the contralateral lower
motor neurons of the spinal cord via corticospinal tract.
14. Function of spinocerebellar tract
a) Equilibrium
b) Coordinates movement
c) Learning induced by vestibular reflexes
d) Planning and Programming
PG-DIAMS ANATOMY 54

15. Tract NOT present in inferior cerebellar peduncle is


a) Dentato-rubro-thalamic
b) Posterior spinocerebellar
c) Olivocerebellar
d) Reticulo cerebellar

Questions: NeuroAnatomy - I
16. A new-born has multiple congenital defects due to dysgenesis of the neural crest. Which of
the following cells is most likely to be spared
a) Dorsal root ganglion cells
b) Geniculate ganglion cells
c) Melanocytes
d) Motor neurons
17. Brainstem nucleus NOT derived from alar plate
a) Dentate
b) Inferior olivary
c) Hypoglossal
d) Substantia nigra
18. Which of the following cranial nerve is associated with special somatic afferent nuclei
a) V
b) VI
c) VII
d) VIII
PG-DIAMS ANATOMY 55

19. Which of the following reflexes test the integrity of nucleus ambiguus
a) Jaw jerk
b) Stapedial reflex
c) Gag reflex
d) Corneal reflex
PG-DIAMS ANATOMY 56

 Gag reflex: Contraction of the constrictor muscle of the pharynx elicited by touching the back of the
pharynx.
 Stapedial reflex: Contraction of the stapedius muscle in response to loud/intense sound.
 Corneal reflex: Irritation of the cornea results in reflex closure of the lids.
20. Cells present in cerebellar cortex are all EXCEPT
a) Bipolar
b) Purkinje
c) Golgi
d) Granule
21. Efferents in superior cerebellar peduncle arise mostly from
a) Purkinje cells
b) Stellate neurons
c) Deep nuclei
d) Grade III fibres
22. Efferents from cerebellum arise from
a) Purkinje cells
b) Stellate neurons
c) Deep nuclei
d) Grade III fibres
 Efferents from the cerebellar cortex arise from the purkinje cells.

Additional Questions
23. True about cerebrospinal fluid is (PGIC)
a) Produced by choroid plexus
b) Travels from sub-arachnoid space to the fourth ventricle
c) Absorbed by arachnoid villi
d) Drains into the dural venous sinuses
e) Aqueductal stenosis dilates 4th ventricle
Ans. a) Produced by chroid plexus; c) Absorbed by arachnoid villi; d) drains into dural venous
sinuses.
Explanation: CSF moves out of fourth ventricle into the sub-arachnoid space. Aqueductal stenosis dilates
the proximal ventricles 1 ,2 and 3 (and not 4th).

24. CSF escapes the fourth ventricle through


a) Foramen of Monro
b) Aqueduct of Sylvius
c) Luschka foramen
d) Magendie foramen
e) Arachnoid granulations
Ans. c) Luschka foramen; d) Magendie foramen.
Explanation: CSF is produced by the choroid (capillary) plexus projecting into the lateral ventricles
(majorly) and also into third ventricle & fourth ventricle. CSF escapes from the lateral ventricle into third
ventricle via foramen of Monro, passes through aqueduct of Sylvius into the fourth ventricle, then it enters
PG-DIAMS ANATOMY 57

central canal eventually. CSF escapes the fourth ventricle into the sub-arachnoid space via three foramina:
One midline Magendie and two lateral Luschka. CSF absorption from the sub-arachnoid space occurs by
the arachnoid villi (granulations) projecting into the dural venous sinuses (for e.g., superior sagittal sinus).

25. Caudal neuropore closes at day


a) 20
b) 25
c) 28
d) 30
Ans. c) 28.
Explanation: Caudal (posterior) neuropore closes three days later (25+3) to cranial (anterior) neuropore.

26. Alpha-fetoprotein levels are elevated in all EXCEPT


a) Anencephaly
b) Myeloschisis
c) Omphalocele
d) Down syndrome
Ans. d) Down syndrome.
Explanation: In Down syndrome the levels are down.
27. Corticospinal fibres pass through which part of internal capsule
a) Posterior one-third of anterior limb
b) Anterior two-third of posterior limb
c) Posterior two-third of anterior limb
d) Anterior two-third of anterior limb
Ans. b) Anterior two-third of posterior limb.
Explanation: Corticospinal fibres pass through the anterior two-third of the posterior limb of internal
capsule. Corticonuclear tract pass through the genu of internal capsule.

28. Primary visual area is located in the walls of


a) Parieto occipital sulcus
b) Superior temporal sulcus
c) Posterior part of calcarine sulcus
d) Central sulcus
Ans. c) Posterior part of calcarine sulcus.
Explanation: Primary visual area (Brodmann area 17) is located at the posterior part of the calcarine
sulcus. This area shows lines (stria) of Gennari and is also called as striate cortex.

29. All are components of basal ganglia EXCEPT


a) Thalamus
b) Amygdaloid nucleus
c) Substantia nigra
d) Globus pallidus
PG-DIAMS ANATOMY 58

Ans. a) Thalamus.
Explanation: Basal ganglia is involved in programming and planning of the voluntary motor activity and is
constituted by numerous nuclei. It has connections with thalamus (but thalamus is not an integral
component), whereas subthalamus is definitely considered as the functional component of basal ganglia.

30. INCORRECT matching pair about basal ganglia lesion is


a) Wilson’s disease: Lentiform nucleus
b) Athetosis: Globus pallidus
c) Chorea: Striatum
d) Hemiballismus: Substantia nigra
Ans. d) Hemiballismus: Substantia nigra.
Explanation: A lesion in the subthalamus often leads to sudden flailing movements of an entire limb, a
condition called hemiballismus. Lesions of the substantia nigra lead to the common and extremely
severe disease of rigidity, akinesia, and tremors known as Parkinson's disease.Wilson’s disease is a
copper metabolism disorder in paediatric patients, leading to hepato-lenticular degeneration. Lentiform
nucleus lesion is a basal ganglia disorder with onset of purposeless involuntary movements (PIMs). Lesions
in the globus pallidus frequently lead to spontaneous and often continuous writhing movements of a hand,
an arm, the neck, or the face-movements called athetosis.
Multiple small lesions in the putamen (of striatum) lead to flicking movements in the hands, face, and other
parts of the body, called chorea.

31. Corpora quadrigemina is present in


a) Midbrain
b) Upper pons
c) Lower pons
d) Medulla
Ans. a) Midbrain.
Explanation: Corpora quadrigemina (four bodies) are the four colliculi (two superior and two inferior)
located at the dorsal aspect of midbrain (tectum). Superior colliculus is related to visual reflexes, and
inferior colliculus to auditory. In Parinaud (posterior midbrain) syndrome, superior colliculus is involved
leading to disturbances in upward gaze.

32. Red nucleus is situated at the level of


a) Mid-brain; superior colliculus
b) Mid-brain; inferior colliculus
c) Pons
d) Medulla

Ans. a) Midbrain; superior colliculus


Explanation: A transverse section of midbrain, taken at the level of superior colliculus displays the red
nucleus. Weber syndrome: anterior midbrain lesion at this level involving crus cerebri and occulomotor
nerve; Benedict’s syndrome: anterior and mid region is affected involving red nucleus. Parinaud
syndrome: Posterior midbrain lesion involving superior colliculi (upward gaze disturbances).
PG-DIAMS ANATOMY 59

33. Optic nerve is which order neuron


a) First
b) Second
c) Third
d) Fourth
Ans. c) Third.
Explanation: Rods and cones are the first order neurones, synapsing with the bipolar cell (second order
neurone), which in turn synapse on the ganglion cell neurone (third order neurone). Optic nerve is
collection of the axons of ganglion cell neurone ,which is third order neurone in the visual pathway.

34. How many nuclei does the trigeminal nerve have in the CNS
a) Three
b) Four
c) Five
d) Six
Ans. b) Four.
Explanation: Trigeminal nerve has one motor and three sensory nuclei. The motor nucleus is located in
pons and send the motor fibres by mandibular nerve (branch of trigeminal) to control the eight muscles
developing in the first pharyngeal arch, for e.g., muscles of mastication. The main sensory nucleus is
present in the pons, whereas midbrain has the mesencephalic sensory nucleus of trigeminal (for
proprioception) and the spinal sensory nucleus of trigeminal has neurone bodies extending into the spinal
cord (carry pain & temperature).

35. Proprioceptive impulses for masseter reflex are carried to which nucleus of trigeminal nerve
a) Mesencephalic
b) Sensory
c) Motor
d) Spinal
Ans. a) Mesencephalic.
Explanation: Masseter reflex (jaw jerk) is elicited by hitting the mentum (mandible) down with the help of
a knee hammer. It is a proprioceptive reflex carried by the mandibular (trigeminal) nerve towards the
mesencephalic sensory nucleus of trigeminal, the fibres then reaching the motor nucleus of trigeminal in the
pons. Motor fibres carried by the trigeminal mandibular nerve activates the masseter muscle in turn, which
leads to elevation of the mandible.

36. Which of the following is/are projected to ventral posterior nucleus of thalamus (PGIC)
a) Lateral lemniscus
b) Medial lemniscus
c) Corticospinal tract
d) Spinal lemniscus
e) Trigeminal lemniscus
Ans. b) Medial lemniscus; d) Spinal lemniscus; e) Trigeminal lemniscus.
Explanation: Thalamus has a ventral posterior (VP) nucleus, which has two parts: medial and lateral. VPM
(Ventero-Posterior-Medial) nucleus receive sensory input from ‘head’ region, whereas VPL (Ventero-
Postero-Lateral) nucleus receive sensory information from the ‘body’. Trigeminal nerve (first order
neurone) carries information from the head region continues in the trigeminal lemniscus (second order
neurone in brainstem) and synapses on VPM nucleus of thalamus. Thalamus has third order neurones,
which in turn project on to the parietal sensory cortex (1,2,3). Spinothalamic tract- spinal lemniscal system
(pain, temperature) & dorsal column- medial lemniscal system (tactile discrimination,vibration etc.) carry
information from the ‘body’ region to synapse on VPL nucleus of thalamus, which further project the
information to area 1,2,3. Lateral lemniscus carry auditory pathway and synapses with medial geniculate
body (meta-thalamus).

37. All of the following pairs are correct for nuclei of hypothalamus EXCEPT
a) Ventero medial: Hunger
b) Supra-optic: Water conservation
c) Posterior nucleus : Shivering centre
d) Supra-chiasmatic: Circadian rhythm
PG-DIAMS ANATOMY 60

Hypothalamic regions and nuclei:

 Anterior hypothalamus has osmoreceptors and centres like supra-optic nucleus secrete
vasopressin (ADH) for water conservation.

38. Tract present in middle cerebellar peduncle is


a) Dentato-thalamic
b) Posterior spinocerebellar
c) Olivocerebellar
d) Ponto cerebellar

Ans. d) Pontocerebellar.
Explanation: Middle cerebellar peduncle has incoming (afferent) fibres from the contralateral pons (ponto-
cerebellar) fibres. Dentato-thalamic fibres pass through superior cerebellar peduncle from the dentate
nucleus to synapse in thalamus. Posterior spino-cerebellar and the olivo cerebellar fibres pass through
the inferior cerebellar peduncle.

39. Tract absent in superior cerebellar peduncle


a) Tecto-cerebellar
b) Dentato thalamic
c) Dorsal spinocerebellar
d) Ventral spinocerebellar
Ans. c) Dorsal spinocerebellar.
Explanation: Dorsal spininocrebellar fibres pass through inferior cerebellar peduncle. Ventral
spinicerebellar tract send fibres through superior cerebellar peuncle. Superior colliculus (visual reflexes)
and inferior colliculus (auditory) are present in the tectum (midbrain).Tectocerebellar fibres in superior
cerebellar peduncle carry visual and auditory information from the colliculi towards cerebellum.

40. Which is the medial most cerebellar nuclei


a) Dentate
b) Emboliform
c) Fastigi
d) Globose
PG-DIAMS ANATOMY 61

Ans. c) Fastigi.
Explanation: According to evolution, the oldest and medial most is fastigii nucleus.

 Dentate nucleus is the lateral most nuclei and the latest in evolution. It has crumbled bag appearance.
 According to evolution, the flocculonodular lobe (vestibulocerebellum) is the oldest part and doesn’t
even connect with the deep cerebellar nuclei. It is involved in maintenance of equilibrium (balance
and spatial orientation) Damage to this region causes disturbances of gait.

41. In cerebellar lesion not seen is


a) Ataxia
b) Nystagmus
c) Resting tremors
d) Hypotonia
Ans. c) Resting tremors.
Explanation: Cerebellar lesions present with intention (and not resting) tremors. Resting tremor is a
feature of basal ganglia disorder like Parkinson’s disease. Ataxia (incoordination of voluntary motor
activity), hypotonia and nystagmus may be the accompanying features.
PG-DIAMS ANATOMY 62

NeuroAnatomy - II
Spinal Cord – Grey and White Matter
PG-DIAMS ANATOMY 63

42. All is true about sympathetic nervous system fibres arising from the spinal cord EXCEPT
a) Neurons are located in the intermedio-lateral column
b) Pre-ganglionic fibres are myelinated & shorter in length
c) Splanchnic nerves carry GVE and GVA neural columns
d) Splanchnic fibres carry postganglionic fibres

 Spinal Nerves:
■ Consist of 31 pairs: 8 cervical, 12 thoracic, 5 lumbar, 5 sacral, and 1 coccygeal.
■ Are formed from dorsal and ventral roots; each dorsal root has a ganglion that is within the intervertebral
foramen.
■ Are connected with the sympathetic chain ganglia by rami communicantes.
■ Contain sensory fibers with cell bodies in the dorsal root ganglion (general somatic afferent,GSA and
general visceral afferent,GVA) fibers), motor fibers with cell bodies in the anterior horn of the spinal cord
(general somatic efferent,GSE fibers), and motor fibers with cell bodies in the lateral horn of the spinal cord
(general visceral efferent,GVE fibers) between T1 and L2.
■ Are divided into the ventral and dorsal primary rami. The ventral primary rami enter into the formation of
plexuses (i.e., cervical, brachial, and lumbosacral); the dorsal primary rami innervate the skin and deep
muscles of the back.
Sympathetic Nerve Fibers:
■ Have preganglionic nerve cell bodies that are located in the lateral horn of the thoracic and upper lumbar
levels of the spinal cord.
■ Have preganglionic fibers that pass through ventral roots, spinal nerves, and white rami communicantes.
These fibers enter adjacent sympathetic chain ganglia, where they synapse or travel up or down the chain
to synapse in remote ganglia or run further through the splanchnic nerves to synapse in collateral ganglia,
located along the major abdominal blood vessels.
■ Have postganglionic fibers from the chain ganglia that return to spinal nerves by way of gray rami
communicantes and supply the skin with secretory fibers to sweat glands (sudomootor), motor fibers to
smooth muscles of the hair follicles (arrectores pilorum – pilomotor), and vasomotor fibers to the blood
vessels.

43. All is true about Brown Sequard syndrome EXCEPT


a) Hemisection of spinal cord
b) Ipsilateral loss of vibration sensations
c) Ipsilateral loss of crude touch sensations
d) Ipsilateral paralysis below the level of lesion
PG-DIAMS ANATOMY 64
PG-DIAMS ANATOMY 65

 Dorsal column – Medial lemniscal system: First order neurons (dorsal root ganglion) carry the
information ipsilaterally in the dorsal column (fasciculus cuneatus and gracilis) to synapse on second
order neurons located in the gracile and cuneate nuclei of the caudal medulla. They give rise to axons
(internal arcuate fibers) that decussate and form medial lemniscus. The medial lemniscus crosses
midline and ascends through the contralateral brain stem and terminates in the ventral posterolateral
(VPL) nucleus of the thalamus.Third order neurons are located in the VPL nucleus of the thalamus.
They project through the posterior limb of the internal capsule to the postcentral gyrus (Brodmann’s
areas 3, 1, and 2).
PG-DIAMS ANATOMY 66

 Lateral spinothalamic tract – Spinal lemniscal system: First order neurons (dorsal root ganglion)
fibres synapse on the posterior horn cells (second order neurone), which further send the fibres
decussating in the anterior white commisure and run as lateral spinothalamic tract (spinal cord) and
further as spinal lemniscus (in the brainstem). Third order neurons are located in the VPL nucleus of
the thalamus. They project through the posterior limb of the internal capsule to the postcentral gyrus
(Brodmann’s areas 3, 1, and 2).
PG-DIAMS ANATOMY 67

 Pyramidal tract: Fibers arise from pyramidal neurons in layer 5 of the precentral gyrus, premotor
areas and somatic sensory cortex and descend through the posterior limb of internal capsule and
basis pedunculi, cross at the spinomedullary junction and form the lateral corticospinal tract in the
lateral funiculus of the spinal cord. They terminate on lower motor neurons in the ventral horn or on
interneurons. • Most muscles are represented in the contralateral motor cortex. However, some (such
as the muscles of the upper face, the muscles of mastication, and muscles of the larynx) are
represented bilaterally. • With the noted bilateral exceptions, lesion of the pyramidal tract above the
decussation results in spastic paralysis, loss of fine movements, and hyperreflexia on the
contralateral side. • Lesion of the corticospinal tract in the spinal cord results in ipsilateral
symptomology.
PG-DIAMS ANATOMY 68

ANS
44. Sweating is mediated by
a) Adrenal hormones
b) Sympathetic adrenergic system
c) Sympathetic cholinergic system
d) Parasympathetic cholinergic system

 Acetylcholine (ACh) is the preganglionic neurotransmitter for both divisions of the ANS as well as the
postganglionic neurotransmitter of the parasympathetic neurons; the preganglionic receptors are
nicotinic, and the postganglionic are muscarinic in type.
 Norepinephrine (NE) is the neurotransmitter of the postganglionic sympathetic neurons, except for
cholinergic neurons innervating the eccrine sweat glands.
45. All of the following statements about the vagus nerve are true EXCEPT that it
a) Supplies heart & lung
b) Carries postganglionic parasympathetic fibers
c) Innervates right two third of transverse colon
d) Stimulates peristalsis & relaxes sphincters
PG-DIAMS ANATOMY 69

Parasympathetic System

 Preganglionic fibres for the sub-mandibular ganglion arise from superior salivatory nucleus.
PG-DIAMS ANATOMY 70

46. All of the following nuclei belong to GVE (General Visceral Efferent) EXCEPT
a) Edinger Westphal nucleus
b) Lacrimatory nucleus
c) Dorsal nucleus of vagus
d) Abducent

Sympathetic System
47. All are seen in Horner’s syndrome due to cavernous sinus pathology EXCEPT
a) Enophthalmos
b) Ptosis
c) Miosis
d) Anhydrosis
PG-DIAMS ANATOMY 71
PG-DIAMS ANATOMY 72

48. All is true about Horner’s syndrome EXCEPT


a) May result due to lesion in sympathetic hypothalamo-spinal pathway
b) Heterochromia iridis may present as a sign
c) Enophthalmos occurs due to paralyzed orbitalis muscle
d) Apparent exophthalmos is evident

Nervous system: Artery Supply


49. Labyrinthine artery is a branch of
a) Superior cerebellar artery
b) Basilar artery
c) Anterior inferior cerebellar artery
d) Posterior inferior cerebellar artery
PG-DIAMS ANATOMY 73

 Occipital visual cortex has lines (striations) of Gennari, hence tremed as striate cortex. It is supplied
by posterior cerebral artery and additionally by middle cerebral artery.
PG-DIAMS ANATOMY 74

 Optic pathway lesions: a. Optic nerve injury – leads to complete blindness in the involved eye; b.
Midline lesions (like pituitary tumour) results in bitemporal hemianopia (tunnel vision); c. A lesion in
the optic tract results in contralateral homonymous hemianopia; d. Cortical blindness due to an
artery block in posterior cerebral artery results in contralateral homonymous hemianopia with
maculkar sparing (macular area on brain has additional supply from middle cerebral artery).
PG-DIAMS ANATOMY 75

Wallenberg Syndrome
50. A 68-year-old woman presents in the emergency room with dizziness and nystagmus.
Examination reveals a loss of pain and temperature sensation over the right side of the face
and the left side of the body. The patient exhibits ataxia and intention tremor on the right in
both the upper and lower extremities and is unable to perform either the finger-to-nose or heel
to-shin tasks on the right. In addition, she is hoarse and demonstrates pupillary constriction
and drooping of the eyelid on the right. Finally, the right side of her face is drier than the left.
Following vascular blockage, necrotic damage in which of the following would explain the
patient’s hoarseness
a) Descending sympathetic pathways
b) Nucleus ambiguus
c) Spinal nucleus of trigeminal nerve
d) Inferior cerebellar peduncle

Medial medullary syndrome:


1. Corticospinal tract (medullary pyramid) lesions result in contralateral spastic hemiparesis.
2. Medial lemniscus lesion result in contralateral loss of tactile and vibration sensation from body (trunk
and limbs).
3. Hypoglossal nucleus lesion result in ipsilateral flaccid hemiparalysis of the tongue. When protruded,
the tongue deviates to the side of the lesion (i.e., the weak side).
PG-DIAMS ANATOMY 76

51. Nuclei cuneatus and gracilis are present in


a) Spinal cord
b) Medulla oblongata
c) Pons
d) Midbrain

Questions: NeuroAnatomy - II
52. Primary motor area (Area no. 4) of brain is supplied by
a) Anterior cerebral artery
b) Middle cerebral artery
c) Anterior and middle cerebral artery
d) Anterior and posterior cerebral artery

 Greater part of the lateral surface receives supply from middle cerebral artery, whereas medial
surface of cerebrum is majorly supplied by anterior cerebral artery. The upper limb and head are
represented on the lateral surface of the cortex in homunculus, whereas pelvis and lower limb are
on the medial surface of the hemispheres. Therefore, the motor and sensory functions of the lower
limb are supplied by the anterior cerebral artery while the motor and sensory functions of the upper
limb and head are supplied by the middle cerebral artery.

53. Which of the following is a complete sulcus


a) Central sulcus
b) Lateral sulcus
c) Collateral sulcus
d) Calcarine sulcus

 Complete sulcus is the one which is deep enough to reach the wall of the ventricle and raise an
elevation on the interior wall. Collateral sulcus produces an elevation called collateral eminence and
posterior part of calcarine sulcus raises the elevation known as calcar avis.
 Central sulcus is a limiting suclus limiting frontal motor cortex from the parietal sensory cortex.
PG-DIAMS ANATOMY 77

54. All of the following have association fibres EXCEPT


a) Superior longitudinal fasciculus
b) Inferior longitudinal fasciculus
c) Cingulum
d) Corpus callosum

 Association fibers connect regions within the same hemisphere of the brain, whereas commissural
fibers are transverse fibers that connect the two hemispheres of the brain. Projection fibers project
from higher to lower centres (or vice versa) in CNS, for e.g., pyramidal tract.

 Tapetum are the commisural fibres of corpus callosum (body) by which temporal lobes communicate.
55. Regarding anterior choroidal artery syndrome, all are true EXCEPT
a) Hemiparesis
b) Hemi-sensory loss
c) Predominant Involvement of anterior limb of internal capsule
d) Homonymous hemianopia
PG-DIAMS ANATOMY 78

56. All of the following pairs are correct for the artery supply to the lower parts of internal capsule
EXCEPT
a) Anterior limb: Recurrent branch of anterior cerebral artery
b) Genu: Internal carotid artery
c) Posterior limb: Anterior choroidal artery
d) Sublentiform part: Heubner’s artery
 The main arteries supplying internal capsule are:
 Middle cerebral artery
 Anterior cerebral artery (including recurrent branch of Heubner)
 Anterior choroidal artery
 Internal capule also receive additional branches from internal carotid artery, posterior communicating
artery, posterior cerebral artery.
 Upper (dorsal) part of the anterior limb, genu and the posterior limb are supplied by the striate
branches of middle cerebral artery.
 Lower (ventral) part of internal capsule:
 Anterior limb: Anterior cerebral artery (including recurrent branch of Heubner)
 Genu: Internal carotid artery
 Posterior limb: Anteriuor choroidal artery
 Sublentiform and retrolentiform parts are chiefly supplied by supplied by anterior choroidal artery.

57. Internal cerebral veins join to form


a) Inferior cerebral vein
b) Middle cerebral vein
c) Great cerebral vein
d) Anterior cerebral vein
PG-DIAMS ANATOMY 79

 Dural venous sinuses are intradural spaces present between the external (periosteal layer) and the
internal (meningeal layer) of the dura mater, containing venous blood drained from the brain.
a. Superior sagittal sinus is located along the superior aspect of the falx cerebri. Arachnoid granulations
drain CSF from the subarachnoid space into the dural venous sinuses, protrude into its wall.
b. Inferior sagittal sinus is located along the inferior free edge of the falx cerebri.
c. Straight sinus (deeper venous drainage of brain) is formed by the meeting of the great cerebral vein of
Galen with inferior sagittal sinus.
d. Occipital sinus is present in the attached border of the tentorium cerebelli.
e. Confluence of sinuses receive three incoming channels (tributaries) SOS: Straight sinus, Occipital
sinus and Superior sagittal sinus.
f. Transverse sinus drains venous blood from the confluence of sinuses to the sigmoid sinus. It also
receives incoming channel (tributary) from superior petrosal sinus.
g. Sigmoid Sinus drains into the internal jugular vein.
h. Inferior petrosal sinus is the first tributary to internal jugular vein.

58. Septic emboli in facial vein can cause cavernous sinus thrombosis because facial vein makes
clinically important connections with the cavernous sinus. The most commonly involved
communicating vein is
a) Superior ophthalmic
b) Deep facial
c) Inferior ophthalmic
d) Pterygoid plexus of veins

 Cavernous sinuses receive blood from the facial vein via the tributaries superior and inferior
ophthalmic veins. Bacteria in the facial veins enter the cavernous sinus via these veins. Bacteria in the
sphenoid and ethmoid sinuses can spread to the cavernous sinuses via the small emissary veins and
are the most common sites of primary infection resulting in septic cavernous sinus thrombosis.
PG-DIAMS ANATOMY 80

59. Identify the structure at marker B

a) Mammillary body
b) Occulomotor nerve
c) Optic chiasma
d) Infundibulum
60. All are contents of interpeduncular fossa EXCEPT
a) Trochlear nerve
b) Tuber cinerium
c) Infundibular stalk
d) Posterior perforated substance
 All structures at the floor of third ventricle belong to interpeduncular fossa except the optic
chiasma and tegmentum of the midbrain.

Additional Questions
61. Speech in words and not in sentence occurs due to the lesion of
a) Wernicke’s sensory speech area
b) Broca’s motor speech area
c) Arcuate fasciculus
d) Primary auditory area
Ans. b) Broca’s motor speech area.
Explanation: Speech in words and not in sentences is a feature suggesting hesitant speech, which
occurs Broca’s motor aphasia. The planning of movement of speech muscles is compromised and
muscles are unable to articulate properly to produce meaningful voice. In Wernicke’s sensory aphasia,
comprehension (understanding) of the language is compromised and the patient incessantly speaks in
irrelevant sentences, making little sense. Lesion in the arcuate fasciculus result in conduction aphasia,
with problems in repetiton of speech.
62. Which of the following is NOT carried by posterior column tract
a) Position sense
b) Temperature
c) Pressure
d) Vibration
Ans. b) Temperature.
Explanation: Posterior (dorsal) column carries sensations like pressure, vibration, tactile discrimination,
proprioception, stereognosis, conscious proprioception. Pain & temperature is carried by the lateral
spinothalamic tract.
63. Which of the following pathway is involved in the ability to recognize an unseen familiar object
placed in the hand
a) Dorsal spinocerebellar tract
b) Anterior spinothalamic tract
c) Posterior spinothalamic tract
d) Dorsal column
Ans. d) Dorsal column.
Explanation: Ability to recognise an unseen familiar object is known as stereognosis and is carried by the
dorsal column.
PG-DIAMS ANATOMY 81

64. An anterolateral cordotomy relieving pain in left leg is effective because it interrupts the
a) Left dorsal column
b) Right lateral spinothalamic tract
c) Left corticospinal tract
d) Left spinocerebellar tract
Ans. b) Right lateral spinothalamic tract.
Explanation: Pain from the left is carried by lateral spinothalamic tract, contra-laterally on the right side of
the spinal cord.
65. Most of the fibres in Pyramidal tract originate from
a) Primary motor cortex
b) Pre-motor cortex
c) Primary somato-sensory cortex
d) Supplementary motor cortex
Ans. c) Primary somato-sensory cortex.
Explanation: About 31% of the corticospinal tract neurons arise from the primary motor cortex. The
premotor cortex and supplementary motor cortex account for 29% of the corticospinal tract neurons. The
largest percentage of 40% originate in the parietal lobe and primary somatosensory area in the postcentral
gyrus.

66. True about medial lemniscus system


a) Formed from fasciculus gracilis and cuneatus
b) Carries discriminative touch and proprioception
c) Convey pain and temperature
d) Joins spinothalamic tract
e) Decussates at lower medulla
Ans. a) Formed from fasciculus gracilis and cuneatus; b) Carries discriminative touch and
proprioception; e) Decussates at lower medulla.
Explanation: Five sensations (pressure, touch, vibration, stereognosis and proprioception) are carried by
dorsal column (fasciculus gracilis and cuneatus) of spinal cord and synapse in the respective nuclei in
the lower medulla. Second order neurons begin as medial lemniscus and cross the midline (internal
arcuate fibres) in the medulla and ascend up to synapse in thalamus (VPL nucleus). Pain and temperature
is carried by lateral spinothalamic tract – spinal lemniscal system.
67. Synaptic transmission in autonomic ganglia is due to
a) Nicotinic
b) Cholinergic
c) Muscarinic
d) Dopaminergic
Ans. b) Cholinergic.
Explanation: Synaptic transmission in autonomic ganglia (sympathetic and para-sympathetic) is chiefly
mediated by acetylcholine (cholinergic pathway).
68. Preganglionic parasympathetic neurons are located in
a) Cervical and sacral spinal cord
b) Thoracic and lower lumbar spinal cord
c) Brainstem and sacral spinal cord
d) Thoracic spinal cord
Ans. c) Brainstem and sacral spinal cord.
Explanation: Preganglionic fibres arise from CNS and parasympathetic fibres from brainstem and sacral
spinal cord (cranio-sacral flow). Sympathetic fibres arise from the intermediao-lateral horns of spinal cord
segment T-1 to L2/3 (thoraco-lumbar flow).
69. Dilator pupillae is supplied by
a) Oculomotor nerve
b) Sympathetic fibers from the fronto-orbital branch of trigeminal nerve
c) Postganglionic sympathetic fibers from cervical sympathetic chain
d) Postganglionic parasympathetic fibers
Ans. c) Postganglionic sympathetic fibers from cervical sympathetic chain .
PG-DIAMS ANATOMY 82

Explanation: Dilator pupilae is supplied by sympathetic fibres, which arise from the inter-medio-lateral horn
of spinal cord segment T-1. These pre-ganglionic T-1 sympathetic fibres climb up the cervical sympathetic
chain and synapse in the superior (highest) cevical ganglion. Post ganglionic fibres make sympathetic
plexus around the internal carotid artery and reach the dilator pupullae muscle.
70. Which of the following is NOT a sign of stellate ganglion block
a) Miosis
b) Exophthalmus
c) Nasal congestion
d) Conjunctival redness
Ans. b) Exophthalmos.
Explanation: T- 1 sympathetic fibres pass through the stellate ganglion in the sympathetic chain and it’s
block results in features of Horner syndrome. Paralysis of dilator pupillae results in sphincter pupillae
activity becoming more prominent (miosis). Loss of T-1 sympathetic vaso-constriction results in vasodilation
and hypermia (Nasal congestion, conjunctival redness). Horner syndrome results in enophtlamos (and not
exophthalmos).
71. Superior salivatory nucleus controls all of the following glands EXCEPT
a) Lacrimal
b) Palatine
c) Sublingual salivary gland
d) Parotid salivary gland
Ans. d) Parotid salivary gland.
Explanation: Inferior salivatory nucleus located at the lower pons supply parotid salivary gland.
72. A 19 year old woman met with a car accident and sustained crushed internal injury in the
abdomen. The fibers in the vagus nerve are lesioned, which interferes with the functions of,
which of the following structure
a) Urinary bladder
b) Splenic flexure of colon
c) Kidney
d) Uterus
Ans. c) Kidney.
Explanation: Vagus nerve supplies till the kidney level. Pelvic viscera like urinary bladder, uterus are
supplied by nervi erigentes. Splenic flexure of colon belongs to hind gut, supplied by nervi erigentes.
73. NOT affected in posterior cerebral artery infarct is
a) Midbrain
b) Pons
c) Thalamus
d) Striate cortex
Ans. b) Pons.
Explanation: Pons are supplied by basilar artery. Midbrain is supplied by branches of posterior cerebral
artery and basilar artery. Thalamus is supplied by numerous arteries including posterior cerebral artery and
posterior communicating artery. Striate cortex is supplied by posterior cerebral arterya and additionally by
middle cerebral artery (macular area on brain).
74. All of the following arteries supply medulla EXCEPT
a) Anterior spinal artery
b) Anterior inferior cerebellar artery
c) Superior cerebellar
d) Basilar
Ans. c) Superior cerebellar.
Explanation: Medulla oblongata is supplied by numerous arteries (but not superior cerebellar). The
arteries supplying medulla are: Vertebral, anterior spinal, posterior spinal, posterior inferior cerebellar,
anterior inferior cerebellar, basilar etc.
75. Which is NOT a branch of the vertebral artery
a) Anterior spinal
b) Posterior spinal
c) Posterior inferior cerebellar artery
d) Anterior inferior cerebellar artery
PG-DIAMS ANATOMY 83

Ans. d) Anterior inferior cerebellar artery.


Explanation: Anterior inferior cerebellar artery is a branch of basilar artery. It gives labyrinthine artery,
which passes through internal auditory meatus (along with facial and vestibulo-cochlear nerve) and supply
the inner ear. Occasionally labyrinthine artery is a direct branch of basilar artery.
76. A block in the posterior cerebral artery supplying occipital lobe results in
a) Ipsilateral homonymous hemianopia
b) Contralateral homonymous hemianopia
c) ipsilateral homonymous hemianopia with macular sparing
d) Contralateral homonymous hemianopia with macular sparing
Ans. d) Contralateral homonymous hemianopia with macular sparing .
Explanation: Posterior cerebral artery supplies occipital visual (striate) cortex, and a block results in loss of
visual field on the opposite side - contralateral homonymous hemianopia. Left half of each eye is blind in
right posterior cerebral artery infarct. There is associated macular sparing, since the macular area on brain
is additionally supplied by branch of middle cerebral artery.
77. All are parts of corpus callosum EXCEPT
a) Forceps minor
b) Forceps major
c) Tapetum
d) Induseum griseum
Ans. d) Induseum griseum.
Explanation: Indusium griseum is a thin layer of grey matter in contact with the dorsal surface of the
corpus callosum and continuous laterally with the grey matter of the cingulate gyrus.
78. Most common site of berry aneurysm
a) Internal carotid bifurcation
b) Anterior cerebral circulation
c) Middle cerebral circulation
d) Anterior choroidal circulation
Ans. b) Anterior cerebral circulation.
Explanation: Berry aneurysms are more common at the site where anterior communicating artery is given
by anterior cerebral artery (~30%), whereas, the incidence is ~ 25% at the origin of posterior
communicating artery (from internal carotid artery). Bifurcation of middle cerebral artery presents with an
incidence of ~20 %.
79. Most commonly lesioned nerve in intracranial aneurysms is
a) Optic
b) Occulomotor
c) Trochlear
d) Abducent
Ans. b) Occulomotor.
Explanation: Intracranial aneurysms may involve oculomotor ,abducent and optic nerve in descending
order. Trochlear nerve is involvement is highest in head trauma.

80. Following are tributaries of cavernous sinus EXCEPT


a) Superior ophthalmic vein
b) Central vein of retina
c) Great cerebral vein
d) Inferior ophthalmic vein
Ans. c) Great cerebral vein.
Explanation: Great cerebral vein is a tributary to te straight sinus.
PG-DIAMS ANATOMY 84

81. Left sided upper motor neuron lesion of facial nerve paralyzes
a) Right half of the face
b) Left half of the face
c) Right upper half of the face
d) Right lower half of the face
Ans. d) Right lower half of the face.
Explanation: Left sided upper motor neuron lesion (as might occur in Weber syndrome), will result in
contralateral (right sided) facial palsy, where only lower face is involved, since the upper face has bilateral
innervation. Lower face muscles like orbicularis oris have only contralateral innervation, as the left cortico-
bulbar tract is lesiones, they get paralysed. Upper face muscles like orbicularis oculi are functional, as they
are additionally innervated by right cortico-bulbar tract.
In Bell’s palsy (lower motor neuron lesion of facial nerve), both the upper face and lower face muscles are
paralysed on the same side (ipsilateral) of the lesion.

82. The auditory pathway consists of all of the following EXCEPT


a) Lateral geniculate body
b) Superior olivary nucleus
c) Trapezoid body
d) Inferior colliculus
PG-DIAMS ANATOMY 85

Ans. a) Lateral geniculate body.


Explanation: Medial (not lateral) geniculate body is related to auditory pathway. Mnemonic: SLIM – 41,42.
S: Superior olivary nucleus, L: Lateral lemniscus, I: Inferior colliculus, M: Medial geniculate body, 41,42:
Temporal auditory cortex.
*Trapezoid body is present in the ventral pons and contains the crossing fibres from the cochlear nuclei
towards the superior olivary nucleus.
High Yield Facts
 Internal carotid artery gives 5 branches: AM-OCP (A – Anterior cerebral artery, M – Middle cerebral
artery; O – Ophthalmic artery (into orbit), C – Choroidal artery (forms choroid plexus), P – posterior
communicating artery).
 In adults the weight ratio of cerebellum to cerebrum is approximately 1:10 and in infants 1:20.
 Traditionally the reticular nuclei are divided into three columns
 In the median column – the raphe nuclei
 In the medial column – magnocellular nuclei
 In the lateral column – parvocellular nuclei (because of smaller size of the cells)
 Bipolar neurons are found in the olfactory nerve (CN I), in the retina and cochlear & vestibular ganglia of
CN VIII.
 Pseudo-unipolar neurons are located in the spinal (dorsal root) ganglia and sensory ganglia of cranial
nerves (CN) V, VII, IX, and X.
 Light Reflex: Optic nerve carries the visual information to pretectal nucleus. Cells in the pretectal area
send axons to the Edinger-Westphal nuclei on both sides.The Edinger-Westphal nucleus sends
preganglionic parasympathetic fibers in the occulomotor nerve to the ciliary ganglion. Due to bilateral
connection with Edinger-Westphal nuclei, shining light into one eye results in constriction of both the
ipsilateral pupil (direct light reflex) and contralateral pupil (consensual light reflex).
PG-DIAMS ANATOMY 86

Section 4. Head and Neck

Embryology
EyeBall
1. Ciliaris muscle is derived from
a) Neural crest cells
b) Neural plate ectoderm
c) Surface ectoderm
d) Mesoderm

2. Corneal endothelium develop from


a) Neural crest cells
b) Neural plate ectoderm
c) Surface ectoderm
d) Mesoderm

Pharyngeal Arches
3. Epiglottis develops from which branchial arch
a) Third
b) Fourth
c) Fifth
d) Sixth
PG-DIAMS ANATOMY 87
PG-DIAMS ANATOMY 88

Pharyngeal Pouches
4. Parafollicular C cells are derived from
a) Ultimo-branchial body
b) Pharyngeal pouch 4
c) Pharyngeal pouch 5
d) Neural crest cells
PG-DIAMS ANATOMY 89

5. Tonsil develops from


a) First pharyngeal arch
b) Second pharyngeal pouch
c) Third pharyngeal pouch
d) Neural crest cells
6. Thymus develops from
a) Second pharyngeal pouch (ventral portion)
b) Third pharyngeal pouch (ventral portion)
c) Third pharyngeal pouch (dorsal portion)
d) Fourth pharyngeal pouch (ventral portion)
7. Which structure develops from all the 3 germ layers
a) Tympanic membrane
b) External acoustic meatus
c) Auditory tube
d) Middle ear
PG-DIAMS ANATOMY 90

8. Within the figure below of a cross section of an embryo at the level of primitive pharynx, which
of the following structures will give rise to inferior parathyroid

a) 1(inside)
b) 2(inside)
c) 3(inside)
d) 4(inside)

Tongue and Thyroid (Embryology)


9. Tongue develops from all EXCEPT
a) Tuberculum impar
b) Hypobranchial eminence
c) Second arch
d) Lingual swellings
PG-DIAMS ANATOMY 91

10. The taste pathway from circumvallate papillae of the tongue goes through
a) Chorda tympani branch of Facial nerve
b) Greater petrosal nerve branch of Facial nerve
c) Superior laryngeal branch of Vagus nerve
d) Lingual branch of Glossopharyngeal nerve
Cranial Cavity
11. Which of the following cranial nerves present in the posterior fossa
a) 3rd to 12th
b) 4th to 12th
c) 5th to 12th
d) 6th to 12th
PG-DIAMS ANATOMY 92
PG-DIAMS ANATOMY 93

12. CSF rhinorrhea leakage occurs through


a) Frontal sinus
b) Sphenoid sinus
c) Ethmoid sinus
d) Tegmen tympani

13. Which of the following is a tributary as well as drainage channel to cavernous sinus
a) Superior ophthalmic vein
b) Inferior ophthalmic vein
c) Spheno-parietal sinus
d) Superficial middle meningeal vein
PG-DIAMS ANATOMY 94

14. Anterior ethmoidal nerve branch of nasociliary nerve supplies all EXCEPT
a) Dura mater in anterior cranial fossa
b) Ethmoidal cells
c) Internal nasal cavity
d) Maxillary sinus lining
PG-DIAMS ANATOMY 95

15. All structures pass through foramen ovale EXCEPT


a) Accessory meningeal artery
b) Middle meningeal artery
c) Lesser petrosal nerve
d) Emissary vein

 Middle meningeal artery is a branch of maxillary artery, which passes through foramen spinosum to
enter cranial cavity. It may be ruptured in skull fracture leading to extradural haemmorhage, which
requires an emergency removal of clot putting burr holes to save the patient.
16. Structure passing through the tendinous ring of Zinn
a) Superior ophthalmic vein
b) Trochlear nerve
c) Naso-ciliary nerve
d) Lacrimal nerve

17. All are features of cavernous sinus thrombosis EXCEPT


a) Papilloedema
b) Proptosis
c) Sensory deficit on face due to involvement of three branches of trigeminal nerve
d) External ophthalmoplegia due to compression of three motor nerves to eyeball muscles

18. Which of the following is a direct content of cavernous sinus


a) Occulomotor nerve
b) Trochlear nerve
c) Maxillary branch of trigeminal
d) Abducent nerve
PG-DIAMS ANATOMY 96

19. Mass in jugular foramen may result in all EXCEPT


a) Difficulty in swallowing
b) Hoarseness
c) Difficulty in turning the neck to opposite side
d) Tongue deviates to same side
PG-DIAMS ANATOMY 97

 Membrana tectoria is continuation of posterior longitudinal ligament on vertebral column and enters cranial
cavity passing through foramen magnum

Questions: Head & Neck - I


20. Auricular hillocks develop from pharyngeal arch
a) 1
b) 2
c) 1 and 2
d) 2 and 3
PG-DIAMS ANATOMY 98

21. TRUE statement regarding branchial anomalies


a) Most commonly second arch is involved
b) Cyst is more common than sinus
c) Sinus should always be excised
d) Cyst cause dysphagia & hoarseness

22. Choose the INCORRECT statement concerning pharyngeal plexus


a) Receives contributions from vagus nerve carrying cranial accessory nerve component
b) Supplies all pharyngeal muscles except stylopharyngeus
c) Supplies tensor tympani
d) Supply palatoglossus
PG-DIAMS ANATOMY 99

 Most of the palate, pharynx and larynx muscles are supplied by cranial part of accessory nerve (fibres
distributed by vagus nerve branches) with few exceptions like stylopharyngeus(9), and tensor
palati(5).
 All the muscles of tongue are supplied by hypoglossal nerve(12) except palatoglossus (supplied by
cranial accessory nerve).
23. Right fourth arch artery gives rise to
a) Right subclavian artery
b) Common carotid artery
c) Internal carotid artery
d) External carotid artery

24. Double aortic arch occurs due to


a) Non – development of right 4th aortic arch
b) Non – development of left 4th aortic arch
c) Non- division of truncus arteriosus
d) Persistent distal portion of right dorsal aorta
PG-DIAMS ANATOMY 100

 Double aortic arch occurs when an abnormal right aortic arch develops in addition to a left aortic arch
due to persistence of the distal portion of the right dorsal aorta. This forms a vascular ring around the
trachea and esophagus, which causes difficulties in breathing and swallowing.
25. Lesser petrosal nerve passes through
a) Foramen rotundum
b) Foramen ovale
c) Canaliculus innominatus
d) Foramen spinosum
26. Choose the INCORRECT statement about cranial nerves
a) Abducent has the longest intracranial course
b) Trochlear shows internal decussation
c) Olfactory is the shortest
d) Vagus has largest distribution

 Trochlear nerve decussates in the superior medullary velum.


PG-DIAMS ANATOMY 101

 The primary (main) action of the superior oblique muscle is intorsion (internal rotation), the secondary
action is depression (primarily in the adducted position) and the tertiary action is abduction (lateral
rotation).
 Superior oblique is inserted into the posterior part of the eyeball; when it contracts, the back of the
eyeball is elevated, and the front of the eyeball is depressed (particularly in the adducted position).

27. All is true about trochlear nerve EXCEPT


a) Innervates contralateral superior oblique
b) Causes depression of eyeball in adducted position
c) Lies outside the ring of Zinn
d) Patient attains Ipsilateral head tilt, in lesion
Ans. d) Patient attains ipsilateral head tilt, in lesion
Explanation: Trochlear nucleus in the midbrain, send fibres to innervate the contralateral superior
oblique muscle. Superior oblique muscle causes depression and abduction of eyeball. Additionally it also
causes inward rotation (intortion). Trochlear nerve passes through the superior orbital fissure but stays
outside the ring of Zinn. Trochlear nerve is rarely paralyzed alone. It results in vertical diplopia (double
vision) on looking down, e.g. when going down stairs. This happens because the superior oblique normally
assists the inferior rectus in pulling the eye downward, especially when the eye is in a medial (adducted)
position. The patient develops contralateral (not ipsilateral) head tilt to compensate for extorted eye on the
affected side.

28. All is true about Trochlear nerve EXCEPT


a) Slender most cranial nerve
b) Has longest intradural course
c) Innervates contralateral superior oblique
d) Shows internal decussation
Additional Questions
29. Most skeletal elements of the face, for example, bone and cartilages are derived from which of
the following
a) Cranial intermediate mesoderm
b) Cervical somites
c) Neural crest cells migrating from the cranial neural tube
d) The somatic layer of cranial lateral plate mesoderm
Ans. c) Neural crest cells migrating from the cranial neural tube.
Explanation: Most of the skeletal elements of the head and neck are derived from neural crest cells
(secondary mesenchyme).
PG-DIAMS ANATOMY 102

30. Upper body of hyoid bone develops from


a) First arch
b) Second arch
c) Third arch
d) Fourth arch
Ans. b) Second arch.
Explanation: Upper body of hyoid and lesser cornu develops in the second pharyngeal arch whereas,
lower body and greater cornu develops in third arch. Hyoid bone is derived from secondary mesenchyme
contributed by neural crest cells.
31. A patient presented with reddish spot (in circle) that is in the center of a slightly raised area just
anterior to his sternocleidomastoid muscle about one and a half inches superior to his jugular
notch. He has had this reddish raised area for as long as he can remember. If you push on it,
it feels attached to something that extends superiorly from this location. At times it leaks a
little clear fluid after he has been heavily exercising for long periods of time. What do you think
this congenital anomaly is

a) Internal branchial sinus


b) Branchial fistula
c) Hyperactive sebaceous gland
d) Thyroglossal duct cyst
Ans. b) Branchial fistula.
Explanation: Branchial fistula presents with an opening in the lower neck, at the anterior border of sterno-
cleido-mastoid muscle, discharging clear fluid (saliva), having through and through communication with
tonsillar fossa. Internal branchial sinus has an internal (not external) opening into the tonsillar fossa.
Hyperactive sebaceous gland presents with sebaceous (not watery) secretions. Thyroglossal cyst is a
midline presentation (not lateral).

32. Following are the pairs describing skull foramina and the nerves passing through them. Choose
the INCORRECT pair
a) Foramen ovale: Mandibular nerve
b) Foramen spinosum: Maxillary nerve
c) Foramen spinosum: Nervus spinosus
d) Internal acoustic meatus: Nervus intermedius
e) Foramen lacerum: Greater petrosal nerve
Ans. b) Foramen spinosum: Maxillary nerve; e) Foramen lacerum: Greater petrosal nerve.
Explanation: In the foramen spinosum pass nervus spinosus (mandibular nerve branch) and middle
meningeal artery. Maxillary nerve passes through foramen rotundum to enter the pterygopalatine fossa.
Nervus intermedius is also called as Wrisberg nerve and is a component of facial nerve (passes internal
auditory meatus). No structure passes through foramen lacerum, but at the floor are seen internal carotid
artery with sympathetic plexus around, deep petrosal nerve joining greater petrosal nerve to form nerve
of pterygoid canal.

33. Superior orbital fissure contains all EXCEPT


a) Superior ophthalmic vein
b) Inferior ophthalmic vein
c) Ophthalmic nerve
d) Naso-ciliary nerve
Ans. c) Ophthalmic nerve.
PG-DIAMS ANATOMY 103

Explanation: Superior orbital fissure lets pass the three branches of ophthalmic nerve (and not the parent
nerve itself). The three branches are lacrimal, frontal and naso-ciliary nerves.

34. All of the following structures pass through optic foramen EXCEPT
a) Optic nerve
b) Ophthalmic artery
c) Ophthalmic nerve
d) Dura mater
Ans. c) Ophthalmic nerve.
Explanation: Ophthalmic nerve is a content of cavernous sinus, gives three branches, which pass
through superior orbital fissure to enter the orbit. Optic nerve passes along with the ophthalmic artery
through optic canal, which is an opening in the lesser wing of sphenoid at the apex of orbit. Optic nerve is
covered by meninges as it exits the optic canal.

35. Thinnest area of sclera


a) Limbus
b) Behind rectus insertion
c) Equator
d) In front of rectus insertion
Ans. b) Behind rectus insertion.
Explanation: Sclera is thinnest under the insertion of recti muscles.

36. Angular vein communicates with


a) Straight sinus
b) Cavernous sinus
c) Superior sagittal sinus
d) Inferior sagittal sinus
Ans. b) Cavernous sinus.
Explanation: Septic emboli from facial vein may enter the angular vein (at the medial angle of eye) and
then superior ophthalmic vein to enter the cavernous sinus, leading to cavernous sinus thrombosis.

37. Pain sensation from the ethmoid sinus is carried by


a) Frontal nerve
b) Lacrimal nerve
c) Nasociliary nerve
d) Infraorbital nerve
Ans. c) Nasociliary nerve.
Explanation: Pain sensation from ethmoid sinus is carried by ethmoidal nerves → nasociliary nerves →
ophthalmic nerve → trigeminal nerve → spinal sensory nucleus of trigeminal.

38. Afferent component of corneal reflex is carried by


a) Vagus nerve
b) Facial nerve
c) Trigeminal nerve
d) Glossopharyngeal nerve
Ans. c) trigeminal nerve.
Explanation: Corneal touch → nasociliary nerve → ophthalmic nerve → trigeminal nerve → main sensory
nucleus of trigeminal.

39. Ptosis is due to lesion of


a) Facial nerve
b) Somatic fibers of oculomotor nerve
c) Superior cervical ganglion
d) Edinger Westphal nucleus
Ans. b) Somatic fibres of oculomotor nerve > c) Superior cervical ganglion.
Explanation: Ptosis may occur due to lesion in the somatic fibres of occulomotor nerve leading to
paralysis of levator palpebrae superioris (skeletal muscle). Partial ptosis may result due to paralysis of
superior tarsal muscle (part of Muller) in a lesion of T-1 sympathetic pathway in the superior cervical
ganglion (Horner syndrome).
PG-DIAMS ANATOMY 104

40. Floor of orbit formed by all EXCEPT


a) Maxilla
b) Ethmoid
c) Palatine
d) Zygomatic
Ans. b) Ethmoid.
Explanation: The floor (3 bones) of the orbit is chiefly contributed by the orbital plate of the maxilla which
articulates with the zygomatic bone anterolaterally and the small triangular orbital process of the palatine
bone posteromedially.
*Ethmoid bone is present on the medial wall of the orbit.

*Medial wall (4 bones) of orbit is formed by maxilla, lacrimal bone, ethmoid and the sphenoid (body).
*Lateral wall (2 bones)of orbit is formed by the zygomatic bone, and sphenoid (greater wing).
*Roof (2 bones) of orbit has frontal bone and sphenoid (lesser wing)
*Superior orbital fissure is formed between the lateral wall and the roof of orbit.
*Inferior orbital fissure is formed between the medial wall and the floor of orbit. Maxillary nerve passes
through it to run at the floor of the orbit as inferior orbital nerve.
41. UNTRUE statement about orbital articulation is
a) Medial wall of orbit is formed by maxilla, sphenoid, ethmoid and the lacrimal bone
b) Floor is formed by maxilla, zygomatic and palatine bone
c) Lateral wall of orbit is formed by the zygomatic bone and greater wing of sphenoid
d) Inferior orbital fissure is formed between the roof and the lateral wall of orbit
Ans. d) Inferior orbital fissure is formed between the roof and the lateral wall of orbit.
Explanation: Superior (not inferior) orbital fissure is present between the roof and lateral wall of orbit.
Inferior orbital fissure is at the junction of floor and lateral wall of the orbit.
42. Optic canal is present in which part of sphenoid bone
a) Greater wing
b) Lesser wing
c) Body
d) Pterygoid
Ans. b) Lesser wing.
Explanation: Optic canal is an opening in the lesser wing of sphenoid, where it attaches to the body of
sphenoid.
43. Blow-out fracture is present in which wall of orbit (PGIC)
a) Lateral wall
b) Medial wall only
c) Floor only
d) Medial wall and floor
e) Roof
Ans. d) Medial wall and floor.
Explanation: Blow-out fractures are more commonly seen in the floor > medial wall of the orbit.
PG-DIAMS ANATOMY 105

44. Diplopia in superior oblique palsy is described as


a) Vertical on looking down
b) Vertical on looking up
c) Horizontal on looking in
d) Horizontal on looking out
Ans. d) Vertical on looking down.
Explanation: A patient with superior oblique palsy develops vertical diplopia on looking down. For e.g.,
while reading a book or going downstairs.

45. The muscle having contralateral innervation is


a) Inferior oblique
b) Superior rectus
c) Lateral rectus
d) Levator palpebrae superioris
Ans. b) Superior rectus.
Explanation: Superior rectus is suppled by the occulomotor nucleus on the opposite side. Hence a lesion
to the nucleus reults paralysis of contralateral superior rectus muscle.
 Both the superior muscles (superior rectus and superior oblique) have contralateral innervation, and
are the only muscles for intortion(SIN: only Superior muscles do INtortion).
 Levator palpebrae is supplied by a single central subnucleus. A lesion of the nucleus results in bilateral
ptosis.
46. Function of superior oblique muscle is
a) Intorsion, adduction and depression
b) Intorsion, abduction and elevation
c) Intorsion, abduction and depression
d) Extortion, abduction and depression

Ans. c) Intorsion, abduction and depression.


Explanation: Superior oblique muscle causes depression and abduction of eyeball. Additionally it also
causes inward rotation (intortion). Depression in adducted eye is chiefly carried out by superior oblique
muscle (assisted by inferior rectus).

47. Intortor(s) of the eyeball are


a) Superior oblique & superior rectus
b) Superior oblique & Inferior oblique
c) Superior rectus & inferior rectus
d) Inferior rectus & inferior oblique
Ans. a) Superior oblique & superior rectus.
Explanation: Only two muscles work for intortion(SIN: only Superior muscles do INtortion).

48. Following muscles are the abductors of eye EXCEPT


a) Superior oblique
b) Superior rectus
c) Inferior oblique
d) Lateral rectus
Ans. b) Superior rectus.
Explanation: Superior rectus pulls the eye inside (Adduction).
PG-DIAMS ANATOMY 106

Head & Neck - II


Facial Nerve
49. A patient with crocodile tears syndrome has spontaneous lacrimation during eating due to
misdirection of regenerating autonomic nerve fibers. The lesion is located at
a) Facial nerve proximal to the geniculate ganglion
b) Chorda tympani in the infratemporal fossa
c) Facial nerve at the stylomastoid foramen
d) Lacrimal nerve
PG-DIAMS ANATOMY 107

 Facial nerve has two parts: motor to facial expression muscles and nervus intermedius. The motor
part carries SVE component, while nervus intermedius carries GSA, SVA, and GVE fibers.
 SVE: Facial nerve supplies the muscles of facial expression (second pharyngeal arch). The fibres arise
from the motor nucleus of facial nerve (pons), loop around the abducent nucleus (internal genu), raising
facial colliculus, exit the brain stem at the ponto-medullary junction, to enter the internal auditory
meatus, pass through the facial canal in the middle ear cavity, give a branch to stapedius muscle, exit
the skull through the stylomastoid foramen to innervate the stylohyoid muscle, the posterior belly of
the digastric muscle, and enter parotid salivary gland and then send branches to innervate the face
muscles.
 Nervus intermedius (nerve of Wrisberg) carries fibres for taste, salivation, lacrimation, and general
sensation (from the external ear). The first-order sensory neurons are found in the geniculate ganglion
within the temporal bone.
 GSA component brings general sensations from the posterior surface of the external ear through the
posterior auricular branch.
 GVA fibers carry fibres from the soft palate and the adjacent pharyngeal wall.
 SVA component carries taste has from palate and the anterior two-thirds of the tongue to the nucleus
tractus solitarius.
 GVE component begins in the superior salivatory nucleus in the lower pons, carry preganglionic
parasympathetic secretomotor fibres to glands. a. Lacrimal pathway - Secretomotor fibres pass through
the nervus intermedius and greater petrosal nerves to the pterygopalatine (spheno-palatine) ganglion to
supply LNP (lacrimal, nasal, palatine) glands. b. Submandibular pathway - Secretomotor fibres pass
through the nervus intermedius and chorda tympani to the submandibular ganglion to innervate the
submandibular and sublingual salivary glands.
 Chorda tympani is given in the middle ear cavity, runs medial to the tympanic membrane and malleus.
It contains the SVA and GVE (parasympathetic) fibers. It carries pre-ganglionic fibres and is joined by
lingual nerve (a branch of mandibular nerve), which carries post-ganglionic parasympathetic fibres to
reach the submandibular and sublingual salivary glands..
50. All is true about facial colliculus EXCEPT
a) Raised by axons of facial nerve internal genu
b) Abducent nucleus lies deep to it
c) Located at the floor of fourth ventricle
d) Present on the dorsal aspect of upper pons

51. Vidian nerve passes through


a) Inferior orbital fissure
b) Foramen lacerum
c) Tympano-mastoid fissure
d) Pterygoid canal
PG-DIAMS ANATOMY 108

52. Facial nerve has all the following neural columns EXCEPT
a) GVE
b) SVE
c) SVA
d) SSA
Hypoglossal Nerve
53. NOT seen in hypoglossal nerve injury
a) Atrophy of same side
b) Ipsilateral deviation of tongue
c) Loss of tactile sensation of anterior part of tongue
d) Larynx deviation toward the opposite side during swallowing

 Complete division of hypoglossal nerve causes unilateral lingual paralysis and eventual hemi-
atrophy; the protruded tongue deviates to the paralysed side, on retraction, the wasted and paralysed
side rises higher than the unaffected side. The larynx may deviate towards the active side in
swallowing, due to unilateral paralysis of the hyoid depressors associated with loss of the first
cervical spinal nerve which runs with the hypoglossal nerve.
PG-DIAMS ANATOMY 109

Cervical Plexus

Larynx
54. FALSE about larynx
a) 9 cartilages: 3 paired and 3 unpaired cartilages
b) Extends from C3 to C6 vertebrae
c) External laryngeal nerve supply all larynx muscles except cricothyroid
d) Cricothyroid is a tensor of vocal cord
PG-DIAMS ANATOMY 110

55. Damage to the external laryngeal nerve during thyroid surgery could result in the inability to
a) Relax the vocal cords
b) Tense the vocal cords
c) Widen the rima glottidis
d) Abduct the vocal cords

 Damage to the external laryngeal (branch of superior laryngeal) nerve can result when ligating the
superior thyroid artery during thyroidectomy. It can be avoided by ligating the superior thyroid artery at
its entrance into the thyroid gland. Injury to the nerve result in a weak voice with loss of projection, and
the vocal cord on the affected side appears flaccid.
 Unilateral damage to the recurrent laryngeal nerve can result while ligating inferior thyroid artery
during thyroidectomy. It results in a hoarse voice, inability to speak for long periods, and movement of
the vocal fold on the affected side toward the midline.
 Bilateral injury to the recurrent laryngeal nerve may result from while ligating inferior thyroid artery
during thyroidectomy. It results in acute breathlessness (dyspnea) since both vocal folds move toward
the midline and close off the air passage (and tracheostomy might be required).

Oesophagus
56. Marker ‘4’ in the following diagram shows oesophageal narrowing produced by

a) Crico-pharyngeus sphincter
b) Arch of aorta
c) Left principal bronchus
d) Left atrium
PG-DIAMS ANATOMY 111

57. Venous drainage of oesophagus


a) Azygous vein, inferior thyroid vein, right gastric vein
b) Azygous vein, inferior thyroid vein, left gastric vein
c) Azygous vein, right gastric vein, left gastric vein
d) Superior thyroid vein, inferior thyroid vein, azygous vein, hemiazygous vein
PG-DIAMS ANATOMY 112

58. Isthmus of thyroid gland overlies the


a) 1st tracheal cartilage
b) 1st and 2nd tracheal cartilage
c) 2nd, 3rd and 4th tracheal cartilage
d) 3rd and 4th tracheal cartilage
Arteries: Head & Neck
59. In subclavian steal syndrome there is reversal of blood flow in
a) Ipsilateral vertebral artery
b) Contralateral vertebral artery
c) Ipsilateral subclavian artery
d) Contralateral subclavian artery
 Subclavian stenosis proximal to the origin of the vertebral artery, results in a reversal of the blood
flow in the ipsilateral vertebral artery. Heavy manual exercise of the ipsilateral arm may increase
demand on vertebral flow, producing posterior circulation TIAs (subclavian steal syndrome).
PG-DIAMS ANATOMY 113

60. In emergency tracheostomy the following structures are damaged EXCEPT


a) Isthmus of the thyroid
b) Inferior thyroid artery
c) Thyroid ima artery
d) Inferior thyroid vein

 Inferior thyroid veins drain into brachio-cephalic vein and are prone to injury in tracheostomy
procedure.

Neck Triangles and Fasciae


61. All of the following are in the anterior triangle of neck EXCEPT
a) Digastric
b) Subclavian
c) Muscular
d) Submental

 Posterior neck triangle is bounded by the trapezius, sternocleidomastoid, and clavicle and is
subdivided by the posterior belly of the omohyoid into the occipital and subclavian triangles. The
contents are spinal accessory nerve; cervical plexus; brachial plexus (roots and trunks); and subclavian,
transverse cervical & suprascapular arteries.
 Anterior neck triangle is bounded by the sternocleidomastoid, mandible, and midline of the neck and
is subdivided by the anterior & posterior bellies of digastric anterior and anterior belly of the omohyoid
into the submandibular, carotid, muscular, and submental triangles.
PG-DIAMS ANATOMY 114

Triangle Main Contents and the underlying structures


A. Posterior triangle
1. Occipital triangle Spinal accessory nerve, brachial plexus (Trunks),
cervical plexus (branches), external jugular vein
2. Omoclavicular (subclavian) triangle Subclavian artery (3rd part), brachial plexus
(Trunks), cervical plexus (branches), external
jugular vein
PG-DIAMS ANATOMY 115

B. Anterior triangle
1. Carotid triangle Carotid sheath (containing common carotid artery,
Internal jugular vein and vagus nerve), Ansa
cervicalis, sympathetic trunk, CN – XI and XII
2. Submandibular (digastric) triangle Submandibular salivary gland, CN – XII, mylohyoid
nerve, facial artery
3. Sub-mental triangle Sub-mental lymph nodes
4. Musculat triangle Strap (ribbon) muscles: sternothyroid, sternohyoid

 Brachial plexus can be blocked in the scalene triangle between scalenus anterior and medius.
 Carotid triangle contains the bifurcation of the common carotid artery (into internal & external carotid
artery at the level of C4). Carotid body and sinus are be found at the bifurcation.
 The carotid sheath contains the common and internal carotid arteries, internal jugular vein, and vagus
nerve. Sympathetic trunk lies posterior to the carotid sheath (embedded in the prevertebral fascia).
62. All is true about digastric triangle EXCEPT
a) On either side is anterior belly of digastric muscle
b) Floor is formed by mylohyoid muscle
c) Floor is formed by hyoglossus muscle
d) Contains mylohyoid nerve and vessels
PG-DIAMS ANATOMY 116

63. If there is a superficial cut in the region of middle part of posterior triangle of neck, patient will
experience problem in
a) Adduction of arm
b) Protraction of scapula
c) Shrugging of shoulder
d) Overhead abduction of arm
64. All is true about cervical fascia EXCEPT
a) Ligament of Berry fixes thyroid gland to cricoid cartilage
b) Prevertebral fascia forms the roof of posterior triangle
c) Ansa cervicalis is embedded in the anterior wall of carotid sheath
d) Carotid sheath is formed by pretracheal and prevertebral fascia

 Deep cervical fascia forms a) investing layer, b) pretracheal layer and c) prevertebral layer.
a). Investing Layer encircles the neck and splits to enclose the trapezius and the sternocleidomastoid
muscles. It is at the roof of posterior triangle.
b). Pretracheal Layer surrounds the thyroid (and the parathyroid) glands, and encloses the infrahyoid
muscles.
c). Prevertebral Layer lies in front of the prevertebral muscles (like scalene muscles) behind the pharynx
& esophagus. It forms the floor of posterior triangl and extends laterally over the first rib into the axilla to
form axillary sheath (which encloses brachial brachial and axillary artery).
 Carotid sheath is condensation of the prevertebral, pretracheal, and the investing layers of the deep
cervical fascia.
 A thyroid mass usually moves with swallowing because the thyroid gland is enclosed by pretracheal
fascia.
PG-DIAMS ANATOMY 117

Questions: Head & Neck - II


65. In the fracture of middle cranial fossa, lacrimation is affected in injury of
a) Nasociliary nerve
b) Greater petrosal nerve
c) Lesser petrosal nerve
d) Auriculotemporal nerve
66. Schirmer’s test evaluates the function of
a) Greater petrosal nerve
b) Lesser petrosal nerve
c) Chorda tympani nerve
d) Auriculotemporal nerve
67. Skin over angle of mandible is supplied by
a) Posterior primary rami of C-2,3
b) Greater auricular nerve
c) Maxillary nerve
d) Mandibular nerve
68. Relaxor of vocal cord is the muscle
a) Thyro-arytenoid
b) Cricothyroid
c) Posterior crico-arytenoid
d) Lateral cricoarytenoid

69. Ascending pharyngeal artery is a branch of


a) External carotid artery
b) Internal carotid artery
c) Common carotid artery
d) Maxillary artery

 Facial artery can be palpated at the anterior border of masseter muscle.


 Middle meningeal artery is a branch of 1st part of maxillary artery. It passes through foramen
spinosum. It is damaged in skull fracture at pterion, leading to extra(epi) dural haematoma.
PG-DIAMS ANATOMY 118

70. The arteries labelled as ‘ABCD’ are

a) Internal Greater palatine → posterior ethmoidal → superior labial → lesser palatine


b) Greater palatine → anterior ethmoidal → superior labial → lesser palatine
c) Sphenopalatine → anterior ethmoidal → superior labial → greater palatine
d) Sphenopalatine → anterior ethmoidal → superior labial → lesser palatine
71. All is true about the openings in the lateral wall of nasal cavity and nasopharynx EXCEPT
a) Nasolacrimal duct opens in the inferior meatus
b) Posterior ethmoidal sinus open in the superior meatus
c) Inferior turbinate is a part of ethmoid bone
d) Eustachian tube opens in nasopharynx behind the inferior turbinate

 The nasolacrimal duct opens into the inferior meatus is partially covered by a mucosal fold (valve of
Hasner). Excess tears flow through nasolacrimal duct which drains into the inferior nasal meatus. It is
directed downward, backward and laterally.
 Maxillary sinus opens into the middle meatus (hiatus semilunaris).
PG-DIAMS ANATOMY 119

72. Which of the following is the type of joints between malleus and incus
a) Primary cartilaginous
b) Secondary cartilaginous
c) Saddle synovial
d) Ball & socket synovial

 Atlanto-occipital joint is an ellipsoid (condylar) synovial joint. Neck flexion and extension occurs at this
joint for the nodding (yes) movement.

 Knee joint is a complex joint (involving more than two bones). Femoro-tibial joint structurally resembles
a hinge joint, but is considered as a condylar type of synovial joint between two condyles of the femur
and tibia. In addition, it includes a saddle joint between the femur and the patella.

73. Atlanto-occipital joint is of synovial variety


a) Trochoid
b) Ellipsoid
c) Condylar
d) Saddle

Additional Questions
74. Nerve if Wrisberg carries (PGIC)
a) Motor fibres
b) Sensory fibres
c) Secretory fibres
d) Parasympathetic fibres
e) Sympathetic fibres
Ans. a) Motor fbres; b) Sensory fibres; c) Secretory fibres; d) Parasympathetic fibres.
Explanation: Nerve of Wrisberg (nervus intermedius) carry all the components of facial nerve except the
somatic motor fibres to the second pharyngeal arch (facial expression) muscles. Hence it carries
parasympathetic secreto-motor (GVE) fibres to the glands like lacrimal, palatine salivary glands etc. It
also carries sensory (GSA) fibres from the external ear canal. Taste (SVA) sensory fibres from palate and
anterior tongue are also carried along this nerve.

75. A patient has a dry eye and reduced nasal secretions. The location of a lesion might be in the
a) Otic ganglion
b) Pterygopalatine ganglion
c) Ciliary ganglion
d) Superior cervical ganglion
Ans. b) Pterygopalatine ganglion.
Explanation: Greater petrosal nerve (facial nerve branch) carries secretomotor fibres to the pterygo-
palatine ganglion which sends post-ganglionic fibres (along the trigeminal nerve branches) to supply
lacrimal, nasal and palatine glands. A lesion in pterygopalatine ganglion reults in dryness of eye, nose,
palate etc.
PG-DIAMS ANATOMY 120

76. Lacrimal secretions are decreased when facial nerve injury occurs at the following site
a) Middle ear
b) Mastoid foramen
c) Geniculate ganglion
d) Sphenopalatine ganglion
Ans. c) Geniculate ganglion.
Explanation: This question specifically mentions facial nerve injury, hence pterygopalatine(sphenopalatine)
ganglion cannot be the answer, since it is not in the course of facial nerve. Here the answer is geniculate
ganglion. Lesion of facial nerve at the geniculate ganglion compromises the secreto motor fibres towards
the lacrimal, nasal and palatine glands, leading to dryness in the areas.
77. All is true about chorda tympani EXCEPT
a) Facial nerve branch given in temporal bone
b) Carries post-ganglionic parasympathetic fibres
c) Carries secretomotor fibres to sublingual & submandibular salivary gland
d) Joins lingual nerve in infratemporal fossa
Ans. b) Carries post-ganglionic parasympathetic fibres.
Explanation: Chorda tympani nerve is the third branch of facial nerve (in the facial canal), given in the
middle ear cavity (temporal bone), it joins the lingual nerve in the infra-temporal fossa and reaches the
submandibular ganglion. Pre-ganglionic parasympathetic secretomotor fibres are carried by chorda
tympani nerve, synapse in the sub-mandibular ganglion, the post-ganglionic fibres pass along the lingual
nerve (branch of mandibular; trigeminal nerve) to supply the salivary glands (submandibular & sublingual).
78. Which of the following do NOT supply submandibular gland
a) Lingual nerve
b) Chorda tympani
c) Sympathetic plexus
d) Auriculotemporal nerve
Ans. d) Auricuotemporal nerve.
Explanation: Auriculotemporal nerve (a branch of mandibular; trigeminal) carries the post-ganglionic para-
sympathetic secretomotor fibres to supply the parotid (and not submandibular) salivary gland.
79. Parasympathetic secretomotor fibers to parotid come from all EXCEPT
a) Otic ganglion
b) Greater petrosal nerve
c) Auriculotemporal nerve
d) Tympanic plexus
Ans. b) Greater petrosal nerve.
Explanation: Parotid salivary gland is supplied by lesser petrosal nerve (and not greater petrosal nerve).
Inferior salivatory nucleus in the lower pons send preganglionic parasympathetic fibres along the
tympanic branch of glossopharyngeal nerve towards the tympanic plexus (in the middle ear cavity).
Lesser petrosal nerve carry pre-ganglionic fibres further from the tympanic plexus to the otic ganglion.
Otic ganglion send the post-ganglionic fibres along the auriculotemporal nerve (branch of mandibular;
trigeminal) to supply the parotid salivary gland.
80. Nerve supply to platysma is
a) Ansa cervicalis
b) Marginal mandibular branch of facial nerve
c) Cervical branch of facial nerve
d) Mandibular nerve
Ans. c) Cervical branch of facial nerve.
Explanation: Platysma muscle develops in second pharyngeal arch (nerve:facial) and is present in the
neck (cervical) region hence, is supplied by cervical branch of facial nerve.
81. The culprit muscle in sleep apnea syndrome is
a) Hyoglossus
b) Genioglossus
c) Posterior cricoarytenoid
d) Lateral cricoarytenoid
Ans. b) Genioglossus.
PG-DIAMS ANATOMY 121

Explanation: In sleep apnoea syndrome genioglossus muscle may not stay active during sleep and
tongue has the tendency to fall back into the respiratory pathway, leading to difficulty in breathing, which
wakes up the patient frequently during sleep.
82. Action of genioglossus
a) Elevation
b) Protrusion
c) Depression
d) Push the tongue towards midline
Ans. b) Protrusion > c) Depression and d) Push the tongue to midline.
Explanation: Genioglossus muscle pulls the tongue anterior and medial, thus protrusion in midlline
occurs. If one genioglossus muscle acts alone, the tip of the tongue deviates to the contralateral side, since
the medial vector is not being cancelled. Left genioglossus muscle acting alone turns the tip of tongue to
the right. Genioglossus muscle also pulls the tongue inferior (depression).
83. Incorrect statement(s) about tongue is/are (PGIC)
a) Facial nerve supplies fungiform papillae
b) Glossopharyngeal nerve supplies circumvallate papillae
c) Posterior most tongue develop from third pharyngeal arch
d) Genioglossus causes tongue protrusion
e) Blood supply is lingual artery
Ans. c) Posterior most tongue develop from third pharyngeal arch.
Explanation: Posterior most tongue develops in fourth pharyngeal arch and taste sensation is carried
by the superior laryngeal nerve (vagus branch). Fungiform papillae are present at the anterior aspect of
tongue and the taste sensation from the anterior 2/3 of tongue is carries by the chorda tympani (facial
nerve) branch. Posterior 1/3 of tongue develops in third pharyngeal arch and glossopharyngeal nerve
supplies the region (along with the circumvalate papillae). Genioglossus muscle takes the tongue
anterior, medial and inferior. Lingual artery is a branch of external carotid artery and supplies the tongue.
84. Ansa cervicalis innervates the following EXCEPT
a) Superior belly of omohyoid
b) Sternohyoid
c) Inferior belly of omohyoid
d) Thyrohyoid
Ans. d) Thyrohyoid.
Explanation: Ansa cervicalis supplies the anterior neck muscles, including strap muscles. Geniohyoid and
thyro-hyoid muscles (attaching to the hyoid bone) are supplied by C-1 fibres carried by hypoglossal nerve
(and not by ansa cervicalis).
]

85. Greater part of the auricle is supplied by


a) Auriculotemporal nerve
b) Lesser occipital nerve
c) Greater auricular nerve
d) Auricular branch of vagus
Ans. c) Greater auricular nerve.
Explanation: Greater part of auricle is supplied by greater auricular nerve (branch of the cervical plexus).
It innervates the lower part of auricle (including ear lobule) and supply posteromedial and posterolateral
aspect both. Lesser occipital nerve supplies upper and posterior part of the auricle (especially the helix).
Anterior part of the auricle (including tragus) is supplied by auriculo-temporal nerve (mandibular;
trigeminal). Facial nerve carries fibres of auricular branch of vagus to supply the concha and most of the
area around the auditory meatus.
PG-DIAMS ANATOMY 122

86. Trachea begins at which level


a) Upper border of thyroid cartilage
b) Lower border of thyroid cartilage
c) Upper border of cricoid cartilage
d) Lower border of cricoid cartilage
Ans. d) Lower border of cricoid cartilage.
Explanation: Trachea and oesophagus begin at the lower border of cricoid cartilage of larynx (at the
lower border of C-6 vertebra).
87. Nerve supply to the larynx mucosa is
a) External laryngeal and recurrent laryngeal
b) Internal laryngeal
c) External laryngeal
d) Superior laryngeal
Ans. b) Internal laryngeal nerve.
Explanation: Larynx mucosa till the vocal cords is supplied by the internal laryngeal nerve (branch of
superior laryngeal nerve); below the vocal cord it is suppled by the recurrent laryngeal nerve. External
laryngeal nerve is a motor nerve to supply the tensor of the vocal cord – cricothyroid muscle.
88. Muscle relaxants are used routinely during anesthesia with resultant closure of the vocal folds.
Laryngeal intubation by the anesthesiologist is necessary because which of the following
muscle is unable to keep the glottis open
a) Cricothyroid muscle
b) Lateral cricoarytenoid muscles
c) Posterior cricoarytenoid muscles
d) Thyroarytenoid muscle
Ans. c) Posterior cricoarytenoid muscle.
Explanation: Posterior crico-arytenoid muscle is the safety muscle of larynx to (abduct) open the vocal
cords, if paralysed, leads to difficulty in breathing, and hence, endotracheal intubation is a pre-requisite
before giving muscle relaxants.
89. During thyroid surgery, a nerve coursing along the superior thyroid artery is injured. What can
be the possible consequence(s) (PGIC)
a) Loss of sensation above the vocal cords
b) Loss of sensation below the vocal cords
c) Paralysis of crico-thyroid muscle
d) Paralysis of posterior crico-arytenoid muscle
e) Loss of sensation in pyriform fossa
Ans. c) Paralysis of cricothyroid muscle.
Explanation: Superior thyroid artery is accompanied by the external laryngeal nerve (branch of superior
laryngeal nerve) to supply the cricothyroid muscle. Pyriform fossa is supplied by the superior laryngeal
nerve.
90. Constrictions of oesophagus when measured from upper incisors are present at
a) 15cm, 20 cm, 40 cm
b) 15 cm, 25 cm, 40 cm
c) 20cm,30 cm,40 cm
d) 30cm, 40 cm, 60 cm
Ans. b) 15cm, 25cm, 40 cm.
Explanation: According to Bailey and Love Surgery, oesophagus has three narrowings: 15
(cricopharyngeal), 25 (aorta-bronchial), 40 (diaphragmatic) cm from the upper incisors.

91. All lie at the vertebra level C-6 EXCEPT


a) Junction of pharynx with oesophagus
b) Junction of larynx with trachea
c) Cricoid cartilage
d) Isthmus of thyroid
Ans. d) Isthmus of thyroid.
Explanation: At the lower level of cricoid cartilage (at C-6 vertebra), pharynx becomes oesophagus and
larynx becomes trachea. Isthmus of thyroid gland lies below C-6 level, in front of the trachea (ring 2&3).
PG-DIAMS ANATOMY 123

92. If a benign tumour is found where the common carotid artery usually bifurcates, it would be
located at the level of the
a) Cricoid cartilage
b) Angle of the mandible
c) Superior border of the thyroid cartilage
d) Jugular notch
Ans. c) Superior border of thyroid cartilage.
Explanation: Carotid body tumour lies at the bifurcation of common carotid artery at the superior border
of thyrod cartilage.
93. All of the following are branches of subclavian artery EXCEPT
a) Vertebral artery
b) Thyrocervical trunk
c) Subscapular artery
d) Internal thoracic artery
Ans. c) Subscapular artery.
Explanation: First part of subclavian artery gives thre branches: V(Vertebral), I (Internal thoracic artery),
T(thyro-cervical trunk). Sub-scapular artery is a branch of axillary artery and goes under the scapula.

94. Inferior thyroid artery is a branch of and is related to


a) External carotid artery; superior laryngeal nerve
b) Internal carotid artery; superior laryngeal nerve
c) Thyrocervical trunk; recurrent laryngeal nerve
d) Brachio-cephalic trunk; recurrent laryngeal nerve
Ans. c) Thyrocervicak trunk; recurrent laryngeal nerve.
Explanation: Inferior thyroid artery is a branch of thyrocervical trunk (first part of subclavian artery) and is
accompanied by the recurrent laryngeal nerve to supply the thyroid gland. During thyroid gland surgery,
recurrent laryngeal nerve might get damaged leading to paralysis of larynx muscle (except cricothyroid) and
laryngeal anaesthesia below the vocal cords.
95. Which structure passes through foramen magnum
a) Internal Carotid Artery
b) Sympathetic chain
c) Hypoglossal Nerve
d) Vertebral Artery
Ans. d) Vertebral artery.
Explanation: The two vertebral arteries enter the foramen magnum to enter the cranial cavity and join to
form the basilar artery at the ponto-medullary junction. Internal carotid artery and hypoglossal nerve has
one canal each to enter the cranial cavity. Sympathetic chain do not enter the cranial cavity, it begins
below the foramen magnum and terminates at the coccyx level.
96. Thoracic duct opens into
a) Subclavian vein
b) Brachiocephalic vein
c) Internal jugular vein
d) Jugulo-subclavian venous angle
Ans. d) Jugulo-subclavian angle.
Explanation: Thoracic duct opens into jugulosubclavian vein junction on the left side of the neck. It may
occasionally open into internal jugular vein or subclavian vein itself. Right lymphatic duct drains the
lymphatics of right upper quadrant of the body and opens into the right venous angle.

97. True about pharyngo-tympanic tube is/are


a) 36 mm in length
b) 1/3 cartilaginous and 2/3 bony
c) Runs antero-medially making an angle of 30°with the sagittal plane
d) Tensor veli palati opens it
e) Narrowest diameter is at the isthmus
Ans. a) 36mm in length; d) Tensor veli palati opens it; e) Narrowest lumen is at the isthmus.
Explanation: Eustachian (pharyngo-tympanic) tube has a length of 36mm. It communicates the middle
ear cavity with the naso-pharynx. Lateral 1/3 (12mm) is bony and begins at the anterior wall of middle ear
PG-DIAMS ANATOMY 124

cavity. Medial 2/3 (24mm) is made up of elastic cartilage and opens in the naso-pharynx, behind the
inferior turbinate of nasal cavity.
 It runs anterior, inferior and medial at an angle of 45°with the sagittal plane and 30°with the horizontal.
It is opened by dilator tubae (tensor veli palatini) and aided by salpingopharyngeus. Levator veli
palatini might allow passive opening.
 The diameter of the tube is greatest at the pharyngeal orifice, least at the junction of the two parts (the
isthmus), and widens again towards the tympanic cavity.
 Arteries to the pharyngotympanic tube arise from the ascending pharyngeal branch (external acrotid
artery branch), and from the middle meningeal artery & the artery of the pterygoid canal (maxillary
artery branches). The veins of the pharyngotympanic tube usually drain to the pterygoid venous
plexus.
 It is supplied by tympanic plexus (which itself is chiefly contributed by glossopharyngeal nerve).

98. Which of the following is condylar synovial joint


a) First carpo-metacarpal
b) Radio-carpal
c) Intercarpal
d) Metacarpo-phalangeal
Ans. d) Metacarpo-phalangeal > b) Radio carpal.
Wrist Explanation: This is a wrong question, because it has double answer. Still you may give preference
to metacarpo-phalangeal joint. Wrist (radiocarpal) joint and knuckle (metacarpo- phalangeal) joint, both
are structurally condylar but functionally ellipsoid synovial joint. First carpo-metacarpal joint is a saddle
synovial joint for the movement of thumb opposition. Intercarpal joints are plane synovial joints.

High Yield Facts


 Stylopharyngeus muscle develops in third pharyngeal archg and is supplied by glossopharyngeal
nerve.
 Posterior cricoarytenoid muscle abducts the true vocal cords by moving the muscular portion of
arytenoid cartilage of larynx.
 Ascending pharyngeal artery is a branch of external carotid artery. External carotid artery gives 8
branches - Superior thyroid artery, Lingual artery, Facial artery, Occipital artery, Posterior auricular
artery, Ascending pharyngeal artery, Maxillary artery, Superficial temporal artery.
 Portal system is a system of vessels in which blood collected from one capillary network passes
through a large vessel and then a second capillary network before it returns to the systemic
circulation – as in the hypophyseal portal system blood from the hypothalamic capillaries passes
through the hypophyseal portal veins and then the pituitary capillary sinusoids to reach the
hypophyseal veins.
 Middle ear (tympanic) cavity is located within the temporal bone and communicates with the
nasopharynx via the auditory tube. The cavity is supplied by tympanic plexus (chief innervation
from the glossopharyngeal nerve). Chorda tympani branch of facial nerve is given in the middle
ear cavity. The footplate of the stapes send sound vibrations into the oval window, creating a
traveling wave in the perilymph-filled scala vestibuli. Tensor tympani (mandibular; trigeminal nerve)
and stapedius (facial nerve) muscles present in the cavity dampen vibrations of the ossicular chain,
thus protecting the cochlea from loud low-frequency sounds (<1000 Hz).
 Risorius is a muscle of facial expression (Arch: II; nerve supply: facial nerve). It retracts the angle
of the mouth to produce grinning (insincere looking smile).
PG-DIAMS ANATOMY 125

 Recurrent tentorial nerve (a branch of ophthalmic division of trigeminal nerve) supplies tentorium
cerebelli.
 Sensory supply to tonsil is by glossopharyngeal nerve and additional supply by lesser palatine
nerve (Trigeminal; maxillary nerve). Tonsillar pathology may be accompanied by pain referred to the
ear, due to common nerve supply (glossopharyngeal nerve).
 Waldeyer’s ring is an arrangement of MALT (mucosa-associated lymphoid tissue) which surrounds
the openings into the digestive and respiratory tracts. It is made up antero-inferiorly by the lingual
tonsil, laterally by the palatine and tubal tonsils, and postero-superiorly by the nasopharyngeal
tonsil.
 Cervical oesophagus is supplied by inferior thyroid artery.
 Suprameatal triangle lies over the mastoid antrum.
PG-DIAMS ANATOMY 126

Section 5. Back

Back - Embryology
1. In a neonate, spinal cord ends at
a) Lower border of T12
b) Lower border of L I
c) Upper border of L 3
d) Lower border of L 3
 In a neonate, at birth, spinal cord extends till the upper border of L-3 vertebra. It takes less than two
months (post birth) to reach the lower border of L-1 (Gray’s Anatomy).

2. Following are the various structures related to spinal cord and their respective terminal extent.
Choose the WRONG pair
a) Adult spinal cord: Transpyloric plane
b) Pia mater: Coccyx
c) Duramater: S2 vertebra
d) Arachnoid sheath: S2 vertebra
PG-DIAMS ANATOMY 127

3. During a procedure to remove cerebrospinal fluid from the subarachnoid space below the end of
the spinal cord, the needle was advanced too far and penetrated the ligament forming the anterior
border of the vertebral canal. Which of the following ligaments, not normally pierced during this
procedure, was accidentally penetrated
a) Anterior longitudinal
b) Ligamentum flava
c) Posterior longitudinal
d) Supraspinous
 In lumbar puncture supraspinous and interspinous are always punctured, ligamentum flava may be
punctured (if the needle is not in the midline); posterior longitudinal ligament is punctured accidently
and anterior longitudinal ligament can never be punctured.
 The spinal cord terminates at approximately the L1 vertebral level in 94% of individuals. In the remaining
6%, the conus extends to the L2-L3 interspace. LP is therefore performed at or below the L3-L4
interspace. A line drawn between the posterior superior iliac crests, which corresponds closely to the
level of the L3-L4 interspace is a guiding landmark.

 In lumbar puncture needle penetrates duramater and arachnoid mater to reach CSF space for injecting
spinal anaesthesia.
 The needle passes through skin → superficial fascia → supraspinous ligament → interspinous ligament
→ ligamentum flavum → epidural space → dura mater → arachnoid → subarachnoid space containing
CSF.
 Internal vertebral venous plexuses lie within the vertebral canal in the epidural space.
4. Disc herniation between L4 and L5 involves nerve root
a) L- 2
b) L- 3
c) L- 4
d) L- 5
PG-DIAMS ANATOMY 128

 In slip disc, a simple formulae to derive the affected nerve root is to add 1 to the upper vertebrae
number (valid in cervical and lumbar vertebrae region). Here, L4 (+1) = L5 nerve root is involved. Slip
disc is rarely seen in thoracic region.

 Spinal cord has a cervical enlargement at spinal segment C3-T2, which contributes to brachial plexus
and supply the upper limb.
 The lumbar enlargement is at spinal segment L1-S3, which gives fibres for the lumbo-scaral plexus to
supply the lower limb. Lumbar enlargement at the level of 9-12 thoracic vertebra.
 The nerve roots exit at a level above their respective vertebral bodies in the cervical region (e.g., the
C7 nerve root exits at the C6-C7 level) and below their respective vertebral bodies in the thoracic and
lumbar regions (e.g., the T1 nerve root exits at the T1-T2 level). The cervical nerve roots follow a short
and horizontal intraspinal course before exiting. By contrast, the lumbar nerve roots follow a long and
oblique course and can be injured anywhere from the upper lumbar spine to their exit at the
intervertebral foramen.
 Pain-sensitive structures of the spine include the periosteum of the vertebrae, dura, facet joints, annulus
fibrosus of the intervertebral disk, epidural veins and arteries, and the posterior longitudinal ligament.
PG-DIAMS ANATOMY 129

Herniated Compressed Dermatome Muscles Movement Nerve and Reflex


Disc Nerve Root Affected Affected Weakness Involved
between
C4 and C5 C5 C5 Deltoid Abduction of arm Axillary nerve
Lateral surface of biceps jerk
the arm
C5 and C6 C6 C6 Lateral surface Biceps Flexion of forearm Musculocutaneous
of the forearm Brachialis Supination/pronation Nerve
Thumb Index Brachioradialis biceps reflex
finger brachioradialis
(supinator) reflex
C6 and C7 C7 C7 Triceps Extension of for arm Radial nerve
Middle finger Wrist extensors Extension of wrist triceps jerk
L3 and L4 L4 L4 Quadriceps Extension of Knee Femoral nerve
Medial surface of knee jerk
the leg
L4 and L5 L5 L5 Tibialis anterior Dorsiflexion of ankle Common fibular
Lateral surface of Extensor hallucis ( patient cannot Nerve
leg longus stand on heels) No reflex loss
Dorsum of foot Extensor Extension of toes
Big toe digitorum longus

L5 and S1 S1 S1 Gastrocnemius Plantar flexion of Tibial nerve


Heel Soleus ankle( patient ankle jerk
Little toe cannot stand on
toes)
Flexion of toes

Additional Questions
5. Primary curvatures of vertebral column are
a) Cervical & lumbar
b) Thoracic & sacral
c) Cervical & thoracic
d) Thoracic & lumbar
Ans. b) Thoracic & sacral.
Explanation: Primary curvatures are present since birth and are concave anteriorly (kyphosis).
Secondary curvatures are acquired after birth and are convex (lordosis) anteriorly. Lumbar lordosis
increases during pregnancy due to centre of gravity shifting more anterior (additional load of conceptus).

6. Numerical variation is least in which vertebrae


a) Cervical
b) Thoracic
c) Lumbar
d) Coccygeal
Ans. a) Cervical.
Explanation: The cervical region is reported to be the most constant, the coccygeal the most
variable.The usual grouping formula of 7 cervical, 12 thoracic, 5 lumbar, 5 sacral, and 4 coccygeal
vertebrae is found in only about 20% of individuals studied.
PG-DIAMS ANATOMY 130

7. Typical cervical vertebrae can be differentiated from thoracic vertebra by


a) Triangular vertebral canal
b) Foramen transversarium
c) Superior articular facet directed backward and upwards
d) Large vertebral body
Ans. b) Foramen transversarium > a) Triangular vertebral canal and c) Superior articular facet
directed backward and upwards
Explanation: Cervical vertebrae are characterized by presence of foramen transversarium. The shape of
the body is oval and the vertebral canal is triangular. The superior articular facets are directed backward
and upwards. Cervical vertebrae has comparatively small body, thoracic vertebrae large and lumbar
vertebrae has the largest size.

 C-1 (Atlas) vertebra has no body. C-2 (Axis) vertebra has a vertical projection (dens/odontoid
process), which articulates with atlas to form atlanto-axial joint (pivot synovial) for rotatory movement
of ‘NO’ at neck region.

8. Vertebral artery passes through foramen transversaria of


a) All cervical vertebrae
b) 2nd to 5th cervical vertebrae
c) All except 1st cervical vertebra
d) All except 7th cervical vertebra
Ans. d) All except 7th cervical vertebra.
Explanation: Vertebral artery passes through the upper six foramen transversaria in most of the
population. Vertebral artery passes through the foramen transversarium of C-7 vertebra in 2% population.

9. Lumbar puncture is done at the vertebral level


a) L-2
b) L-3
c) L-4
d) L-5
Ans. c) L-4.
Explanation: Lumbar puncture for (CSF sample or spinal anaesthesia) is carried out at L – 3 and L-4
vertebra level (the highest point of iliac crest). It may also be done at L -4 and L-5 vertebra level.
PG-DIAMS ANATOMY 131

10. All is true about spinal cord in vertebral canal EXCEPT


a) Transverse section is oval at cervical region
b) Cervical enlargement is present at C3-T2
c) Lumbar enlargement at L1-S3
d) Lumbar enlargement at 9-12 thoracic vertebra
Ans. a) Transverse section is oval at cervical region.
Explanation: Transverse section of spinal cord is triangular at cervical region and oval in thoracic region.
11. Which spinal segment corresponds to fourth thoracic vertebra
a) T- 2
b) T- 4
c) T- 6
d) T- 8
Ans. c) T – 6.
Explanation: In the upper thoracic region, we need to add 2 to the vertebral number to get the segment
number of spinal cord.

12. Boundary of triangle of auscultation is NOT formed by (AIIMS)


a) Scapula
b) Trapezius
c) Latissimus dorsi
d) Serratus anterior
Ans. d) Serratus anterior.
Explanation: Serratus anterior is inserted on the medial border of scapula but lies anterior to scapula
hence is not in the triangle of auscultation.
 Triangle of Auscultation is bounded by 2 muscles and scapula. Superiorly – Trapezius, Inferiorly –
Latissimus dorsi and Laterally – medial wall of Scapula. Rib 7 and Rhomboideus major lie in the floor of
the triangle.

 Applied anatomy: Since, minimal muscle fibres lie over the triangle, auscultation by stethoscope is
better over this triangle, especially, the sounds of swallowed fluids. Cardiac end of the stomach lies
deep to this triangle.

High Yield Facts


 C7 vertebra has the most prominent spinous process.
PG-DIAMS ANATOMY 132

Section 6. THORAX

Embryology
 Cardiovascular tube develops from the ventral visceral (splanchnic) lateral plate mesoderm under
the influence of multiple signals, including those derived from neural crest cells.
 The myocardial cells secretes an extracellular matrix rich in hyaluronic acid (cardiac jelly) which
accumulates within the endocardial cushions, which are precursors cardiac valves.
PG-DIAMS ANATOMY 133

 The crista terminalis is the junction of the smooth and rough (trabeculated) part of the right atrium. It is
a vertical muscular ridge running anteriorly along the right atrial wall from the opening of the SVC to the
opening of the IVC, providing the origin of the pectinate muscles and is indicated externally by the sulcus
terminalis.
 SA node is present in the right atrium at the opening of superior vena cava, at the upper end of crista
terminalis. It does not occupy the full thickness of the right atrial wall from epicardium to endocardium in
humans, but rather sits as a wedge of specialized tissue subepicardially.
 Pulmonary veins (total four) develop from the left atrial wall.

Heart tube derivatives:


Embryonic structure Adult derivative
Ascending Aorta
Truncus arteriosus
Pulmonary trunk
Smooth part of right ventricle (conus arteriosus)
Bulbus cordis
Smooth part of left ventricle (aortic vestibule)
Trabeculated part of right ventricle
Primitive ventricle
Trabeculated part of left ventricle
Trabeculated part of right atrium
Primitive atrium
Trabeculated part of left atrium
Right horn: Smooth part of right atrium (sinus venarum)
Sinus venosus
Left horn: Coronary sinus and oblique vein of left atrium
PG-DIAMS ANATOMY 134

1. The structure present anterior to transverse pericardial sinus is


a) Inferior vena cava
b) Superior venae cava
c) Aorta
d) Pulmonary artery
 As the heart tube folds, a space develops between arterial and venous end – transverse pericardial
sinus. Anterior to the sinus are two arteries derived from truncus arteriosus: ascending aorta and
pulmonary trunk (and not pulmonary artery). A finger can be put into the sinus to pull the two major
arteries and a ligature put around, during cardio-thoracic surgeries. Superior vena cava lies posterior
to the sinus and bifurcation of pulmonary trunk is superior to it,
PG-DIAMS ANATOMY 135

2. Transposition of great vessels occurs due to


a) Failure of cono-truncal ridge to fuse and descend towards the ventricles
b) Anterior displacement of aortico-pulmonary septum
c) Aortico-pulmonary septum not following its spiral course
d) Migration of neural crest cells towards truncal & bulbar ridges
 AP septum anomalies like PTA, TGV and TOF present with right to left shunt, blood reaches
systemic circulation without proper oxygenation, hence leading to cyanosis.
 If the AP septum is not spiral, the great vessels are not spiral and they open in the opposite ventricles
(transposition of great vessels). The aorta arises from the right ventricle, and the pulmonary artery
emerges leftward and posteriorly from the LV (two separate parallel circulations); some communication
between them must exist after birth to sustain life. Most patients have an interatrial communication,
two-thirds have a patent ductus arteriosus, and about one-third have an associated VSD.
 Failure of cono-truncal ridge to fuse and descend towards the ventricles result in absence of AP
septum – Persistent truncus arteriosus.
 Anterior displacement of aortico-pulmonary septum – Tetralogy of Fallot. It consists of: pulmonic
stenosis, a wider aorta (which over-rides the right and left ventricles), VSD and right ventricular
hypertrophy. Right-to-left shunt across the VSD results in cyanosis at an early stage.
PG-DIAMS ANATOMY 136

 Fetal circulation: Oxygenated blood travels from the placenta along the left umbilical vein. Most
blood by-passes the liver in the ductus venosus joining the inferior vena cava and then travelling to
the right atrium.
 Most of the blood passes through the foramen ovale into the left atrium so that oxygenated blood can
enter the aorta and reach the brain at earliest. The remainder goes through the right ventricle with
returning systemic venous blood into the pulmonary trunk. The unexpanded lungs present high
resistance to flow so that blood in the pulmonary trunk tends to pass down the low-resistance ductus
arteriosus into the aorta.
 Blood returns to the placenta via the umbilical arteries (branches of the internal iliac arteries).
 At birth, when the baby breathes, the left atrial pressure rises, pushing the septum primum against the
septum secundum and closing the foramen ovale. Blood flow through the pulmonary artery increases
and becomes poorly oxygenated as it now receives systemic venous blood. Pulmonary vascular
resistance is abruptly lowered as lungs inflate and the ductus arteriosus is obliterated over the next
few hours to days.
 At removal of placenta, ligation of the umbilical cord causes thrombosis of the umbilical arteries
(becomes medial umbilical ligaments), vein (becomes ligamentum teres) and ductus venosus
(Becomes ligamentum venosum).

Heart Arteries
PG-DIAMS ANATOMY 137

 Right coronary artery arises from the anterior aortic sinus of ascending aorta and left coronary
artery from left posterior.
 The term ‘dominant’ is used to refer to the coronary artery that gives the posterior interventricular
artery, which supplies the posterior part of the ventricular septum and often part of the posterolateral
wall of the left ventricle. The dominant artery is usually the right (60- 65%).
 The first branch of right coronary artery is called as conus artery. This is sometimes termed a ‘third
coronary’ artery (may arise separately from the anterior aortic sinus in 36% of individuals or may be
a branch of left coronary artery occasionally).
 The sinu-atrial node is supplied by the right (51–65%) or left (35–45%) coronary arteries, and fewer
than 10% of nodes receive a bilateral supply. The atrioventricular node is supplied by the right (80–
90%) or left (10–20%) coronary arteries.

3. The right coronary artery is the main supply to all of the following parts of the conducting system
in the heart EXCEPT
a) SA Node
b) AV Node
c) AV Bundle
d) Right bundle branch
PG-DIAMS ANATOMY 138

Veins: Thorax

 The azygos vein is formed by the union of the right ascending lumbar and right subcostal veins. Its
lower end is connected to the IVC. It arches over the root of the right lung and empties into the SVC.
 The hemiazygos vein is formed by the union of the left subcostal and ascending lumbar vein, receives
the 9th, 10th, and 11th posterior intercostal veins, and enters the azygos vein. Its lower end is
connected to the left renal vein. The accessory hemiazygos vein receives the fifth to eighth posterior
intercostal veins and terminates in the azygos vein.
 The superior intercostal vein is formed by the second, third, and fourth intercostal veins and drains
into the azygos vein on the right and the brachiocephalic vein on the left.
4. IVC obstruction presents with
a) Oesophageal varices
b) Haemorrhoids
c) Para-umbilical dilatation
d) Thoraco-epigastric dilatation
Sternal Angle & Mediastinum
5. Arch of aorta begins at the vertebra level
a) T2
b) T3
c) T4
d) T5
PG-DIAMS ANATOMY 139

 The sternal angle (of Louis) is the junction between the manubrium and the body of the sternum and
is located at the level where the second ribs articulate with the sternum, the aortic arch begins and
ends, and the trachea bifurcates into the right and left primary bronchi.
 Trachea bifurcates at the upper border of T-5 vertebra (Gray’s Anatomy).
 The posterior mediastinum contains the esophagus, thoracic aorta, azygos and hemiazygos veins,
thoracic duct, vagus nerves, sympathetic trunks, and splanchnic nerves.
6. Trachea bifurcates at the vertebra level
a) T2
b) T3
c) T4
d) T5
 Trachea bifurcates at the upper border of T-5 vertebra, in deep inspiration may be pulled down to
T-6 vertebrae level. In a cadaver, it terminates at T-4 level.

BPS (Broncho-Pulmonary Segments)


PG-DIAMS ANATOMY 140

 The right principal bronchus is wider, shorter, and more vertical than the left principal bronchus, and
therefore, is where large aspirated objects commonly lodge. • The right lower lobar bronchus is most
vertical, most nearly continues the direction of the trachea, and is larger in diameter than the left, and
therefore, is where small aspirated objects commonly lodge and the fluid aspirations reach the right
lower lobes more often.
 A bronchopulmonary segment is defined by a segmental bronchus and accompanying segmental
artery (branch of pulmonary artery) that lie centrally, whereas the veins (branch of pulmonary vein) are
intersegmental and lie at the margins of bronchopulmonary segments.
7. A bed-ridden patient on liquid diet develops aspiration pneumonia. Which of the following is
bronchopulmonary segment is most likely affected
a) Posterior of right upper lobe
b) Inferior lingular of left upper lobe
c) Apical of right lower lobe
d) Posterior of right lower lobe

 In erect posture (sitting or standing) aspirated material most commonly enters the right lower lobar
bronchus and lodges within the posterior basal bronchopulmonary segment (no. 10) of the right lower
lobe.
 Aspiration in supine posture most commonly involves the right lower lobar bronchus and aspitare
lodges within the superior(apical) bronchopulmonary segment of the right lower lobe.
PG-DIAMS ANATOMY 141

Intercostal Drainage
8. In pleural tap in the mid-axillary line, muscle NOT pierced is
a) External intercostal
b) Serratus anterior
c) Innermost intercostal
d) Transversus thoracis
PG-DIAMS ANATOMY 142

Rib - I

9. All of the following lie between first rib and the apex of the lung EXCEPT
a) Superior intercostal artery
b) First intercostal vein
c) Thoracic duct
d) Sympathetic trunk

Questions: Thorax
10. Which of the following is NOT a boundary of the Koch’s triangle
a) Tendon of Todaro
b) Limbus fossa ovalis
c) Coronary sinus
d) Tricuspid valve ring

 Triangle of Koch : A roughly triangular area on the septal wall of the right atrium, bounded by the
base of the septal leaflet of the tricuspid valve, the anteromedial margin of the orifice of the
coronary sinus, and the tendon of Todaro; it marks the site of the atrioventricular node. In a case of
AV nodal re-entry tachycardia, radiofrequency ablation of this triangular area improves the symptoms.
 Koch’s triangle is usually supplied by right coronary artery.
PG-DIAMS ANATOMY 143

11. In PA view CXR, right border of the heart is contributed by

a) Pulmonary trunk
b) Ascending aorta
c) Right auricle
d) Right ventricle

12. Pleural reflection on midaxillary line is in space


a) 5
b) 6
c) 8
d) 10
PG-DIAMS ANATOMY 144

 The lower border of the lung (midway between inspiration and expiration) crosses sixth rib in the
midclavicular line, eighth rib in the midaxillary line and tenth rib at the lateral border of erector spinae
(paravertebral line).

Additional Questions
13. UNTRUE about cardiac jelly (AIIMS)
a) Secreted by cardiac myocytes surrounding primitive heart tube
b) Found exterior to endothelium
c) Forms myocardium
d) Transforms into the connective tissue of the endocardium
Ans. c) Forms myocardium.
Explanation: Cardiac jelly is secreted by the cardiac myocytes (myocardium) around the endothelial lining
of heart tube, and transforms into the connective tissue of endocardium.

14. Limbus fossa ovalis and floor of fossa ovalis represents


a) Septum Primum
b) Septum secundum
c) Septum primum and septum secundum
d) Septum secundum and septum primum

 The primitive atrium is divided first by a septum primum, which grows down from the superior wall to
the atrio-ventricular cushions; as this fusion occurs, the midportion resorbs in the center forming the
ostium secundum. Rightward of the septum primum, a second septum secundum membrane grows
down from the ventral-cranial wall toward—but not reaching—the cushions, and covering most, but not
all, of the ostium secundum, resulting in a flap of the foramen ovale.
 As the septum primum and septum secundum get fused with each other, foramen ovale in septum
secundum is closed (becomes fossa ovalis), at it’s floor is seen septum primum.
 ASD: Secundum type ASD is the most common ASD. It is caused by either an excessive resorption of
the Septum primum or an underdevelopment and reduced size of the Septum secundum. Primum type
ASD is less common than secundum ASD and results from a failure of the septum premium to fuse with
the endocardial cushions
 Atrial septal defect (ASD) : Fusion between septum primum and septum secundum takes place at
about 3 months after birth. Ostium secundum type: If septum secundum is too short to cover foramen
secundum (in the septum primum), it allows shunting of blood from left to right atrium (Atrial septal
defect). Ostium primum type: If septum primum fails to fuse with endocardial cushions, the defect lies
immediately above the atrioventricular (AV) boundary (may also be associated with a ventricular septal
defect).
PG-DIAMS ANATOMY 145

15. All is true about the development of heart EXCEPT


a) Sinus venarum develop from right horn of sinus venosus
b) Left horn of sinus venosus forms the coronary sinus
c) Fossa ovalis is a remnant of septum secundum
d) Ductus venosus connects portal vein and inferior vena cava
Ans. d) Ductus venosus connects portal vein and inferior vena cava.
Explanation: Ductus venosus connect left umbilical vein with inferior vena cava and becomes ligamentum
venosum of liver in the adults.

16. All of the following pairs for adult derivatives of embryonal structures is correct EXCEPT
a) Umbilical artery: Lateral umbilical ligament
b) Umbilical vein: Ligamentum teres
c) Ductus venosus: Ligamentum venosum
d) Foramen ovale: Fossa ovalis
Ans. a) Umbilical artery: Lateral umbilical ligament.
Explanation: Umbilical arteries become medial umbilical ligaments. Lateral umbilical ligaments are
raised by the inferior epigastric arteries. Median umbilical ligament is raised by urachus attaching to the
apex of urinary bladder.

17. Heart begins to beat in the week


a) 4
b) 5
c) 6
d) 7
Ans. a) 4.
Explanation: Heart beat begins by day 22 post-ovulation and can be detected by doppler ultrasound. It is
week 4 post-ovulation (or fertilization) and week 6 from LMP (Last Menstrual Period).

18. Pulmonary veins develops from


a) 6th aortic arch
b) Primitive left atrium
c) Left common cardinal vein
d) Left vitelline vein
Ans. b) Primitive left atrium.
Explanation: According to some authorities, pulmonary veins develop from the left atrial wall. There is no
consensus about whether the pulmonary vein as a branch from the left atrium obtains a connection to the
lung plexus or the pulmonary vein forms as a solitary vessel in the dorsal mesocardium and is only
secondarily incorporated into the atrium.
19. Anatomical closure of ductus arteriosus occurs at
a) Birth
b) 3-4 days
c) 10 days
d) 30 days
PG-DIAMS ANATOMY 146

Ans. d) 30 days.
Explanation: Physiological closure of ductus arteriosus occurs within 1 – 4 days of birth. Often a small
shunt of blood stays for 24-48 hours in a normal full term infant. At the end of 24 hours (one day), 20 %
ducts are functionally close, 82 % by 48 hours and 100% at 96 hours (4 days). Anatomical closure of
ductus arteriosus oocurs within 2 – 12 postnatal weeks (1 month to 3 months).
20. Anatomical closure of ductus arteriosus occurs at
a) 2 weeks
b) 4 weeks
c) 12 weeks
d) 16 weeks
Ans. c) 12 weeks.
Explanation: Anatomical closure of ductus arteriosus oocurs within 2 – 12 postnatal weeks (1 month to
3 months).

21. Cardiac defects causing right to left shunt, leading to early cyanosis are all EXCEPT
a) Transposition of great vessels
b) Tetralogy of Fallot
c) Patent ductus arteriosus
d) Persistent truncus arteriosus
Ans. c) Patent ductus arteriosus.
Explanation: PDA carries the blood towards the lungs and promotes oxygenation thus, reduces cyanosis.
AP septum anomalies like PTA, TGV and TOF present with right to left shunt, blood reaches systemic
circulation without proper oxygenation, hence leading to cyanosis.
22. Absence of cono-truncal septum gives rise to
a) Tetralogy of Fallot
b) Patent truncus arteriosus
c) Transposition of great vessels
d) Coarctation of aorta
Ans. b) Patent truncus arteriosus.
Explanation: Absence of Aorta Pulmonary (AP) septum leads to persistent (patent) truncus arteriosus.
Conotruncal septum is the other name for AP septum.
23. Pentalogy of Fallot is characterized by
a) Ventricular septal defect
b) Patent ductus arteriosus
c) Atrial septal defect
d) Pulmonary stenosis
Ans. c) Atrial septal defect.
Explanation: Tetrology plus ASD (Atrial Septal Defect) is a feature of pentalogy of Fallot.
24. The base of the heart is formed by
a) Left and right ventricle
b) Left atrium and ventricle
c) Right atrium and ventricle
d) Left and right atrium
PG-DIAMS ANATOMY 147

Ans. d) Left and right atrium


Explanation: Base of the heart is the posterior surface of heart and is mainly contributed by left atrium
and partly right atrium. Diaphragmatic surface of heart is majorly contributed by left ventricle and partly
right ventricle. Anterior sternocostal surface has all the four chmabers participating.

25. The Great cardiac vein lies in which groove


a) Anterior part of right coronary sulcus
b) Posterior part of right coronary sulcus
c) Anterior interventricular groove
d) Posterior interventricular groove
Ans. c) Anterior interventricular groove.
Explanation: Great cardiac vein runs along with the anterior interventricular artery (in anterior
interventricular groove) and next with circumflex artery (in coronary sulcus), to drain eventually into
coronary sinus.

26. Bleeding comes from the vein that is accompanied by the posterior interventricular artery.
Which of the following veins is most likely to be ruptured
a) Great cardiac vein
b) Middle cardiac vein
c) Small cardiac vein
d) Oblique veins of the left atrium
Ans. b) Middle cardiac vein.
Explanation: Posterior interventricular artery is accompanied by middle cardiac vein, which itself
drains into the coronary sinus.

27. Even if thrombosis is present in the coronary sinus, which of the following cardiac veins might
remain normal in diameter
a) Middle cardiac vein
b) Anterior cardiac vein
c) Small cardiac vein
d) Oblique cardiac vein
PG-DIAMS ANATOMY 148

Ans. b) Anterior cardiac vein.


Explanation: Anterior cardiac veins drain directly into the right atrium and not into the coronary sinus,
hence, they might remain normal in coronary sinus thrombosis.

28. Occlusion of the left anterior descending artery will lead to infarction in which area of heart
a) Posterior part of the interventricular septum
b) Anterior wall of the left ventricle
c) Lateral part of the heart
d) Inferior surface of right ventricle

Ans. b) Anterior wall of left ventricle.


Explanation: LAD (Left Anterior Descending) artery is also known a santerio interventricular artery and
runs in the same named groove, supplying anterior 2/3 of interventricular septum lying deep to it and also
the adjacent anterior wall of the left ventricle. Posterior part of the interventricular septum and
inferior surface of right ventricle is supplied by PIVA (Posterio InterVentricular Artery). Left lateral
surface of the heart is suppled by circumflex artery.

29. Cardiac BPS of right lung is


a) Medial
b) Lateral
c) Medial basal
d) Anterior basal
Ans. c) Medial basal.
Explanation: Cardiac BPS of right lung is medial basal segment of lower lobe. This BPS is absent in the
left lung, the space being encroached by the heart.

30. Which is the most superior structure at hilum of left lung


a) Pulmonary vein
b) Pulmonary artery
c) Bronchus
d) Bronchial artery
Ans. b) Pulmonary artery.
Explanation: The arrangement of structures in the hilum of left lung is reebered by the mnemonic ABV
(Atal Bihari Vajpayi) in superior to inferior direction. Artery (pulmonary) → Bronchus (principal) → Vein
(pulmonary). It is the same sequence in right lung as well but with the addition of a bronchus above the
artery (epi-arterial bronchus). In all these structures bronchus is the most posterior structure at the lung
hilum.
PG-DIAMS ANATOMY 149

31. Coronary dominance is determined by


a) Posterior interventricular artery
b) Anterior interventricular artery
c) Circumflex artery
d) Right coronary artery
Ans. a) Popsterior interventricular artery.
Explanation: Coronary dominance is determined by PIVA (Posterior InterVentricular Artery). In about 65%
of the poulation PIVA is given by right coronary artery alone (right cardiac dominance), in 10 % cases it’s
a branch of left coronary artery alone (left cardiac dominance) and in the remaining 25 % it is given by
both (balanced dominance).
32. Posterior interventricular artery is a branch of right coronary artery in most of the people (right
dominance). In 10% population it arises from
a) Circumflex artery
b) Left coronary artery
c) Pulmonary artery
d) Right coronary artery
Ans. a) Circumflex artery.
Explanation: In 10 % population, PIVA is a branch of the circumflex artery, which itself is a branch of left
coronary artery (left cardiac dominance).
33. Trachea lies in which mediastinum
a) Superior
b) Anterior
c) Middle
d) Posterior
Ans. a) Superior.
Explanation: Trachea and arch of aorta lies in the superior mediastinum. Oesophagus lies in the
superior mediastinum, passes through posterior medastinum and eventually enters the abdomen.
34. The order of neurovascular bundle in intercostal space from above to below is: vein-artery-
nerve. This order is NOT observed at rib number
a) 1
b) 2
c) 11
d) 12
Ans. a) 1.
Explanation: In first rib, the vein-artery-nerve order is medial to lateral. SVAN structures are sandwiched
between the lung apex and first rib. S – Sympathetic trunk, V – Vein (posterior intercostal), A – artery
(posterior intercostal), N – Nerve (T – 1).
35. All is true about vertebrae levels EXCEPT
a) Heart lies at T5 – 8 in recumbent position
b) Superior vena cava enters right atrium at T5
c) Azygous vein enters SVC at T4
d) Hemiazygous vein crosses left to right at T5
PG-DIAMS ANATOMY 150

Ans. d) Hemiazygous vein crosses left to right at T5.


Explanation: Hemiazygous vein cross left to right at the level of T- 8 vertebrae, which is also the inferior
extent of heart in supine/recumbent position. IVC enters the heart at the same level after passing through
the central tendon of diaphragm. Azygous vein enters the SVC at T- 4 vertebra level, which then enters
heart at T- 5 level (superior extent of heart).
36. Which of the following veins drains into the brachiocephalic vein
a) Internal thoracic vein
b) Hemiazygos vein
c) Right superior intercostal vein
d) Left superior intercostal vein
Ans. d) Left superior intercostal vein.
Explanation: First posterior intercostal vein on each side drains into brachiocephalic vein. Posterior
intercostal veins of left intercostal space 2, 3 and 4 drains into the left superior intercostal vein, which
itself drains into the left brachiocephalic vein. Internal thoracic vein drains into subclavian vein. Hemi-
azygous vein drains into azygous vein.

37. All is true about phrenic nerve EXCEPT


a) Right is shorter and more vertical
b) Sole motor supply to diaphragm
c) Passes anterior to scalenus anterior
d) Passes posterior to hilum of lung

Ans. d) Passes posterior to the hilum of lung.


PG-DIAMS ANATOMY 151

Explanation: Phrenic nerve passes anterior to the hilum of lungs, vagus nerve passes posterior to it.
Diaphragm receives somatic motor fibers solely from the phrenic nerve; its central part receives sensory
fibers from the phrenic nerve, whereas the peripheral part is supplied by intercostal nerves. Right dome of
diaphragm is at higher level (pushed up by liver) and the left dome of diaphragm is lower (pushed down by
heart).

High Yield Facts


 The true ribs are the first seven ribs (ribs 1 to 7), which attach to the sternum, the false ribs are the lower
five ribs (ribs 8 to 12). Last two ribs (ribs 11 and 12) are also known as floating ribs as their anterior ends
are floating with no bony articulation.
 Diaphragm is the chief muscle of inspiration, the accessory muscles are external, internal (interchondral
part), and innermost intercostal muscles, sternocleidomastoid, levator costarum, serratus anterior,
serratus posterior superior, scalenus, and pectoral muscles.
 Pump Handle movement: Elevation of upper 6 ribs causes sternum to be pushed forward and upward,
which increases the antero-posterior diameter of the thorax and lungs expand (inspiration).
 Bucket handle movement: The lower ribs elevate by swinging upward and laterally leading to an
increase in the transverse (lateral) diameter of the thorax for lung expansion (inspiration).
 Expiration is a largely a passive process caused by the elastic recoil of the lungs. Muscles of expiration
include anterior abdominal, internal intercostal (costal part), and serratus posterior inferior muscles.
 Great cardiac vein accompanies anterior interventricular artery in the anterior interventricular groove.
PG-DIAMS ANATOMY 152

Section 7. Upper Limb

Embryology
1. Limb buds appear at week
a) 3
b) 4
c) 5
d) 6

2. During development, the scapula is formed by which of the following


a) Splanchnic lateral plate mesoderm
b) Neural crest cells
c) Axial mesoderm
d) Somatic lateral plate mesoderm

3. Root value of radial nerve is


a) C3,4,5,6,7
b) C4,5,6,7,8
c) C5,6,7,8; T1
d) C6,7,8; T1,2

Dermatomes and Axial lines


4. Dermatome of thumb and index finger is
a) C5; C6
b) C6; C6
c) C6; C7
d) C7; C7
PG-DIAMS ANATOMY 153

 Upper limbs rotate laterally by 90 degrees, so that the thumb becomes lateral and little finger medial.
The flexor compartment comes anterior and the extensor compartment goes posterior.
 Lower limb rotates medially by 90-degree, so the extensor aspect of the leg faces anteriorly.
 Developmentally, radial artery is pre-axial and ulnar is a post-axial artery.

Brachial plexus
5. Which of the following is a branch from the trunk of brachial plexus
a) Dorsal scapular nerve
b) Long thoracic nerve
c) Nerve to subclavius
d) Suprascapular nerve
 The nerve to subclavius is small and arises near the junction of the fifth and sixth cervical ventral rami.
PG-DIAMS ANATOMY 154

 Brachial plexus is formed by the ventral primary rami of the lower four cervical nerves and the first
thoracic nerves (C5–T1). It has roots & trunks (in the neck), divisions (passing behind clavicle), cords
and branches (in the axilla). It is covered by a prolongation of prevertebral fascia (axillary sheath)
around the nerves in the axilla.
 Two branches are given directly from the roots in the neck: 1. Dorsal scapular nerve (C5), which
supplies rhomboid major & monor levator scapulae muscles. 2. Long thoracic nerve of Bell (C5–
C7), which is given in the neck, enters axilla and descends on the external surface of the serratus
anterior muscle and supplies it.
 Lateral cord gives three branches (LML), medial and posterior cords give 5 branches each. Radial
nerve is a branch of posterior cord (STARS) and supplies posterior (extensor) compartment of upper
limb. Ulnar nerve is a branch of medial cord (UM4) and runs on the ulnar (medial) side of the limb.
Median nerve runs in the midline of the limb and has contributions from both medial and lateral
cords.

Nerve Injuries
PG-DIAMS ANATOMY 155
PG-DIAMS ANATOMY 156

Brachial Plexus – Cords and Branches (Axilla)

 Radial nerve is the largest branch of brachial plexus. It carries all the five root values of brachial
plexus.
 Ulnar nerve carries root value: C-7, 8; (T-1).

Median Nerve Injuries


Nerve involved Cause of injury Clinical features
Median nerve Supracondylar fracture Ape thumb deformity; Benediction hand; Weakness
(humerus); wrist slash in wrist flexion; hand deviates to ulnar side on flexion;
injury; carpal tunnel flexion of index and middle finger is lost; anterior
syndrome abduction, opposition and flexion of thumb is
compromised; sensory loss on the lateral 3 & 1/2
fingers; difficulty in making an ‘O’ with thumb and index
finger;

Hand of Benediction
PG-DIAMS ANATOMY 157

6. Most common nerve damaged in supracondylar fracture is


a) Median
b) Anterior interosseous
c) Radial
d) Ulnar

SUPRA-CONDYLAR FRACTURE WRIST SLASH INJURY

7. Which of the following is the most commonly damaged nerve in wrist slash injury (AIIMS)
a) Median
b) Ulnar
c) Radial
d) Anterior interosseous

8. Which of the following nerve is damaged in wrist slash injury (UPSC)


a) Median
b) Ulnar
c) Median & ulnar
d) Median & radial
PG-DIAMS ANATOMY 158

 Tinel sign is a tingling sensation in the distal end of a limb when percussion is made over the site of a
divided nerve. It indicates a partial lesion or the beginning regeneration of the nerve.
 Phalen test : The size of the carpal tunnel is reduced by holding the affected hand with the wrist fully
flexed or extended for 30 to 60 seconds, or by placing a sphygmomanometer cuff on the involved arm
and inflating to a point between diastolic and systolic pressure; appearance of numbness or
paresthesias indicates carpal tunnel syndrome.
PG-DIAMS ANATOMY 159

9. Carpal tunnel syndrome produces inability to (AIIMS)


a) Abduct the thumb
b) Adduct the thumb
c) Flex the distal phalanx of the thumb
d) Oppose the thumb
PG-DIAMS ANATOMY 160

10. Which of the following muscle has dual nerve supply


a) Flexor digitorum profundus
b) Interossei
c) Palmaris brevis
d) Flexor carpi ulnaris

Radial Nerve Injuries


Nerve involved Cause of injury Clinical features
Radial nerve Crutch palsy; Saturday Loss of extension at multiple joints; Wrist drop (loss of
night palsy; Fracture mid- wrist extension); weakness of supination and finger
shaft humerus extension; sensory loss on arm, forearm and dorsum of
hand

Wrist drop
PG-DIAMS ANATOMY 161

11. Finger drop with no wrist drop is caused by lesion of


a) Radial nerve in the radial groove
b) Posterior interosseous nerve
c) Anterior interosseous nerve
d) Ulnar nerve behind medial epicondyle
Explanation: Injury to posterior interosseous nerve results in paralysis of extensor muscles in the posterior
forearm. Finger drop (loss of finger extension at metacarophalangeal joint) occurs, along with weakning of
wrist extension. Wrist extension is still possible (no wrist drop) because of the functional ECRL (Extensor
Carpii Radialis Longus) muscle, a powerful wrist extensor.

12. All are affected in low radial nerve palsy EXCEPT


a) Extensor carpi radialis longus
b) Extensor carpi radialis brevis
c) Finger extensors
d) Sensation on dorsum of hand
Explanation: Low radial nerve injuries occur around the elbow joint (for e.g., supracondylar fracture) and
may spare the ECRL (Extensor Carpi Radialis Longus) muscle. All the muscles supplied by radial nerve
distal to the lesion get paralysed.

13. Injury to radial nerve in lower part of spiral groove may result in all EXCEPT
a) Spare nerve supply to extensor carpi radialis longus
b) Results in paralysis of anconeus muscle
c) Leaves extension at elbow joint intact
d) Weakens supination movement
Explanation: Injury to radial nerve in lower part of radial groove results in paralysis (not sparing) of ECRL
(Extensor Carpi Radialis Longus). The muscle spared is triceps, and elbow extension is still possible.
Anconeus may (or may not) be paralysed, depending upon the involvement of the branch in the fracture.
Supinator muscle is paralysed , hence there will be difficulty in supination.
PG-DIAMS ANATOMY 162

Arteries – Upper Limb

 Thyrocervical trunk is a branch from the first part of subclavian artery. It gives three branches SIT: S –
Supra-scapular artery; I – Inferior thyroid artery and T – Transverse cervical artery.
 Axillary artery has three parts and 6 branches. First part (1 branch – superior thyroid artery); second
part (2 branches – thoraco-acromial and lateral thoracic artery) and third part (3 branches – anterior
and posterior circumflex humeral arteries and subscapular artery).
PG-DIAMS ANATOMY 163

Scapular Anastomosis
14. In a subclavian artery block at the outer border of first rib all of the following arteries help in
maintaining the circulation to upper limp EXCEPT
a) Thyrocervical trunk
b) Suprascapular
c) Subscapular
d) Superior thoracic

15. Which branch of subclavian contributes to scapular anastomosis


a) Vertebral
b) Internal thoracic
c) Thyrocervical truck
d) Dorsal scapular
 Dorsal scapular artery is often a direct branch of subclavian artery and participate in scapular
anastomosis. Thyrocervical trunk also participate in scapular anastomosis by giving suprascapular
artery and transverse cervical artery.

Allen Test
16. Allen’s test is done for checking
a) Neural disorders
b) Patency of ulnar artery
c) Patency of radial artery
d) Blood flow in cephalic vein
PG-DIAMS ANATOMY 164

 Allen test is done to check the patency of the radial and ulnar arteries at the wrist and so determines
whether each individual artery is sufficient to maintain the arterial supply to the hand in isolation.
Questions: Upper Limb
17. All is true about clavicle EXCEPT
a) No marrow cavity
b) Long bone in horizontal disposition
c) Two secondary centres of ossification
d) Fractures at the junction of lateral and intermediate third

 Long bones generally have one primary centre of ossification, clavicle bein an exception to have
double primary centre of ossification. Clavicle is a membranous bone (intra-membranous
ossification). Fracture of the clavicle may result from a fall on the shoulder or outstretched hand. The
fracture is most often in the middle third (at the junction of lateral 1/3 and medial 2/3) and results in
upward displacement of the proximal fragment pulled by the sternocleido-mastoid muscle and
downward displacement of the distal fragment by the deltoid muscle and gravity.
18. The accompanying x-ray shows the shoulder of an 11-year-old girl who fell off the monkey bars,
extending her arm in an attempt to break her fall. The small arrows indicate the fracture area.
The large arrows indicate which of the following

a) Fracture at the surgical neck of the humerus


b) Glenohumeral joint
c) Joint space between the proximal humerus and the acromion of the scapula
d) Proximal humeral epiphyseal plate

19. All the pairs about bony attachments around shoulder joint are correctly matched EXCEPT
a) Latissimus dorsi : Floor of intertubercular sulcus
b) Short head of biceps : Tip of coracoid process
c) Subscapularis : Lesser tubercle
d) Teres major : Greater tubercle
PG-DIAMS ANATOMY 165

 Rotator (Musculotendinous) cuff is contributed by the tendons of the supraspinatus, infraspinatus,


teres minor, and subscapularis (SITS); fuses with the joint capsule; and provides mobility. It keeps the
head of the humerus in the glenoid fossa during movements and stabilizes the shoulder joint.
Subscapularis muscle is sometime referred to as forgotten muscle, in the rotator cuff.
 Intertubercular (Bicipital) groove lies between the greater and lesser tubercles and lodges the tendon
of the long head of biceps brachii muscle, It provides insertions for the pectoralis major on its lateral lip,
the teres major on its medial lip, and the latissimus dorsi on its floor (Lady between two majors).
 Three muscles attach to the coracoid process of scapula: coraco-brachialis, short head of biceps
brachii and pectoralis minor.
 Greater and lesser tubercles of humerus are traction epiphysis and extracapsular.
PG-DIAMS ANATOMY 166

 Quadrangular space is bounded superiorly by the teres minor (and subscapularis muscle), inferiorly by
the teres major muscle, medially by the long head of the triceps, and laterally by the surgical neck of the
humerus. It transmits the axillary nerve and the posterior circumflex humeral vessels.
 Upper triangular space is formed superiorly by the teres minor muscle, inferiorly by the teres major
muscle, and laterally by the long head of the triceps. Circumflex scapular vessels course through it.
 Lower triangular space is bounded superiorly by the teres major muscle, medially by the long head of
the triceps and laterally by the shaft of the humerus (and medial head of the triceps). Radial nerve and
the profunda brachii (deep brachial) vessels course through it.
20. The accompanying artery with axillary nerve in the quadrangular space is
a) Anterior circumflex humeral artery
b) Posterior circumflex humeral artery
c) Profunda brachii artery
d) Circumflex scapular artery
21. The cubital fossa is bounded laterally by the muscle
a) Brachioradialis
b) Pronator teres
c) Brachialis
d) Supinator
PG-DIAMS ANATOMY 167

 Cubital fossa is a triangular space on the anterior aspect of the elbow that is bounded by the
brachioradialis muscle laterally, pronator teres muscle medially, and superiorly by an imaginary
horizontal line connecting the two epicondyles of the humerus. At the floor are brachialis and
supinator muscles. The contents (in lateral to medial order) are the radial nerve, biceps tendon,
brachial artery, and median nerve. Ulnar nerve passes behind the medial epicondyle (not a content of
cubital fossa). At its lower end, the brachial artery divides into the radial and ulnar arteries.
Antecubital vein lies at the roof draining cephalic vein into the basilic vein.
22. WRONG about the first metacarpal is
a) Epiphysis is at the head
b) Base is convexo-concave for sellar synovial joint
c) Doesn’t articulate with other metacarpals
d) More anterior and medially rotated

 Aberrant epiphyses are deviations from the norm (not always present). Epiphysis at the head of the
first metacarpal bone is an example.
PG-DIAMS ANATOMY 168

23. A 43 year old sportsperson suddenly notices that he can no longer hit his normal three-point
shot in basketball. He has been suffering some mild neck pain of 6 weeks duration with pain
down the back of his right arm and extending to the dorsal surface of his hand, including his
middle finger. He has diminished triceps tendon reflex on the right side. Which of the following
investigation is ordered, because you are concerned he has herniated which intervertebral
disk
a) Lateral x-ray; C6–C7
b) Cervical MRI; C6–C7
c) Cervical MRI; C8–T1
d) CT; C6–C7
24. Content of anatomical snuff box
a) Radial nerve
b) Radial artery
c) Cephalic vein
d) Abductor pollicis longus

 Anatomic snuffbox is a triangular interval bounded antero-laterally by the abductor pollicis longus
(and extensor pollicis brevis) and postero-medially by the tendon of the extensor pollicis longus. it has
a floor formed by the styloid process of the radius, scaphoid, trapezium and the base of first
metacarpal bone. Radial artery is the content of the fossa, whereas, cephalic vein and cutaneous
branch of radial nerve lies on the roof.
 De Quervain's tenosynovitis: Inflammation of the two tendons forming antero-lateral boundary of
anatomical snuff box. The tendons involved are abductor pollicis longus and extensor pollicis
brevis and Finkelstein test becomes positive.

25. Froment test is to check which muscle


a) Opponens pollicis
b) Flexor pollicis brevis
c) Flexor pollicis longus
d) Adductor pollicis

 Froment sign: Abnormal flexion of the distal phalanx of the thumb when a sheet of paper is held
between the thumb and the radial surface of the index finger; a sign of a lesion of the ulnar nerve.
PG-DIAMS ANATOMY 169

 Froment sign indicates thumb adductor weakness and consists of flexion of the thumb at the
interphalangeal joint when attempting to oppose the thumb against the lateral border of the second
digit.
26. Mammary gland is supplied by (PGIC)
a) Subscapular artery
b) Musculo-phrenic artery
c) Internal mammary artery
d) Superior thoracic artery
e) Superior epigastric artery

 Mammary gland receives blood from the axillary artery branches (lateral thoracic artery,
thoracoacromial artery); the posterior intercostal arteries and the internal thoracic (mammary)
artery branches.
27. The terminal axillary lymph nodes are
a) Apical
b) Central
c) Posterior
d) Anterior
 Lymphatics from mammary gland drain predominantly (75%) into the axillary nodes, more specifically
to the pectoral (anterior) nodes (including drainage of the nipple). 20% lymphatics enter the
parasternal (internal thoracic) nodes, which lie along the internal thoracic artery/vein. Some
lymphatic vessels drains to the apical nodes and may connect to lymphatics draining the opposite
breast and to lymphatics draining the anterior abdominal wall.
 Apical (medial or infraclavicular) nodes lie at the apex of the axilla (medial to the axillary vein) and
above the upper border of the pectoralis minor muscle, receive lymph from all of the other axillary
nodes and drain into the subclavian trunks.
PG-DIAMS ANATOMY 170

 Mammary gland is supported by the suspensory ligaments (of Cooper), strong fibrous attachments,
from the dermis of the skin to the deep layer of the superficial fascia passing through the breast.
 Breast cancer in advanced stages. infiltrates Cooper’s ligaments, produces shortening of the
ligaments, causing depression or dimpling of the overlying skin. Advanced sign of inflammatory breast
cancer, peau d’orange (texture of orange peel) is the edematous swollen and pitted breast skin due
to obstruction of the subcutaneous lymphatics.
28. All of the following structures pierce the clavipectoral fascia EXCEPT
a) Lateral pectoral nerve
b) Lateral thoracic artery
c) Cephalic vein
d) Axillary lymphatics
 Clavipectoral fascia extends between the coracoid process, clavicle, and the thoracic wall and
envelops the subclavius and pectoralis minor muscles. It has a costocoracoid membrane, which lies
between the subclavius and pectoralis minor muscles and is pierced by the cephalic vein, the
thoracoacromial artery, and the lateral pectoral nerve.

29. Weight transmission from upper limb to axial skeleton is done by all EXCEPT (AIIMS)
a) Costo- clavicular ligament
b) Coraco-acromial ligament
c) Coraco-clavicular ligament
d) Inter-clavicular ligament

30. Which of the following movements DOESN’T happen in abduction of shoulder


a) Medial rotation of scapula
b) Elevation of humerus
c) Rotation of Clavicle at the sterno-clavicular joint
d) Rotation at the axis of acromioclavicular joint
 Supraspinatus muscle initiates shoulder abduction (0 - 15°), deltoid is the chief muscle for 15 - 90°
and overhead aduction (90 - 180°) is carried out by serratus anterior and trapezius muscles.
PG-DIAMS ANATOMY 171

Additional Questions
31. Anterior axial line reaches till
a) Shoulder
b) Elbow
c) Wrist
d) Knuckle
Ans. c) Wrist.
Explanation: An axial line is the junction between two dermatomes supplied by discontinuous spinal
nerves. AAL (anterior axial line) starts from sternal angle (2nd rib) and reaches the wrist joint level. PAL
(posterior axial line) begins at shoulder and reaches the elbow joint level.

32. Muscle forming the medial wall of axilla is


a) Subscapularis
b) Teres major
c) Pectoralis minor
d) Serratus anterior
Ans. d) Serratus anterior.
Explanation: Medial wall of the axilla has upper 4 ribs on the thoracic wall and the serratus anterior
muscle.
Anterior wall Pectoralis major & minor and subclavius muscle
Posterior wall Subscapularis, teres major and latissimus dorsi
Medial wall Serratus anterior and ribcage
Lateral wall Inter-tubercular sulcus and coracobrachialis & short head of biceps muscle
Roof Clavicle, scapula and first rib
Base Skin
Contents Axillary artery, vein & lymphatics, brachial plexus (cords and branches), long thoracic
nerve, intercostobrachial nerve
 During sentinel lymph node biopsy the nerves at risk are: intercostobrachial nerve (most common),
long thoracic nerve, thoracodorsal nerve.
 Axillary tail (tail of Spence) is a superolateral extension of the mammary gland and reaches the axilla.
33. Carpal bone(s) is/are
a) Capitate
b) Scaphoid
c) Cuboid
d) Cuneate
e) Navicular
Ans. a) Capitate; b) Scaphoid.
Explanation: There are 8 carpal bones in the upper limb. Proximal row (lateral to medial): Scaphoid,
Lunate, Triquestral, Pissiform; Distal row: Trapezium, Trapezoid, Capitate, Hamate. Mneomonic: She
Looks Too Pretty; Try To Catch Her. Lower limb has 7 tarsal bones:Talus, Calcaneum, Cuboid (lateral),
Navicular (medial), 3 Cuneiforms (medial intermediate and lateral). Cuneate is no bone in hand or foot.

34. Root value of hand muscles is


a) C - 5
b) C - 6
c) C - 7
d) C - 8
e) T - 1
Ans. d) C-8; e) T-1.
Explanation: Root value of hand muscles is C-8; T-1. All lumbrical & interossei muscles carry this root
value,and are paralysed in Klumpke’s palsy (C-8;T-1 lesion).

35. Claw hand is hyperextension at metacarpo-phalangeal joint & flexion at the interphalangeal(s).
Which muscles have become non-functional
a) Lumbricals
b) Lumbricals & palmar interossei
c) Lumbricals & dorsal interossei
d) Lumbricals and all interossei
PG-DIAMS ANATOMY 172

Ans. d) Lumbricals and all interossei.


Explanation: 12 muscles: 4 lumbricals and 8 interossei (4 dorsal & 4 palmar) act together to achieve glass
holding position: MCP (meta-carpo-phalangeal) flexion and IP (Inter-phalangeal) extension. Paralysis of
these 12 muscles results in comparative increased activity of anatagonistic (opposite) muscles, leading to
Claw hand deformity: hyperextension at MCP and flexion at IP joints.

36. Action of dorsal interossei


a) Extension at metacarpo-phalangeal joint
b) Adduction at metacarpo-phalangeal joint
c) Flexion at metacarpo-phalangeal joints
d) Flexion at interphalangeal joints
Ans. c) Flexion at metacarpo-phalangeal joints.
Explanation: Digital abduction is a function of the 4 dorsal interossei (‘DAB’-dorsal abduction) and digital
adduction is a function of 4 palmar interossei (‘PAD’-palmar adduct). The 8 interossei muscles work along
with 4 lumbricals for MCP flexion and IP extension (glass holding position).

37. Nerve supply to dorsal interossei are by


a) Radial
b) Ulnar
c) Median
d) Ulnar & median

Ans. b) Ulnar.
Explanation: All the 8 interossei are supplied by the ulnar nerve.
Nerve involved Cause of injury Clinical features
Ulnar nerve Fracture medial Claw hand deformity; Weakness in wrist flexion;
epicondyle (humerus); hand deviates to radial side on flexion; flexion of
wrist slash injury ring and little finger is weak at distal IP joint; MCP
flexion and IP extension of ring & little finger lost;
loss of finger abduction and adduction; loss of
thumb adduction; sensory loss on palmar and
dorsal aspect of medial 1 & ½ fingers; Froment sign
positive, card test positive

Ulnar claw hand

38. Composite muscles are all EXCEPT


a) Flexor digitorum profundus
b) Flexor pollicis brevis
c) Opponens pollicis
d) Flexor carpi ulnaris
Ans. b) Flexor carpi ulnaris.
PG-DIAMS ANATOMY 173

Explanation: Composite (hybrid) muscles have more than one motor supply. Flexor carpi ulnaris is
supplied by only ulnar nerve. The other muscles mentioned in the question have additional nerve supply
from medial nerve as well.
39. While skiing, a person catches a tree to stop and suffers hyper-abduction injury. The neural
involvement is/are (PGIC)
a) C-8; T-1 nerve root
b) Upper trunk of brachial plexus
c) Lower trunk of brachial plexus
d) Ulnar nerve
e) Median nerve

Ans. a) C-8; T-1 nerve root; c) Lower trunk of brachial plexus; d) Ulnar nerve; e) Median nerve.
Explanation: This a case of Klumpke’s palsy (C-8; T-1 lesion) due to stretching of lower trunk of brachial
plexus. It leads to partial injury of median and ulnar nerve and muscles of the hand like lumbricals and
interossei are paralysed (claw hand deformity)

40. TRUE about the upper trunk of brachial plexus


a) Carries root value C - 5, 6, 7
b) Can be blocked medial to scalenus anterior muscle
c) Long thoracic nerve arises from it
d) Lesion leads to partial injury of radial nerve
Ans. d) Lesion leads to partial injury of radial nerve.
Explanation: Upper trunk of brachial plexus carries C-5, 6 root values. Trunks of brachial plexus pass in
the scalene triangle bounded by scalenus anterior and medius muscle, It lies lateral (and not medial) to
the scalenus anterior muscle, where a block can be carried out. Long thoracic nerve arises directly from
the roots of brachial plexus (C-5, 6, 7). Lesion of upper trunk of brachial plexus (e.g., Erb’s palsy) leads to
partial injury of radial nerve.

41. Injury to the upper trunk of brachial plexus results in (PGIC)


a) Supination of forearm
b) External rotation of arm
c) Inability to initiate abduction
d) Decreased sensation on medial side of hand
e) Paralysis of deltoid muscle
Ans. c) Inability to initiate abduction; e) Paralysis of deltoid muscle.
Explanation: Injury to upper trunk of brachial plexus results in Erb’s palsy and policeman tip hand
deformity: Adduction and medial (internal) rotation at shoulder joint; extension at elbow joint and
pronation at radio-ulnar joint. Initiation of shoulder abduction (supraspinatus paralysed) and raising the
arm to 90° (deltoid paralysed) is not possible. Outer (lateral) surface of the upper limb (C-5, 6
dermatome) has sensory disturbance. Decreased sensation on medial side of hand (C-8 dermatome)
occurs in Klumpke’s palsy.

42. Which muscle may NOT be paralysed in Erb’s palsy


a) Brachioradialis
b) Coraco-brachialis
c) Teres minor
d) Deltoid
Ans. b) Coraco-brachialis.
Explanation: In Erb’s palsy there is C-5, 6 root value injury. Coracobrachialis (C- 5, 6, 7), may not be
completerly paralysed. Brachioradialis (C-5, 6) is found paralysed along with deltoid and teres minor.

43. All of the following features can be observed after fracture of surgical neck of humerus, EXCEPT
a) Loss of rounded contour of shoulder
b) Loss of sensation on skin over the upper part of deltoid
c) Weakness of abduction at shoulder joint
d) Atrophy of deltoid muscle
Ans. b) Loss of sensation on skin over the upper part of deltoid.
Explanation: There is loss of sensation on C- 5 dermatome - the upper lateral aspect of arm (on the
lower half of the deltoid). Fracture surgical neck of humerus may damage axillary nerve leading to
PG-DIAMS ANATOMY 174

paralysis of deltoid (abduction) and teres minor (lateral rotation) problem at shoulder joint. Since deltoid
undergoes atrophy, rounded contour of shoulder is lost.

44. A patient is unable to adduct his thumb. The nerve involved is characterized by (PGIC)
a) Having C-8; T-1 root value
b) Arise from medial cord of brachial plexus
c) Arise from the medial and lateral cord of brachial plexus
d) Musician’s nerve
e) Supply first two lumbricals
Ans. a) Having C-8; T-1 root value; b) Arise from medial cord of brachial plexus; d) Musician’s
nerve.
Explanation: Loss of thumb adduction occurs due to paralysis of adductor pollicis (ulnar nerve lesion).
Ulnar nerve has C-(7), 8; T-1 root value, is the continuation of medial cord of brachial plexus, supplies
intrinsic muscles of the hand like all interossei and medial (last) two lumbricals, hence controls finer
movement of fingers for playing musical instruments (appropriately called musician’s nerve).

45. All is TRUE about median nerve EXCEPT


a) Labourer’s nerve
b) Injury produces total claw hand
c) Damaged in supracondylar fracture of humerus
d) Damaged in wrist slashing
Ans. b) Injury produces total claw hand.
Explanation: Total claw hand is due to injury of both median and ulnar nerve. Median nerve supplies the
anterior forearm and the thenar muscles, which are required for labourer’s job like pulling, pushing, lifting
heavy loads. In median nerve injuries the task of a labourer may get severely compromised. Median nerve
is the most commonly damaged nerve in supracondylar fracture and wrist slash injuries as well.

46. All is true about carpal tunnel syndrome EXCEPT


a) Most common entrapment mono-neuropathy
b) Rheumatoid arthritis is a cause
c) Wasting and weakness of adductor pollicis
d) Numbness in lateral 3 ½ fingers
Ans. c) Wasting and weakness of adductor pollicis.
Explanation: Carpal tunnel syndrome results in median nerve injury and not ulnar nerve (adductor pollicis
is not affected).

47. Pen test in the hand is performed to assess the neuromuscular status of
a) Opponens pollicis
b) Flexor pollicis brevis
c) Abductor pollicis brevis
d) 1st palmar interossei
Ans. c) Abductor pollicis brevis.
Explanation: Pen test is to check anterior abduction of thumb, carried out by abductor pollicis brevis
(median nerve supply).

48. Structure NOT passing deep to flexor retinaculum is


a) Flexor carpi radialis
b) Flexor digitorum superficialis
c) Flexor digitorum profundus
d) Median nerve
Ans. a) Flexor carpi radialis.
Explanation: Tendon of flexor carpi radialis is embedded in the substance of flexor retinaculum and is
neither superficial nor deep to the flexor retinaculum.
49. All are branches of the posterior cord of brachial plexus EXCEPT
a) Axillary nerve
b) Radial nerve
c) Long thoracic nerve
d) Thoracodorsal nerve
Ans. c) Long thioracic nerve.
PG-DIAMS ANATOMY 175

Explanation: Long thoracic nerve arises directly from the roots of brachial plexus (C- 5, 6, 7). Posterior
cord of brachial plexus gives five branches (STARS): S – subscapular (upper), T - Thoracodorsal nerve, A
– Axillary nerve, R – Radial nerve, S – subscapular (lower).

50. All is true about Radial nerve EXCEPT


a) Continuation of lateral cord of brachial plexus
b) Root value is C-5,6,7,8 and T-1
c) Damaged in fracture shaft of humerus
d) Cock up splint is used for its injury in radial groove
Ans. a) Continuation of lateral cord of brachial plexus.
Explanation: Radial nerve is continuation of the posterior cord of brachial plexus. Lateral cord continues
as musculocutaneous nerve. Radial nerve has all the five root values of brachial plexus. Fracture mid-
shaft of humerus damagers the radial nerve in the radial groove leading to wrist drop, the patient is given
cock-up splint to prevent the resuting deformities.

51. All of the following are post-axial veins EXCEPT


a) Cephalic vein
b) Basilic vein
c) Axillary vein
d) Subclavian vein
Ans. a) Cephalic vein.
Explanation: Cephalic vein is a pre-axial vein.

52. Infection draining the ring finger goes to


a) Thenar space
b) Mid palmar space
c) Ulnar bursa
d) Radial bursa
Ans. B) Mid palmar space.
Explanation: Ring finger pus drains towards the midpalmar space.
 Fascial spaces of the palm are deep to the palmar aponeurosis and divided by a midpalmar (oblique
septum attached to third metacarpal) into the thenar space and the midpalmar space. Thenar Space is
the lateral space that contains the flexor pollicis longus tendon and the other flexor tendons of the index
finger. Midpalmar Space is the medial space that contains the flexor tendons of the medial three digits.

 The tendons of the second, third, and fourth digits have separate synovial sheaths so that the infection is
confined to the infected digit, but rupture of the proximal ends of these sheaths allows the infection to
spread to the midpalmar space. The synovial sheath of the little finger is usually continuous with the
common synovial sheath (ulnar bursa), and thus, tenosynovitis may spread to the common sheath and
thus through the palm and carpal tunnel to the forearm. Likewise, tenosynovitis in the thumb may spread
through the synovial sheath of the flexor pollicis longus (radial bursa).
PG-DIAMS ANATOMY 176

 First lumbrical space communicates with thenar space whereas, 2, 3 and 4 lumbrical canals are
continuous with mid-palmar space.
 Infection from thumb and index finger passes towards the thenar space along the first lumbrical canal.
 Middle, ring finger and little finger drain towards mid palmar space along the 2, 3 & 4th lumbrical canals.
 Ulnar bursa is the common synovial flexor sheath which envelops the tendons of both the flexor
digitorum superficialis and profundus muscles. Radial Bursa is the synovial flexor sheath for flexor
pollicis longus.
Thumb infection may lead to inflammation of the radial bursa, whereas, little finger infection involves the
ulnar bursa.
 Fore arm space of Parona lies proximal to the flexor retinaculum and is continuous with the radial &
ulnar bursa.
Flexor retinaculum separates Fore arm space of Parona from the thenar & mid-palmar space and they are
non-continuous.
Note: Bursa is defined as a potential space lined by synovial membrane.

High Yield Facts


 Biceps brachii is a powerful supinator at the radio-ulnar joint and helps in screw driving movements.
 Guyon’s canal syndrome : Entrapment of the ulnar nerve in the Guyon’s canal, causing pain,
numbness, and tingling in the ring and little fingers, and motor weakness later. It can be treated by
surgical decompression of the nerve. Guyon’s canal (ulnar tunnel) is formed by the pisiform, hook of the
hamate, and pisohamate ligament, deep to the palmaris brevis and palmar carpal ligament and transmits
the ulnar nerve and artery.
 Common interosseous artery is a branch of ulnar artery near the elbow joint and divides into anterior
and posterior interosseous artery.
 Cephalic vein begins as a lateral continuation of the dorsal venous arch on hand, runs on the radial
side, and is connected with the basilic vein by the median cubital vein (at the roof of cubital fossa). It
ascends along the lateral surface of the arm and lies in the deltopectoral groove and pierces the
clavipectoral fascia to drain into the axillary vein.
 Median cubital vein connects the cephalic vein to the basilic vein over the cubital fossa. It lies
superficial to the bicipital aponeurosis and is the most commonly used vein in upper limb for
withdrawal of blood samples, intravenous injections, and blood transfusions.
 Impingement syndrome is a type of overuse injury with progressive pathologic changes resulting from
mechanical impingement by the acromion, coracoacromial ligament, coracoid process, or
acromioclavicular joint against the rotator cuff (especially supraspinatus muscle) ; changes may include
reversible edema and hemorrhage, fibrosis, tendinitis, pain, bone spur formation, and tendon rupture.
 Inferior angle of scapula corresponds to T – 7 vertebra (surface marking).
 There is often an enlargement or pseudoganglion on the axillary nerve branch to teres minor. The
termination of posterior interosseous nerve also shows a pseudo-ganglion Deep fibular nerve in lower
limb may also develop a pseudoganglion in a branch to extensor digitorum brevis.
PG-DIAMS ANATOMY 177

Section 8. Abdomen

Embryology
Umbilical Cord Contents
1. Which is NOT associated with vitello-intestinal duct
a) Ileal diverticulum
b) Umbilical sinus
c) Enterocystoma
d) Mesenteric cyst

 Ileal (Meckel) diverticulum is found in about 2% of the population, located within 2 ft of the ileocecal
junction (on the anti-mesenteric side of the ileum), and is usually about 2 inches long. Often contain
two types of ectopic tissue (gastric and pancreatic). Peptic ulceration of adjacent ileal mucosa and
volvulus are complications.
PG-DIAMS ANATOMY 178

2. Identify the clinical condition

a) Omphalocele
b) Gastroschisis
c) Morgagnian hernia
d) Bochdalek hernia

3. Regarding Gastroschisis and omphalocele, which one is FALSE


a) Intestinal obstruction is common in gastroschisis
b) Liver is the content of omphalocele
c) Gastroschisis is associated with multiple anomalies
d) Umbilical cord is attached in normal position in gastroschisis

Diaphram and Mesentery Development


4. All are derivatives of Septum Transversum EXCEPT
a) Falciform ligament
b) Ligamentum teres
c) Coronary ligament
d) Lesser omentum
PG-DIAMS ANATOMY 179

5. The umbilical vein carries


a) Oxygenated blood towards the placenta
b) Deoxygenated blood towards the placenta
c) Oxygenated blood away from the placenta
d) Deoxygenated blood away from the placenta
6. Diaphragm develops from all EXCEPT
a) Septum transversum
b) Dorsal mesocardium
c) Pleuroperitoneal membrane
d) Cervical myotomes
 The diaphragm is a musculotendinous partition to separate the pleural and peritoneal cavities. It is
formed from fusion of the following structures: 1. Septum transversum, 2. Paired pleuroperitoneal
membranes 3. Dorsal meso-oesophagus (mesentery). 4. Cervical somites (Body wall), hence
innervated by the phrenic nerves (C3, C4, and C5).
 Deficiency in the pleuroperitoneal membrane or its failure to fuse with the other parts of the
diaphragm leads to Bochdalek hernia (congenital diaphragmatic hernia), presenting as neonatal
emergency. Abdominal contents are herniate into the left pleural cavity, leading to left lung hypoplasia
and right mediastinal shift (and resulting cyanosis). Mother presnts with polyhydramnios and baby has
scaphoid (flattened) abdomen, cyanosis, and difficulty in breathing. Immediate nasogastric intubation is
performed, and the surgery is postponed by few days till the patient is stabilised.
PG-DIAMS ANATOMY 180

7. Bochdalek hernia occurs in


a) Antero-lateral part of the diaphragm
b) Postero-lateral part of diaphragm
c) Retrosternal area
d) Posterior to diaphragm
8. Structure passing through the marker B

a) Inferior vena cava


b) Oesophagus
c) Aorta
d) Morgagni hernia
PG-DIAMS ANATOMY 181

 There are 3 major openings in the diaphragm:(a) the vena caval hiatus, which lies in the central
tendon at the level of T-8 and transmits the IVC and the right phrenic nerve branches; (b) the
esophageal hiatus, which lies in the muscular part of the diaphragm at the level of T-10 and transmits
the esophagus, vagus nerve and branches of left gastric vessels; and (c) the aortic hiatus, which lies
between the two crura at the level of T-12 and transmits the aorta, thoracic duct, azygos vein, and
sometimes greater splanchnic nerve.
 Right crus of diaphragm is longer than the left. Fibres of right crus surrounds the oesophagus at the
passage into the abdomen (? Sphincter action).

9. Which of the following structure DOESN’T develop in mesentery of stomach


a) Liver
b) Kidney
c) Spleen
d) Pancreas

Gut Rotation
PG-DIAMS ANATOMY 182

10. Physiological hernia reduces at month


a) 1
b) 2
c) 3
d) 4
Physiological umbilical hernia: At week 6, the gut tube connected to the yolk sac herniates into the
region of umbilical cord. The hernia regresses at week 11 and the gut tube returns back to the abdominal
cavity. Non-regression of the hernia reults in Omphalocele.

11. The mesentery of small intestine, along its attachment to the posterior abdominal wall, crosses
all of the following structures EXCEPT
a) Left gonadal vessels
b) Third part of duodenum
c) Aorta
d) Right ureter
 The root of the mesentery lies along a line running diagonally from the duodenojejunal flexure on the
left side of the second lumbar vertebral body to the right sacroiliac joint. The root crosses over the
third part of the duodenum, aorta, inferior vena cava, right ureter and right psoas major.
PG-DIAMS ANATOMY 183
PG-DIAMS ANATOMY 184
PG-DIAMS ANATOMY 185

Abdomen: NeuroVascular Supply

 The splanchnic nerves contain preganglionic sympathetic GVE fibers with cell bodies located in the
lateral horn (intermediolateral cell column) of the spinal cord and GVA fibers with cell bodies located in
the dorsal root ganglia.
PG-DIAMS ANATOMY 186

12. Which of the following is the terminal group of lymph node in colonic drainage
a) Preaortic
b) Intermediate
c) Para colic
d) Epicolic
13. Testicular lymphatics drain into which lymph nodes
a) Superficial inguinal
b) Internal iliac
c) Preaortic
d) Paraaortic
 Gondas (testis and ovary) drain into the para-aortic lymph nodes→ cisterna chyli→thoracic duct
→Left jugulo-subclavian angle.
14. All is true about thoracic duct EXCEPT
a) Begins at level of T12
b) Enters thorax through aortic opening
c) Crosses from right to left at level of T8
d) Passes the superior aperture of thorax
e) Passes in posterior and superior mediastinum
PG-DIAMS ANATOMY 187

15. All of the following pass through aortic hiatus EXCEPT


a) Aorta
b) Greater splanchnic nerve
c) Thoracic duct
d) Azygous vein

 Right lymphatic duct begins as a convergence of the right subclavian lymph trunk, right jugular lymph
trunk, and right bronchomediastinal lymph trunk and terminates at right jugulo-subclavian angle at the
base of the neck. It drains right upper quadrant (RUQ) of the body: right side of the head & neck, right
thoracic region (including medial and lateral quadrant of right breast, right lung) and the right upper
limb.
PG-DIAMS ANATOMY 188
PG-DIAMS ANATOMY 189

16. Kidney derives nerve supply from


a) Coeliac plexus
b) Lumbar plexus
c) Inferior mesenteric nerve
d) Nervi erigentes
PG-DIAMS ANATOMY 190

17. In first stage of labour the referred pain from uterus is carried to the dermatome (AIIMS)
a) T-10, 11
b) T-12; L-1
c) L-1, 2
d) S-2, 3
 Pain during first stage of labour is initially confined to T11 – T12 dermatomes (latent phase), but
eventually labour enters active phase and much of the pain is due to dilatation of cervix and lower
uterine segment and pain passes through hypogastric plexus and aortic plexus before entering the
spinal cord at T10 – L1 nerve roots. Stretching and compression of the pelvic and perineal structures
involves pudendal nerve (S2-4), so pain during second stage of labour involves T10 – S4
dermatomes.
18. To provide pain relief during first stage of labour which sensory level should be blocked
a) T8 to L1
b) T9 to L2
c) T10 to L1
d) T11 to L2

 Spinal anesthesia up to spinal nerve T10 is necessary to block pain for vaginal Delivary and up to
spinal nerve T4 for cesarean section (due to the sympathetic fibre levels being at higher level than
motor or sensory blockade).
Questions: Abdomen- I
19. Hirschsprung’s disease is specifically known as
a) Congenital megacolon
b) Aganglionic megacolon
c) Congenital aganglionic megacolon
d) Congenital atretic aganglionic megacolon

 Hirschsprung disease occurs due to non-migration of neural crest cells into the distal part of the gut
tube colon/rectum. There is absence of myenteric (Auerbach’s) ganglia, which is a parasympathetic
component for faecal evacuation. The diseased segment gets narrowed down and the normal
proximal segment is dilated (maga-colon) due to faecal retention. Rectal biopsy is a-ganglionic. The
presenting complaint is chronic constipation and on per rectal examination, there occurs sudden gush
of the retained faeces.
20. An infant presents with an omphalocele at birth. Which of the following applies to this condition
a) It is also seen in patients with aganglionic megacolon
b) It results from herniation at the site of regression of the right umbilical vein
c) It is caused by a failure of recanalization of the midgut part of the duodenum
d) It is caused by failure of the midgut to return to the abdominal cavity after herniation into the
umbilical stalk
PG-DIAMS ANATOMY 191

21. Which type of gut rotation is shown below

a) Normal
b) Non-rotation
c) Mixed rotation
d) Reverse rotation

22. All of the following contribute to celiac plexus EXCEPT


a) Vagus
b) Lesser splanchnic nerve
c) Ilio-hypogastric nerve
d) Phrenic nerve

23. Pain of distension in congenital hypertrophic pyloric stenosis is carried by


a) Vagus
b) Greater splanchnic nerve
c) Lesser splanchnic nerve
d) Sympathetic root value T-10

24. A 46-year-old woman complains of significant abdominal pain that her physician thinks is
localized to the epigastric region. Which of the following organs is most likely involved in this
problem
a) Duodenum
b) Ileum
c) Kidney
d) Transverse colon
25. A 23-year-old female in good health suddenly doubles over with pain in the area of the
umbilicus. She feels warm and uneasy and has no appetite. The pain seems to have moved to
the lower right abdominal region, which nerves, perceived in the area of the umbilicus, most
likely carried the painful sensations into CNS
a) Vagus nerves
b) Lesser splanchnic nerves
c) 10th Intercostal nerve
d) Greater splanchnic nerves

26. Double inferior vena cava is formed due to


a) Persistence of sacrocardinal vein
b) Persistence of supracardinal vein
c) Persistence of subcardinal vein
d) Persistence of posterior cardinal vein
Ans. b) Persistence of supracardinal vein > c). Persistence of subcardinal vein .
Explanation: Double inferior vena cava results from persistence of left supra-cardinal vein.
Inferior vena cava has contributions from multiple embryonic veins: 1. Right subcardinal veins forms
suprarenal IVC (left subcardinal vein regresses). 2. Right supracardinal vein forms infrarenal IVC (left
supracardinal vein regresses).3. Posterior cardinal vein form distal IVC i.e. iliac bifurcation.
PG-DIAMS ANATOMY 192

Additional Questions: Abdomen- I


27. A child complains of fluid coming out of umbilicus on straining. The diagnosis is
a) Patent vitello-intestinal duct
b) Gastroschisis
c) Umbilical hernia
d) Urachal fistula
Ans. d) Urachal fistula
Explanation: Non-obliteration of allantois (hindgut diverticulum) may result in patent allantoic (urachal)
fistula , which leads to leakage of urine from the urinary bladder towards the umbilicus, especially on
straining.
PG-DIAMS ANATOMY 193

Abdomen - II
Liver Segments
28. Liver is divided into anatomical segments by following all EXCEPT
a) Hepatic vein
b) Portal vein
c) Bile ducts
d) Hepatic artery
PG-DIAMS ANATOMY 194

29. All of the following segment of liver which drains into right hepatic duct EXCEPT
a) I
b) III
c) V
d) VIII

Arteries - Duodenum
30. Superior pancreatico-duodenal artery is a branch of
a) Superior mesenteric artery
b) Gastroduodenal artery
c) Celiac trunk
d) Inferior mesenteric artery
PG-DIAMS ANATOMY 195

 The gastroduodenal artery is prone to erosion by posterior perforation of duodenal ulcer. Splenic
artery may be eroded by penetrating ulcer of the posterior wall of the stomach into the lesser sac.
Left gastric artery may be subjected to erosion by a penetrating ulcer of the lesser curvature of the
stomach.

Peritoneal cavity
31. A 32 year old computer operator with history of heartburn, develops sever excruciating pain in
the epigastric region. She is taken for immediate surgical exploration, which reveals evidence
of ruptured gastric ulcer. Where will the surgeon find the stomach contents
a) Omental bursa
b) Hepatorenal pouch of Morrison
c) Paracolic gutter
d) Pouch of Douglas
PG-DIAMS ANATOMY 196

32. In Pringle’s manoeuvre the structure ligated is


a) Portal pedicle
b) Hepatic vein
c) Inferior vena cava
d) Cystic duct
 In Pringle manoeuvre a large atraumatic haemostat is used to clamp the hepatoduodenal ligament
(free border of the lesser omentum) interrupting the flow of blood through the hepatic artery and the
portal vein and thus helping to control bleeding from the liver. If bleeding though continue, it is likely that
the inferior vena cava or the hepatic vein were also traumatised.
PG-DIAMS ANATOMY 197

33. Following are the boundaries of epiploic foramen EXCEPT


a) Free margin of lesser omentum
b) Inferior vena cava
c) Quadrate lobe of liver
d) Right adrenal
Trans-Pyloric Plane
34. NOT present at the transpyloric plane
a) L-1 vertebra
b) Right suprarenal gland
c) Tip of ninth cartilage
d) Fundus of gallbladder

Anterior Abdominal Wall

35. Gubernaculum is attached to


a) Cranial pole of testis
b) Caudal pole of testis
c) Body of testis
d) Epididymis
PG-DIAMS ANATOMY 198

36. Testes completely descend in the scrotum by the age of


a) End of 7th month of intrauterine life
b) End of 8 month of intrauterine life
c) End of 9 month of intrauterine life
d) After birth

37. Triangle of Doom bounded by all EXCEPT


a) Vas deferens
b) Testicular vessels
c) Cooper ligament
d) Reflected peritoneal fold

38. While performing an operation in the region of femoral canal, the surgeon reaches the femoral
ring. All of the following statements describe the structures forming its boundaries EXCEPT
a) Lacunar ligament
b) Inguinal ligament
c) Pectineal ligament of Cooper
d) Falx inguinalis
PG-DIAMS ANATOMY 199

 The femoral triangle is bounded by the inguinal ligament, the sartorius, and the adductor longus. Its
floor is formed by the iliopsoas, pectineus and adductor longus (& not the sartorius), and the roof is
formed by the fascia lata and cribriform fascia and contains the femoral artery and vein in the femoral
sheath but the femoral nerve outside it.
PG-DIAMS ANATOMY 200

39. Accessory obturator artery is a branch of


a) Inferior epigastric
b) External iliac
c) Internal iliac
d) Obturator
Abdomen - Relations
40. All of the following statements regarding relations of pancreas are true EXCEPT
a) Right renal vein is immediately posterior to the head
b) Superior mesenteric vein lies anterior to the uncinate process
c) First part of duodenum is posterior to the head
d) Superior mesenteric vein lies posterior to the neck

 The left renal vein may be compressed by an aneurysm of the superior mesenteric artery as the
vein crosses anterior to the aorta. Patients with compression of the left renal vein may result in renal
(and adrenal) hypertension on the left. A varicocele may also be found on the left side.

41. All are anterior relations of Right kidney EXCEPT


a) Adrenal gland
b) 4th part of duodenum
c) Liver
d) Hepatic flexure of colon
PG-DIAMS ANATOMY 201

Questions: Abdomen - II
42. Calot’s triangle is bounded by all EXCEPT
a) Inferior surface of liver
b) Common hepatic duct
c) Cystic duct
d) Cystic artery
 The cystic artery commonly arises from the right hepatic artery is the boundary for Calot’s triangle.
The triangle lies between three Cs - Common hepatic duct, Cystic duct, and Cystic artery. Inferior
surface of liver forms the boundary for the triangle of cholecystetctomy. In the angle between
Common hepatic duct and cystic duct lies the Callot’s lymph node of Lund, which gets inflammed in
cholecystitis.

43. Left renal vein crosses aorta


a) Posterior at the level of superior mesenteric artery
b) Anterior above the level of superior mesenteric artery
c) Anterior below the superior mesenteric artery
d) Anterior below the inferior mesenteric artery

44. Which of the following is present in the peritoneal reflection which forms one of the borders of
the paraduodenal fossa
a) Inferior mesenteric vein
b) Middle colic vein
c) Left colic vein
d) Splenic vein

45. Ascent of horse shoe shaped kidney is prevented by


a) Superior mesenteric artery
b) Inferior mesenteric artery
c) Superior mesenteric vein
d) Inferior mesenteric vein

46. Renal angle lies between


a) 12th rib and latissimus dorsi
b) 12th rib and quadratus lumborum
c) 12th rib and lateral border of sacrospinalis
d) Iliac crest and erector spinae

47. Appendices epiploicae are present in


a) Caecum
b) Appendix
c) Sigmoid colon
d) Rectum
PG-DIAMS ANATOMY 202

48. Appendix posses


a) Taenia coli
b) Appendices epiploicae
c) Sacculations
d) Mesentery
49. Spleen projects into the following space of peritoneal cavity
a) Greater sac
b) Left subhepatic space
c) Infracolic compartment
d) Paracolic gutter
50. NOT a content of spermatic cord
a) Ductus deferens
b) Testicular artery
c) Ilio-inguinal nerve
d) Genital branch of genitofemoral nerve

 The spermatic cord is surrounded by the external spermatic fascia, which is derived from the
aponeurosis of the external oblique abdominal muscle, the cremasteric fascia (cremaster muscle and
fascia) originating from the internal oblique abdominal muscle, and the internal spermatic fascia, which
is derived from the transversalis fascia.

Additional Questions
51. Liver is divided into two surgical halves by all EXCEPT
a) Cantlie’s line
b) Right hepatic vein
c) Portal vein at porta hepatis
d) Biliary duct at porta hepatis
Ans. b) Right hepatic vein.
Explanation: Liver is divided into two surgical halves by following middle hepatic vein (and not the right
hepatic vein)
PG-DIAMS ANATOMY 203

52. A Segmental resection was performed removing part of liver lying left of the falciform ligament.
The segments still retained in the left surgical liver are (AIIMS)
a) 2,3
b) 1,4
c) 2,4
d) 1,4,5
Ans. b) 1,4.
Explanation: During hepatic resection, segment 2 and 3 (lying left to the falciform ligament) have been
removed, and segment 1 and 4 are retained in the left surgical liver.
53. Regarding artery supply of pancreas, which of the following is/are correct (PGIC)
a) Both superior and inferior pancreatico-duodenal arteries are branches of gastro-duodenal artery
b) Posterior superior pancreatico-duodenal artery is a branch of superior mesenteric artery
c) Anterior inferior pancreatico-duodenal artery is a branch of superior mesenteric artery
d) Posterior inferior pancreatico-duodenal artery is a branch of gastro-duodenal artery
e) Body and tail are supplied by splenic artery
Ans. c) and e) .
Explanation: Gastroduodenal artery gives superior pancreatico-duodenal arteries (anterior and posterior
both). Superior mesenteric artery gives inferior pancreatico-duodenal arteries (anterior and posterior both).
Splenic artery runs on the superior border of pancreas and give multiple branches to body and tail of
pancreas.
54. Structures damaged while resecting the free edge of lesser omentum (PGIC)
a) Portal vein
b) Hepatic vein
c) Proper hepatic artery
d) Cystic duct
e) Common bile duct
Ans. a) Portal vein; c) Proper hepatic artery; e) Common bile duct.
Explanation: Free edge of lesser omentum contains the structures that enter the posrta hepatis (DAV). D –
Duct (Common bile), A- Artery (proper hepatic), V – Vein (portal).
55. A patient has a penetrating ulcer of the posterior wall of the first part of the duodenum. Which
blood vessel is subject to erosion
a) Common hepatic artery
b) Gastroduodenal artery
c) Proper hepatic artery
d) Anterior superior pancreatico-duodenal artery
Ans. b) Gastroduodenal artery.
Explanation: Gastroduodenal artery passes behind the first part of duodenum and is prone to bleeding in
posterior perforation of duodenal ulcer.
56. Wrong about ileum, as compared with jejunum is
a) Short club shaped villi
b) Long vasa recta
c) More lymphoid nodules
d) More fat in mesentery
Ans. b) Long vasa recta.
Explanation: Ileum has short vasa recta with relatively more arcades.
PG-DIAMS ANATOMY 204

57. Maximum mucosa associated lymphoid tissue is seen in


a) Stomach
b) Duodenum
c) Jejunum
d) Ileum
Ans. d) Ileum
Explanation: The amount of lymphoid tissue increases towards the ileum. Peyer patches are
aggregations of lymphoid tissue (MALT) seen in the terminal ileum.

58. Ovarian ligament and Round ligament of uterus are derivatives of


a) Peritoneum
b) Transversalis fascia
c) Processus vaginalis
d) Gubernaculum
Ans. d) Gubernaculum
Explanation: During development of uterus, gubernaculum becomes ovarian ligament and round ligament
of uterus

59. What is most medial in the femoral triangle


a) Lymphatics
b) Nerve
c) Vein
d) Artery
Ans. a) Lymphatics
Explanation: Deep inguinal lymph nodes (and lymphatics) is the medial most structure in the femoral
triangle.

60. WRONG statement concerning femoral region is


a) Medial boundary of femoral ring is lacunar ligament
b) Femoral hernia lies infero-lateral to the pubic tubercle
c) Femoral nerve lies inside the femoral sheath
d) Adductor longus forms the medial boundary of femoral triangle
Ans. c) Femoral nerve is inside the femoral sheath.
Explanation: Femoral nerve lies outside the femoral sheath.

61. All are boundaries of inguinal triangle EXCEPT


a) Inguinal ligament
b) Medial border of pyramidalis
c) Inferior epigastric vein
d) Lateral border of rectus abdominis
Ans. b) Medial border of pyramidalis.
PG-DIAMS ANATOMY 205

Explanation: Hesselbach’s inguinal triangle is present on the anteroinferior abdominal wall bounded by
the rectus abdominis muscle, the inguinal ligament, and the inferior epigastric vessels.
Medial border: lateral margin of the rectus sheath (linea semilunaris); superolateral border: inferior
epigastric vessels; inferior border: inguinal ligament (Poupart's ligament).
It is the site in which a direct inguinal hernia begins.

62. Posterior wall of rectus sheath below the level of anterior superior iliac spine is formed by
a) Internal oblique
b) Transversus abdominis
c) Lacunar ligament
d) Fascia transversalis
Ans. d) Fascia transversalis.
Explanation: Rectus abdominis lies on the transversalis fascia below thr arcuate line.
 The rectus sheath is the fibrous condensation of the aponeurotic layers on the anterior aspect of the
abdominal wall investing the rectus abdominis muscle. It also encloses the epigastric vessels, the inferior
five intercostal and subcostal vessels and nerves, and occasionally pyramidalis.
 It is incomplete posteriorly at a level inferior to the arcuate line and superiorly above the costal margin.
 The anterior wall is formed from the external oblique aponeurosis and a superficial layer of the internal
oblique aponeurosis where it divides at the lateral edge of the rectus abdominis muscle.
 The posterior wall is formed from the aponeurosis of the transversus abdominis muscle where it joins the
deeper layer of the internal oblique aponeurosis. Together, both walls form the linea alba.
 Since the tendons of the Obliquus internus and Transversus only reach as high as the costal margin, it
follows that above this level the sheath of the Rectus is deficient behind, the muscle resting directly on the
cartilages of the ribs, and being covered merely by the tendon of the Obliquus externus.
 The Rectus, in the situation where its posterior sheath is deficient (below arcuate line), is separated from
the peritoneum only by the transversalis fascia, in contrast to the upper layers, where part of the internal
oblique also runs beneath the rectus. Because of the thinner layers below, this region is more susceptible
to herniation.
PG-DIAMS ANATOMY 206

High Yield Facts


 Umbilical cord contains two umbilical arteries and one umbilical vein. (‘Right’ umbilical vein ‘regresses’
and only ‘left is left’. At the sight of regressing right umbilical vein, there is a weak spot, which might result
in intestinal herniation (Gastroschisis).
 Aorta passes posterior to the diaphragm(and not through it), which is an osseo-aponeurotic opening in
front of T-12 vertebra and behind the median arcuate ligament.
 Oesophagus passes through the muscular opening in the right crus of diaphragm at T-10 vertebra level,
along with the two vagal nerves and branches of left gastric artery(and vein) which supply lower 1/3 of
oesophagus.
 Bochdalek hernia occurs through left postero-lateral opening in the diaphragm.
 The lienogastric (spleno-gastric) ligament contains the short gastric and left gastroepiploic vessels; the
lienorenal (splenorenal) ligament contains the splenic vessels and the tail of the pancreas; and the free
margin of the falciform ligament contains the ligamentum teres hepatis (fibrous remnant of the left
umbilical vein, and the paraumbilical vein.
 The renal fascia (the false capsule or Gerota fascia) is a discrete fascial layer that surrounds each
kidney. Paranephric fat outside this capsule and perinephric fat inside this fascial layer support the
kidney.
 Portal system is a system of vessels in which blood collected from one capillary network passes through
a large vessel and then a second capillary network before it returns to the systemic circulation. In hepatic
portal system blood from the intestinal capillary bed passes through the hepatic portal vein and then
hepatic capillaries (sinusoids) to the hepatic veins.
 Porto-systemic anastomosis

 Para-umbilical veins connects the left branch of the portal vein with the subcutaneous veins in the region
of the umbilicus.
 The median umbilical fold or ligament contains the fibrous remnant of the obliterated urachus, the medial
umbilical fold contains the fibrous remnant of the obliterated umbilical artery, and the lateral umbilical fold
contains the inferior epigastric vessels.
 A level of sensory blockade extending to the T 10 is desired in vaginal delivery (and T4 dermatome in
caesarean delivery). –William’s Obstetrics.
 The adrenal gland receives arteries from three sources: the superior suprarenal artery from the inferior
phrenic artery, the middle suprarenal from the abdominal aorta, and the inferior suprarenal artery from the
renal artery. It is drained via the suprarenal vein, which empties into the IVC on the right and the renal
vein on the left.
 The suprarenal and gonadal veins drain into the IVC on the right and the left renal vein. The azygos vein
is connected to the IVC, but the hemiazygos vein is connected to the left renal vein.
 An obstruction in the flow through the portal system (valveless) may cause reversal of blood flow and
portal hypertension. Blood flows in a retrograde direction and pass through porto-systemic
anastomosis to reach the caval system. Sites for these anastomoses include the esophageal veins and
rectal veins leading to varices and thoracoepigastric veins leading to caput medusae.
 Hepatic lobules are the small vascular units composing the substance of the liver, each of which is
polygonal, with a central vein at its center and portal canals peripherally at the corners. Portal lobule is a
triangular mass of liver tissue, larger than a liver acinus, containing portions of three adjacent hepatic
lobules, and having a portal vein at its center and a central vein peripherally at each corner.
PG-DIAMS ANATOMY 207

 Liver acinus is a functional unit of the liver, smaller than a portal lobule, being a diamond-shaped mass
of liver parenchyma surrounding a portal tract. It consists of adjacent sectors of neighboring hexagonal
fields of classic lobules partially separated by distributing blood vessels. The zones, marked 1, 2, and 3,
are supplied with blood that is most oxygenated and richest in nutrients in zone 1 and least so in zone 3.
The terminal hepatic venules (central veins) in this interpretation are at the edges of the acinus instead of
in the center, as in the classic lobule. The vessels of the portal canals, namely, terminal branches of the
portal vein and hepatic artery that, along with the smallest bile ducts, make up the portal triad, are shown
at the corners of the hexagon that outlines the cross-sectioned profile of the classic lobule.
PG-DIAMS ANATOMY 208

Section 9. Pelvis and Perineum

Embryology
1. WRONG about genital system development is
a) Develop from mesoderm
b) Genital ridge forms at week 5
c) Testes develops earlier to ovary
d) External genitalia are fully differentiated at week 10

 Genotype of the embryo is established at fertilization, but male and female embryos are
phenotypically indistinguishable till weeks 6. Testis starts developing at week 7, whereas ovarian
development begins at week 10. Male and female characteristics of the external genitalia can be
recognized by week 12. Phenotypic differentiation is completed by week 20.
 Before the seventh week of gestation, the fetal gonads are not differentiated into either the male or
female genotype. Primordial germ cells migrate into the genital ridge (to form spermatocytes or
oocytes). The presence or absence of the Y chromosome (SRY gene - sex-determining region of the Y
chromosome) determine gonadal differentiation. All humans are destined to become females (default
mechanism) until interrupted by Y chromosome.
2. Trigone of urinary bladder develops from
a) Mesoderm
b) Ectoderm
c) Endoderm of urachus
d) Endoderm of urogenital sinus

 Genitourinary system develops from Intermediate mesoderm. It forms the Urogenital ridges on each
side of the aorta. Three pairs of kidneys develop in cranio-caudal sequence in the urogenital ridge of
intermediate mesoderm: pronephros, mesonephros, and metanephros.
 Pronephros regresses by the fifth week. Mesonephric duct (Mesonephros) at caudal end gives the
Ureteric bud (that later forms the ureter, renal pelvis, calyces, and collecting tubules).
 Ureteric bud penetrate and induces Metanephros to develop into the adult kidney. Kidney is formed
during the fifth week from the metanephric mass (develops into nephrons for urine formation) and the
ureteric bud (collecting system).
 Kidney develops in the pelvic cavity and ascends from sacral levels to lower thoracic levels later.
 Mesonephric duct also give Wolffian duct (which develops into the efferent ductules, epididymal duct,
ductus deferens, ejaculatory duct and seminal vesicles). In females it gives vestigeal remnants:
epoophoron, paroophoron and Gartner’s duct.
 Urogenital sinus forms the urinary bladder, urethra (and urethral and paraurethral glands, greater
vestibular glands) and lower vagina in females and urinary bladder, urethra (and prostate &
bulbourethral glands in males).
PG-DIAMS ANATOMY 209

 Allantois is continuous with urinary bladder, later gets obliterated to form urachus (median umbilical
ligament).
 Paramesonephric (Müllerian) ducts develop on the sides of Mesonephric duct and form uterine tubes
and the uterus, cervix, and upper vagina in females and form the prostatic utricle (and appendix of
testes) in males.

3. All are derivatives of ureteric bud EXCEPT


a) Ureter
b) Renal pelvis
c) Minor calyx
d) Connecting tubule
PG-DIAMS ANATOMY 210
PG-DIAMS ANATOMY 211
PG-DIAMS ANATOMY 212

Prostate Gland
4. Urethral crest is due to
a) Opening of prostatic glands
b) Puboprostatic spread
c) Insertion of detrusor
d) Insertion of trigone
PG-DIAMS ANATOMY 213

5. Which lobe of prostate gland raises uvula vesicae


a) Anterior lobe
b) Posterior lobe
c) Median lobe
d) Lateral lobe
PG-DIAMS ANATOMY 214

There are four distinct glandular regions in prostate:


Fraction of
Name Description
gland
The sub-capsular portion of the posterior aspect of the prostate gland
Peripheral zone Up to 70% in
that surrounds the distal urethra. It is from this portion of the gland
(PZ) young men
that ~70–80% of prostatic cancers originate.
This zone surrounds the ejaculatory ducts. The central zone accounts
Central zone Approximately
for roughly 2.5% of prostate cancers although these cancers tend to
(CZ) 25% normally
be more aggressive and more likely to invade the seminal vesicles.
~10–20% of prostate cancers originate in this zone. The transition
Transition zone zone surrounds the proximal urethra and is the region of the prostate
5% at puberty
(TZ) gland that grows throughout life and is responsible for the disease
of benign prostatic enlargement.
Anterior fibro-
Approximately This zone is usually devoid of glandular components, and composed
muscular zone
5% only, as its name suggests, of muscle and fibrous tissue.
(or stroma)
PG-DIAMS ANATOMY 215

6. Which of the following zones is prone to benign prostatic hypertrophy

a) Anterior muscular zone


b) Transitional zone
c) Peripheral zone
d) Central zone
Perineal Pouches
7. Contents of deep perineal pouch are all EXCEPT
a) Dorsal nerve of penis
b) Urethral sphincter
c) Root of penis
d) Bulbourethral gland
PG-DIAMS ANATOMY 216

 Urogenital Diaphragm: It is a poorly defined structure, consists of the deep transverse perineal
muscle and the sphincter urethrae and is invested by superior and inferior fasciae. It stretches
between the two pubic rami and ischial rami but does not reach the pubic symphysis anteriorly. It is
pierced by the membranous urethra in the male and by the urethra and the vagina in the female.
Recently the existence of urogenital diaphragm has been challenged, suggesting it is not actually an
identifiable entity.
 The deep perineal space (pouch) lies between the superior and inferior fasciae of the urogenital
diaphragm. It contains the deep transverse perineal muscle and sphincter urethrae, the
membranous part of the urethra, the bulbourethral gland of Cowper in the male, and branches of the
internal pudendal vessels and pudendal nerve.
 The male external urethral sphincter is formed by two muscles: sphincter urethra and compressor
urethrae muscles, both in the deep perineal space. The female external urethral sphincter is formed by
three muscles: sphincter urethra, compressor urethrae, and urethrovaginalis muscles.
 Bulbourethral (Cowper’s) glands lie among the fibers of the deep transverse perineal muscle in the
deep perineal pouch in the male, on the posterolateral sides of the membranous urethra. Ducts pass
through the inferior fascia of the urogenital diaphragm (perineal membrane) to open into the bulbous
portion of the spongy urethra.
 The superficial perineal space (pouch) lies between the inferior fascia of the urogenital diaphragm
(perineal membrane) and the superficial perineal fascia (Colles’s fascia) and contains perineal
muscles, the crus of the penis or clitoris, the bulb of the penis or vestibule, the greater vestibular
glands of Bartholin in the female, branches of the internal pudendal vessels, and the pudendal nerve.
8. All is true about Bartholin gland EXCEPT
a) Homologous of male bulbo-urethral gland
b) Present in the superficial perineal pouch
c) Located at the junction of anterior 1/3 and middle 1/3 of labia majora
d) Opens into the vestibule between hymen and labia minora
PG-DIAMS ANATOMY 217

 Bartholin duct opens in the postero-lateral wall of vagina (vestibule). In questions Lateral wall
> posterior wall.
 The epithelium of the Bartholin duct is cuboidal near the gland, but becomes transitional and
finally stratified squamous near the opening of the duct.
Urethra rupture and Extravasation of Urine
9. A patient exposed to bomb explosion injury presents with rupture of the fundus of urinary
bladder. The extravasated urine reaches
a) Space of Retzius
b) Deep perineal pouch
c) Superficial perineal pouch
d) Peritoneal cavity
 Rupture of the dome (superior wall) of the urinary bladder, leads to rupture of peritoneum and results in
an intraperitoneal extravasation of urine within the peritoneal cavity (ascites). It is caused by a
compressive force on a full bladder.

10. Injury to the male urethra above the perineal membrane due to a pelvic fracture, causes urine
to accumulate in all of the following EXCEPT
a) Space of Retzius
b) Deep perineal pouch
c) Superficial perineal pouch
d) Peritoneal cavity
PG-DIAMS ANATOMY 218

 Rupture of membranous part of the urethra may lead to urine escaping into the space around the
prostate and bladder and extraperitoneal space. If the urogenital diaphragm is also disrupted urine
leaks into deep perineal space and into the superficial perineal space (as the perineal membrane
is also ruptured).
 The most common type of urethral injury is at the junction of posterior and anterior (bulbous) urethra.
Radiologists consider a type III urethral injury as a combined anterior/posterior urethral injury.

11. A 16-year-old boy presents to the emergency department with straddle injury and rupture of the
bulbous urethra. Extravasated urine from this injury can spread into which of the following
structures
a) Scrotum
b) Ischiorectal fossa
c) Deep perineal space
d) Thigh
 Extravasation of urine may result from rupture of the bulbous spongy urethra below the perineal
membrane; the urine may pass into the superficial perineal pouch and spread inferiorly into the
scrotum, anteriorly around the penis, and superiorly into the lower part of the abdominal wall. The
urine cannot spread laterally into the thigh because the perineal membrane and the superficial fascia of
the perineum are firmly attached to the ischiopubic rami and are connected with the deep fascia of the
thigh (fascia lata). It cannot spread posteriorly into the anal region (ischiorectal fossa) because the
perineal membrane and Colles’s fascia are continuous with each other around the superficial
transverse perineal muscles.
PG-DIAMS ANATOMY 219

12. After fracture of the penis (injury to the tunica albuginea) with intact Buck’s fascia, there occurs
hematoma at
a) The penis and scrotum
b) At the perineum in a butterfly shape
c) Penis, scrotum, perineum and lower part of anterior abdominal wall
d) Shaft of the penis only
 If the Buck fascia is intact, penile ecchymosis is confined to the penile shaft. If the Buck fascia has
been violated, the swelling and ecchymosis are contained within the Colles’ fascia. In this instance, a
‘butterfly-pattern’ ecchymosis may be observed over the perineum, scrotum, and lower abdominal
wall.

Pelvis & Perineum: Nerves


13. FALSE statement regarding pudendal nerve is
a) Both sensory and motor
b) Derived from S2,3,4 spinal nerve roots
c) Leaves pelvis through the lesser sciatic foramen
d) Only somatic nerve to innervate the pelvic organs

 The pudendal nerve (S2–S4) passes through the greater sciatic foramen (below the piriformis
muscle) and enters the gluteal region. Then passes through the lesser sciatic along with the internal
pudendal vessels to enter the pudendal canal, gives rise to the inferior rectal and perineal nerves, and
terminates as the dorsal nerve of the penis (or clitoris).
PG-DIAMS ANATOMY 220
PG-DIAMS ANATOMY 221

Pelvis & Perineum: Lymphatics

Questions: Pelvis & perineum


14. While a patient was being given an obstetric nerve block she gradually lost her voluntary control
over urination. The muscle which got anaesthetized is located in the
a) Trigone of the bladder
b) Urogenital diaphragm
c) Superficial perineal pouch
d) Pelvic diaphragm
PG-DIAMS ANATOMY 222

15. All insert into perineal body EXCEPT


a) Superficial transverse perenei
b) Deep transverse perenei
c) Bulbo spongiosus
d) Ischio cavernosus

16. Levator ani muscle include all EXCEPT


a) Puborectalis
b) Pubococcygeus
c) Iliococcygeus
d) Ischiococcygeus

 Pelvic diaphragm: Forms the pelvic floor and supports all of the pelvic viscera. It is formed by the
levator ani (pubococcygeus and ilio-coccygeus) and coccygeus (ischio-coccygeus) muscles and
their fascial coverings. It lies posterior and deep to the urogenital diaphragm and medial and deep to
the ischiorectal fossa.
PG-DIAMS ANATOMY 223

17. All form anorectal ring EXCEPT


a) External sphincter
b) Internal sphincter
c) Puborectalis
d) Anococcygeal raphe
Ans. d) Ano-coccygeal raphe.
Explanation: Anorectal ring is a muscular present at the junction of rectum and anus.It is formed by
fusion of fibres of puborectalis, uppermost fibres of external anal sphincter and internal anal sphincter.
* Applied anatomy - Damage to the anorectal ring results in rectal incontinence.

18. Normal ano-rectal angle is


a) 70°
b) 90°
c) 120°
d) 150°

19. Uterine tube is supplied by


a) T: 6-8
b) T: 8-10
c) T: 10-12
d) L: 1-2
PG-DIAMS ANATOMY 224

20. Urinary bladder is supplied by sympathetic root value


a) L1 & L2
b) L2 & L3
c) L3 & L4
d) L2,L3 & L4

21. Parasympathetic nerve innervation of urinary bladder has


a) Inhibitory control on detrusor contraction
b) Excitatory effect on detrusor
c) No effect on detrusor
d) Excitatory control over internal urethral sphincter
22. The narrowest lumen in ureter is at
a) 1
b) 3
c) 4
d) 5
PG-DIAMS ANATOMY 225

23. All are branches of anterior division of internal iliac artery EXCEPT
a) Ovarian
b) Vesical
c) Middle rectal
d) Pudendal

24. All are branches of posterior division of internal iliac artery EXCEPT
a) Superior vesical
b) Superior gluteal
c) Lateral sacral
d) Ilio-lumbar
25. All the following pairs are correct concerning the lymphatics of uterus EXCEPT
a) Fundus: Para-aortic
b) Mid-uterus: External iliac
c) Cervix: Superficial inguinal lymph nodes
d) Cervix: Sacral
PG-DIAMS ANATOMY 226

1. Fundus and upper part of the body: Pre- and para-aortic lymph nodes along the ovarian vessels (few
lymphatics from the lateral angles of the uterus travel along the round ligaments of the uterus and drain into
superficial inguinal lymph nodes .
2. Middle part of the body : External iliac nodes via broad ligament.
3. From cervix, on each side the lymph vessels drain in three directions:
Laterally: External iliac and obturator nodes.
Posterolaterally: Internal iliac nodes
Posteriorly: Sacral nodes

Additional Questions
26. Clitoris in females is embryologically derived from
a) Urogenital sinus
b) Genital swelling
c) Genital tubercle
d) Urogenital membrane
Ans. c) Genital tubercle.
Explanation: Glans penis and clitoris develop from the genital tubercle (phallus).

27. The transitional epithelium lining the urethra and the bladder is derived from
a) Mesoderm
b) Endoderm
c) Wall of the yolk sac
d) Paramesonephric duct
Ans. b) Endoderm.
Explanation: Epithelium of urinary bladder, urethra and vagina develop from endoderm of urogenital sinus.

28. Mullerian duct anomaly may include the absence of any of the following EXCEPT
a) Uterus
b) Vagina
c) Ovary
d) Uterine tube
Ans. c) Ovary.
Explanation: Ovaries develop from genital ridge.
PG-DIAMS ANATOMY 227

29. Organ of Rosenmüller (epoophoron) is derivative of


a) Mullerian duct
b) Wolffian duct
c) Urogenital sinus
d) Paramesonephric duct
Ans. b) Wolffian duct.
Explanation: Epoophoron is a vestigeal remnant of Wolffian duct in females.
30. UNTRUE about prostate is
a) Behind the urethra and between the two ejaculatory ducts lie the median lobe
b) Colliculus seminalis is an elevation on urethral crest showing three openings
c) Its urethra is convex anteriorly
d) Its urethra appears crescentic in transverse section
Ans. c) Its urethra is convex anteriorly.
Explanation: Male urethra is concave anteriorly.
31. Urogenital diaphragm is contributed by all EXCEPT
a) Sphincter urethra
b) Perineal body
c) Colles’ fascia
d) Perineal membrane
Ans. c) Colles’ fascia.
Explanation: Urogenital diaphragm has a floor of perineal membrane (not Colle’s fascia).
32. The deep perineal space
a) Is formed superiorly by the perineal membrane
b) Contains a segment of the dorsal nerve of the penis
c) Is formed inferiorly by Colles' fascia
d) Contains the greater vestibular glands
Ans. b) Contains a segment of the dorsal nerve of penis.
Explanation. Dorsal nerve of penis a content of both superficial and deep perinela poch. Other choices are
applicable to superficial perineal pouch.
33. All are content of sphincter of vagina EXCEPT
a) Pubovaginalis
b) External urethral sphincter
c) Internal urethral sphincter
d) Bulbospongiosus
Ans. c) Internal urethral sphincter.
Explanation: Internal urethral sphincter is present in males to prevent retrograde ejaculation of semen into
urinary bladder, it is absent in females.
34. Not a part of superficial perineal pouch
a) Posterior scrotal nerves
b) Sphincter urethrae
c) Ducts of bulbourethral glands
d) Bulbospongiosus muscle
Ans. b) Sphincter urethrae.
Explanation: Sphincter urethrae (part of external urethral sphincter) is present in the wall of membranous
urethra, in the deep perineal pouch. It also extends vertically, around the anterior aspect of the prostatic
urethra.
*Posterior scrotal nerves are the branches of pudendal nerve, and do pass the superficial perineal pouch.
*Cowper’s (bulbourethral) gland is present in the deep perineal pouch, but its duct pierces the perineal
membrane and opens into the bulbous urethra in the superficial perineal pouch.
*Bulbospongiosus muscle is a content of superficial perineal pouch, working as a vaginal sphincter in a
female; and for a male it help to empty the urethra of the urine/semen.
*The superficial perineal pouch is a fully enclosed compartment. Its inferior border is the perineal fascia
(Colles’ fascia). Its superior border is the perineal membrane.
- Contents:
1. Muscles
- Ischiocavernosus muscle
- Bulbospongiosus muscle
PG-DIAMS ANATOMY 228

- Superficial transverse perineal muscle


2. Others
- Crura of penis (males) / Crura of clitoris (females)
- Bulb of penis (males) / Vestibular bulbs (females)
- Greater vestibular glands (female)

35. Root value of inferior rectal nerve supplying external anal sphincter is
a) L– 3, 4, 5
b) L– 5; S-1
c) S– 2, 3 4
d) S– 4, 5
Ans. c) S – 2, 3, 4.
Explanation: External anal sphincter is upplied by inferior rectal nerve branch of pudendal nerve.
36. All of the following pairs about the boundaries of ischiorectal fossa are correct EXCEPT
a) Anterior: Perineal membrane
b) Posterior: Gluteus maximus
c) Medial: Levator ani
d) Lateral: Obturator externus
Ans. d) Obturator externus.
Explanation: Obturator internus is present at the lateral wall of ischiorectal fossa. It is covered by
obturator fascia, which has pudendal canal in it.

37. UNTRUE about ischiorectal fossa


a) Obturator fascia meets anal fascia at the apex
b) A communication is present between the two IRF in front of anal canal
c) Alcock's canal is located at the lateral wall
d) Inferior rectal nerve and vessels pass through it
PG-DIAMS ANATOMY 229

Ans. b) A communication is present between the two IRF in front of anal canal
Explanation: A communication is present between the two IRF in behind the anal canal. The ischiorectal
fossa is separated from the pelvis by the levator ani and its fasciae and is bounded by the sphincter
urethrae and deep transverse perineal muscles (anteriorly), the gluteus maximus and the sacrotuberous
ligament (posteriorly), the sphincter ani externus and levator ani (superomedially), the obturator fascia
covering the obturator internus (laterally), and the skin (floor). Alcock’s pudendal canal is present in the
lateral wall of ischiorectal fossa and send inferior rectal nerve and vessels medially through the fossa
towards the anal canal.

38. UNTRUE about female pelvis is


a) Subpubic angle is wide (>80 degrees)
b) Obturator foramen in triangular
c) Greater sciatic notch is wide (~90 degrees)
d) Long and narrow sacrum
Ans. d) Long and narrow sacrum.
Explanation: Female pelvis has short and wide sacrum.
PG-DIAMS ANATOMY 230

39. Almost half of the females have which of the following type of pelvis
a) Anthropoid
b) Android
c) Platypelloid
d) Gynaecoid
Ans. d) Gynaecoid.
Explanation: The gynaecoid pelvis is the normal female type; its pelvic inlet typically has a rounded oval
shape and a wide transverse diameter. A platypelloid or markedly android (masculine or funnel-shaped)
pelvis in a woman may present with difficult vaginal delivery of a fetus.

 The obstetric conjugate is the least anteroposterior diameter of the pelvic inlet from the sacral
promontory to a point a few millimeters below the superior margin of the pubic symphysis.
40. Artery supply to ureter is by all EXCEPT
a) Gonadal artery
b) Common iliac artery
c) External iliac artery
d) Vesical artery
Ans. c). External iliac artery.
Explanation: Ureter has numerous arteries supplying as shown in the diagram (but not external iliac).

41. Lymphatic drainage of distal spongy urethra is towards the lymph nodes
a) Superficial inguinal
b) Deep inguinal
c) External iliac
d) Internal iliac
Ans. b) Deep inguinal.
Explanation: Distal spongy urethra and the glans penis drain into the deep inguinal lymph nodes of
Cloquest and Rosenmuller.
PG-DIAMS ANATOMY 231

42. Lymphatic drainage of uterine cervix is all EXCEPT


a) Obturator
b) Sacral
c) External iliac
d) Internal iliac
Ans. a) Obturator.
Explanation: Uterus drains into all the lymphatic destinations mentioned in the choices, hence this appears
to be a wrong question, though some standard textbooks do not mention obturator lymph nodes in the
lymphatic drainage.

43. Pelvic fascia between rectum and sacrum is


a) Denonvillier’s fascia
b) Colle’s fascia
c) Waldeyer’s fascia
d) Scarpa’s fascia
Ans. c) Waldeyer’s fascia.
Explanation: Waldeyer's fascia (presacral fascia) lines the anterior aspect of the sacrum, enclosing the
sacral vessels and nerves. It is limited postero-inferiorly, as it fuses with the mesorectal fascia, lying above
the levator ani muscle. Identification and preservation of the Waldeyer’s fascia is of fundamental
importance in preventing complications and reducing local recurrences of rectal cancer.
*Denonvillier’s fascia: A membranous partition separating the rectum from the prostate and urinary
bladder; this structure in the male corresponds to the fascia rectovaginalis in the female.

High Yield Facts


 Anal canal

 Rectum: Has a mucous membrane and a circular muscle layer that forms three permanent transverse
folds (Houston’s valves). Per rectal examination is performed for palpating for prostate, seminal vesicle,
ampulla of the ductus deferens, bladder, uterus, cervix, ovaries, perineal body etc.
 Cremasteric artery (external spermatic artery) is a branch of the Inferior epigastric artery which
accompanies the spermatic cord, and supplies the cremaster and other coverings of the cord,
anastomosing with the testicular artery. In the females the cremasteric artery accompanies the round
ligament and is very small.
 Corpora amylacea, are small hyaline masses (detected microscopically) found in the prostate gland,
neuroglia, and pulmonary alveoli. They are derived from degenerate cells or thickened secretions and
occur more frequently with advancing age.
 Parts of fallopian tube (medial to lateral): Interstitial part→ Isthmus→ Ampulla→ Infundibulum.
 Superior Hypogastric Plexus is the continuation of the aortic plexus below the aortic bifurcation and
receives the lower two lumbar splanchnic nerves. It bifurcates into the right and left hypogastric nerves in
front of the sacrum. It contains preganglionic and postganglionic sympathetic fibers, visceral afferent
fibers, and few, if any, parasympathetic fibers, which may run a recurrent course through the inferior
hypogastric plexus.
 Hypogastric Nerve is the lateral extension of the superior hypogastric plexus and lies in the extra-
peritoneal connective tissue lateral to the rectum. It provides branches to the sigmoid colon and the
descending colon and is joined by the pelvic splanchnic nerves to form the inferior hypogastric or pelvic
plexus.
 Inferior Hypogastric (Pelvic) Plexus is formed by the union of hypogastric, pelvic splanchnic, and sacral
splanchnic nerves and lies against the posterolateral pelvic wall, lateral to the rectum, vagina, and base of
PG-DIAMS ANATOMY 232

the bladder. it contains pelvic ganglia, in which both sympathetic and parasympathetic preganglionic
fibers synapse. It gives rise to rectal plexus, utero-vaginal plexus, vesical plexus, and prostatic plexus.
 Sacral Splanchnic Nerves consist of preganglionic sympathetic fibers that come off the sympathetic
chain and synapse in the inferior hypogastric (pelvic) plexus.
 Pelvic Splanchnic Nerves (Nervi Erigentes) arise from the sacral segment of the spinal cord (S2–S4)
and are the only splanchnic nerves that carry parasympathetic fibers. (All other splanchnic nerves are
sympathetic.) They contribute to the formation of the pelvic (or inferior hypogastric) plexus, and supply the
descending colon, sigmoid colon, and other viscera in the pelvis and perineum.
 Pelvic fascia condensations as the supports of uterus: (1) Lateral or transverse cervical (Cardinal or
Mackenrodt’s) ligaments of the uterus extending from the cervix and the vagina to the lateral pelvic walls,
running laterally below the base of the broad ligament. (2) Pubocervical ligaments are bands of connective
tissue that extend from the posterior surface of the pubis to the cervix of the uterus. (3) Sacrocervical
ligaments extend from the lower end of the sacrum to the cervix and the upper end of the vagina. (4)
Rectouterine (Sacrouterine) Ligaments hold the cervix back and upward and sometimes elevate a shelf-
like fold of peritoneum (rectouterine fold), which passes from the isthmus of the uterus to the posterior wall
of the pelvis lateral to the rectum. It corresponds to the sacrogenital (rectoprostatic) fold in the male.
PG-DIAMS ANATOMY 233

Section 10. Lower Limb

Embryology
1. Root value of sciatic nerve is (AIPG)
a) L-1,2,3,4,5
b) L-2,3,4,5;S-1
c) L-3,4,5;S-1,2
d) L-4,5;S-1,2,3

Dermatomes

 Lower limbs rotate medially by 90 degrees, the great toe becomes medial and little toe lateral. The
extensor compartment comes anterior and the flexor compartment becomes posterior.
 The dorsal and ventral axial lines both reach the ankle joint (ventral reaches the medial aspect).
PG-DIAMS ANATOMY 234

Thigh Muscles
PG-DIAMS ANATOMY 235

\
PG-DIAMS ANATOMY 236

Movements – Hip and Knee Joint


2. Rectus femoris is a part of quadriceps femoris causes
a) Hip flexion and knee extension
b) Hip and knee flexion
c) Hip and knee extension
d) Hip extension and knee flexion
PG-DIAMS ANATOMY 237

3. Biceps femoris, a hamstring muscle causes


a) Hip flexion and knee extension
b) Hip and knee flexion
c) Hip and knee extension
d) Hip extension and knee flexion
PG-DIAMS ANATOMY 238

 Superior gluteal nerve passes through the greater sciatic foramen (above the piriformis muscle) to
supply three muscles: gluteus medius, gluteus minimus and tensor fascia lata.
 Gluteus maximus is supplied by inferior gluteal nerve which passes through greater sciatic
foramen, along with sciatic and pudendal nerve (all pass below piriformis muscle).
PG-DIAMS ANATOMY 239

4. In walking, gravity tends to tilt pelvis and trunk to the unsupported side, major factor in
preventing this unwanted movement is
a) Adductor muscles
b) Quadriceps
c) Gluteus maximus
d) Gluteus medius and minimus

Hybrid Muscles
5. Hybrid muscles are all EXCEPT
a) Pectineus
b) Adductor magnus
c) Tensor fascia lata
d) Biceps femoris
PG-DIAMS ANATOMY 240

Knee Joint

6. Physiological locking involves


a) Internal rotation of femur over stabilized tibia
b) Internal rotation of tibia over stabilized femur
c) External rotation of tibia over stabilized femur
d) External rotation of femur over stabilized tibia
7. A healthy young athlete sitting at table with knee at 90 degree flexion. What will happen when he
fully extends the knee
a) Movement of tibial tuberosity towards lateral border of patella
b) Movement of tibial tuberosity towards medial border of patella
c) Movement of tibial tuberosity towards centre of patella
d) No change in relationship

 Terrible triad (MOI): Foot fixed, knee flexed, twisting fall. Lachman test is carried out at 20-30° of knee
flexion and is less painful. It has high sensitivity and specificity as compared with the original
anterior drawer test.
 ACL and PCL are intracapsular but extrasynovial ligaments (lie inside the knee joint capsule but outside
the synovial cavity of the joint), still covered by synovial membrane.
 ACL: Arises from the anterior intercondylar area of the tibia and passes backward, upward, and
laterally (BUL) to insert into the medial surface of the lateral femoral condyle.The anterior cruciate
ligament prevents forward sliding of the tibia on the femur (or posterior displacement of the femur on the
tibia) and prevents hyperextension of the knee joint. It is taut during extension of the knee and is lax
during flexion. It may be injured in hyperextension injuries.
 Medial meniscus is also intracapsular but extrasynovial. It is C shaped (forms a semicircle) attaching
to the superior surface of tibia at intercondylar area, and is also attached to the medial collateral
ligament. It is more frequently torn in injuries than the lateral meniscus because of its strong attachment
to the tibial collateral ligament.
PG-DIAMS ANATOMY 241

Leg Muscles

Anterior leg muscles (Nerve: Deep peroneal nerve (L-5); Action: Foot extension/dorsiflexion
Muscle Additional Action
Tibialis anterior Foot inversion
Extensor digitorum longus Extends lateral 4 toes
Extensor hallucis longus Extends the great toe
Peroneus tertius Assists in foot eversion
PG-DIAMS ANATOMY 242

Posterior leg muscles; Nerve: Tibial nerve (L-4,5; S-1,2); Action: Plantarflexion of foot & toes
Muscle Additional Action
Gastrocnemius (S1,2) Knee flexion; plantar flexion at ankle (in extended
leg)
Plantaris(S1,2) Works with gastrocnemius
Soleus(S1,2) Plantar flexion at ankle
Popliteus (L4,5;S1) Knee flexion; medial rotation of tibia in unplanted
leg (unlock the knee)
Tibialis posterior(L4,5) Ankle plantarflexion; foot inversion
Flexor digitorum longus(S2,3) Ankle plantarflexion; lateral 4 toes flexion
Flexor hallucis longus(S2,3) Ankle plantarflexion; great toe flexion

Lateral leg muscles (Nerve: Superficial peroneal nerve (L-5; S-1,2)


Muscle Action
Peroneus longus Foot eversion
Peroneus brevis Foot eversion
8. All are seen in Injury to neck of fibula EXCEPT
a) Common peroneal nerve injury
b) Loss of sensation over sole
c) Foot drop
d) Loss of dorsiflexion of toe
PG-DIAMS ANATOMY 243

9. Medial aspect of great toe is supplied by


a) Saphenous nerve
b) Deep peroneal nerve
c) Superficial peroneal nerve
d) Sural nerve

LL - Nerve Injuries
PG-DIAMS ANATOMY 244

Ateries - Lower Limb


10. Head of femur is chiefly supplied by
a) Obturator artery
b) Medial circumflex femoral artery
c) Lateral circumflex femoral artery
d) Superior gluteal artery
PG-DIAMS ANATOMY 245

Questions: Lower Limb


11. Marked dermatome is

a) L-4
b) L-5
c) S-1
d) S-2
12. Posterior cutaneous nerve of thigh supplies skin overlying (PGIC)
a) Medial aspect of thigh
b) Posterior inferior aspect of buttock
c) Scrotum
d) Back of thigh
e) Popliteal fossa
 The root value of posterior cutaneous nerve of thigh is S – 1, 2, 3.
13. Abduction of the thigh is limited by
a) Tension in the adductors
b) Tension in the adductors and iliofemoral ligament
c) Tension in the adductors and pubofemoral ligament
d) Tension in the adductors and ischiofemoral ligament
PG-DIAMS ANATOMY 246

 Pubofemoral ligament reinforces the fibrous capsule inferiorly, extends from the pubis bone to the
femoral neck, and limits abduction and extension.
14. Identify the marked muscle in the gluteal region

a) Obturator externus
b) Obturator internus
c) Quadratus femoris
d) Piriformis
15. Structures passing through lesser sciatic foramen (PGIC)
a) Internal pudendal vessels
b) Obturator internus muscle
c) Pudendal nerve
d) Nerve to obturator internus
e) Pyriformis muscle
 PIN (Pudendal nerve, Internal pudendal vessels and Nerve to obturator internus) structures come from
pelvic cavity, pass through the greater sciatic notch, hook behind the ischial spine (in gluteal region)
and move into the lesser sciatic notch. The tendon (and not muscle) of obturator internus passes
through the lesser sciatic notch.
PG-DIAMS ANATOMY 247

16. All of the following pairs regarding adductor canal are true EXCEPT
a) Roof: Sartorius muscle
b) Contents: Femoral nerve
c) Floor: Adductor longus and magnus
d) Antero-lateral boundary: Vastus medialis
 Adductor canal (Sub-sartorial/Hunter’s canal): This passes from the apex of the femoral triangle to the
popliteal fossa.

17. Clergyman’s knee is an inflammation of


a) Anserine bursa
b) Pre-patellar bursa
c) Suprapatellar bursa
d) Infrapatellar bursa

18. All of the following pairs for boundaries of popliteal fossa are correct EXCEPT
a) Supero-medial boundary: semimembranosus
b) Supero-lateral boundary: Biceps femoris
c) Infero-lateral: Gastrocnemius and plantaris
d) Infero-medial: Gastrocnemius and soleus
PG-DIAMS ANATOMY 248

 The popliteal fossa is the diamond-shaped space bounded superomedially by the semimembranosus
and semitendinosus, superolaterally by the biceps femoris, inferomedially by the medial head of the
gastrocnemius, and inferolaterally by the lateral head of the gastrocnemius and plantaris. It contains the
popliteal vessels, the common peroneal and tibial nerves, and the small saphenous vein.

19. In the following nutrient arteries to bones, choose the WRONG pair
a) Humerus : Profunda brachii
b) Radius: Anterior interosseous
c) Fibula: Peroneal
d) Tibia: Anterior tibial

20. Hunterian perforator is at


a) Upper thigh
b) Mid-thigh
c) Lower thigh
d) Knee

21. Deltoid ligament has all the following components EXCEPT


a) Anterior tibiotalar
b) Tibionavicular
c) Tibiocalcaneal
d) Calcaneonavicular
PG-DIAMS ANATOMY 249

 Medial (Deltoid) Ligament of ankle joint is attached to the medial malleolus on tibia. It has four parts:
the tibionavicular, tibiocalcaneal, anterior tibiotalar, and posterior tibiotalar ligaments. It prevents
overeversion of the foot and helps maintain the medial longitudinal arch.
 Calcaneus bone has a shelf-like medial projection called the sustentaculum tali, which supports the
head of the talus (with the spring ligament) and has a groove on its inferior surface for the tendon of
flexor hallucis longus (which uses the sustentaculum tali as a pulley).
 The plantar calcaneonavicular (spring) ligament passes from the sustentaculum tali of the calcaneus
to the navicular bone. It supports the head of the talus and thereby maintains medial longitudinal
plantar arch. Laxity of this ligament results in fallen arches (flat feet).
 Avulsion or rupture of the Achilles tendon disables the triceps surae (gastrocnemius and soleus)
muscles; thus, the patient is unable to plantar flex the foot.

 Flexor retinaculum is a band of deep fascia , passes between the medial malleolus and the medial
surface of the calcaneus and forms the tarsal tunnel with tarsal bones for the tibial nerve, posterior
tibial vessels, and flexor tendons. It holds three tendons and blood vessels and a nerve in place
deep to it (from anterior to posterior): the tibialis posterior, flexor digitorum longus, posterior tibial artery
and vein, tibial nerve, and flexor hallucis longus (mnemonic: Tom, Dick ANd Harry).
 Tarsal tunnel syndrome is a complex symptom resulting from compression of the tibial nerve or its
medial and lateral plantar branches in the tarsal tunnel, with pain, numbness, and tingling sensations
on the ankle, heel, and sole of the foot. It may be caused by repetitive stress with activities, flat feet, or
excess weight.
 Extensor retinaculum: Bands of deep fascia , under which pass the tendons of the tibialis anterior,
extensor digitorum longus, extensor hallucis longus and the peroneus tertius. Inferior extensor
retinaculum is ‘Y’ shaped.
PG-DIAMS ANATOMY 250

22. All of the following pairs concerning layers of sole muscles are correct EXCEPT
a) First layer: Adductor hallucis
b) Second layer: Lumbricals
c) Third layer: Flexor hallucis
d) Fourth layer: Interossei
I – Abductor Hallucis II - Lumbricals
III – Adductor and flexor hallucis IV – Interossei (Deepest)

 Lumbricals and interossei flex the MTP and extend the IP joints. Their paralysis might result in claw
foot.
 Adductor hallucis muscle is located in the third layerof foot and help in maintenance of transverse
plantar arch.
PG-DIAMS ANATOMY 251

Additional Questions
23. Hip flexion is done by all EXCEPT
a) Ilio-psoas
b) Pectineus
c) Sartorius
d) Semitendinosus
Ans. d) Semitendinosus.
Explanation: Hip flexion is chiefly carried out by ilio-psoas muscle and assisted by muscles like
pectineus, sartorius etc. Semitendinosus is a hamstring muscle for hip extension along with the gluteus
maximus.

24. Superior gluteal nerve acts at hip joint for


a) Abduction & lateral rotation
b) Abduction & medial rotation
c) Adduction & medial rotation
d) Adduction & lateral rotation
Ans. b) Abduction & medal rotation.
Explanation: Superior gluteal nerve supplies three muscles: gluteus medius, gluteus minimus and tensor
fascia lata, which act at the hip joint for abduction, medial rotation and pelvic rotation.

25. WRONG about Trendelenburg test is


a) Contraction of gluteus maximus is assessed
b) Positive in superior gluteal nerve damage
c) Right pelvis drops down in left superior gluteal nerve lesion
d) Bilateral damage results in Waddling gait
Ans. a) Contraction of gluteus maximus is assessed.
Explanation: Trendelenburg test is to check the gluteus medius and minimus muscle (not the gluteus
maximus). Superior gluteal nerve lesion paralyses gluteus medius and minimus, which leads to fall of
contralateral pelvis during swing phase of walking cycle. If the nerve injury is bilateral, it leads to bilateral
lurching (waddling gait).

26. INCORRECT statement regarding popliteus is


a) Intracapsular origin
b) Attaches to medial lemniscus
c) Supplied by tibial nerve
d) Causes flexion and medial rotation at knee joint
Ans. b) Attaches to medial meniscus.
PG-DIAMS ANATOMY 252

Explanation: Popliteus muscle has intracapsular origin from the lateral condyle of femur, has attachment
with the lateral lemniscus (not medial) and inserts into the posterior surface of tibia (floor of popliteal fossa).
It is supplied by tibial nerve and unlocks the knee joint by medial rotation of tibia (in unplanted foot). It
also works with hamstring muscles for knee flexion.

27. Which of the following muscle has intracapsular origin


a) Anconeus
b) Coracobrachialis
c) Long head of biceps femoris
d) Popliteus
e) Peroneus longus
Ans. d) Popliteus.
Explanation: Long head of biceps brachii and the popliteus muscle has intra-capsular origin.

28. A boy playing football received a blow to the lateral aspect of the knee and suffered a twisting
fall. His medial meniscus is damaged, which other structure is most likely to be injured
a) Deltoid ligament
b) Posterior cruciate ligament
c) Anterior cruciate ligament
d) Patellar-ligament

Ans. c) Anterior cruciate ligament.


Explanation: This is a case of terrible (triple) triad and leads to damage of three ligaments: TCL (Tibial
Collateral Ligament), medial meniscus and ACL (Anterior Cruciate Ligament). Lachman test becomes
positive.

29. Following are the nerves and muscles of the leg. Choose the CORRECT pair
a) Superficial peroneal: Soleus
b) Deep peroneal: Peroneus brevis
c) Tibial nerve: Tibialis anterior
d) Common fibular nerve: Short head of biceps
Ans. d) Common fibular nerve: Short head of biceps..
Explanation: Short head of biceps is supplied by the common peroneal nerve. Soleus is calf muscle
suppled by posterior tibila nerve. Peroneus brevis is a lateral leg muscles innervated by superficial peroneal
nerve. Tibialis anterior is supplied by deep peronela nerve.

30. Postero-lateral herniation of nucleus pulposus at L5 – S1 vertebrae level will result in pain
located along the
a) Anterior aspect of the thigh
b) Medial aspect of the thigh
c) Antero-medial aspect of the leg
d) Lateral side of the foot
Ans. d) Lateral side of the foot.
Explanation: The nerve root involved is S-1, and the corresponding dermatome involved is the lateral side
of the foot and little toe.

31. Femoral pulsation can be best felt at


a) Below and medial to pubic tubercle
b) Near anterior superior iliac spine
c) Mid-point of inguinal ligament
d) Mid-inguinal point
Ans. d) Mid-inguinal point.
Explanation: Femoral artery pulsation can be felt against the head of femur at mid-inguinal point.

32. Middle genicular artery is a branch of


a) Femoral artery
b) Popliteal artery
c) Anterior tibial artery
d) Posterior tibial artery
PG-DIAMS ANATOMY 253

Ans. b) Popliteal artery.


Explanation: Five genicular arteries are given by popliteal artery (including middle genicular artery) to
supply knee joint. There are two superior (medial & lateral) and two inferior (medial & lateral).

33. All are Valveless EXCEPT


a) Dural venous sinus
b) Hepatic veins
c) Inferior vena cava
d) Femoral vein
Ans. d) Femoral vein.
Explanation: Femoral veins contain between one and six valves, and popliteal veins contain between zero
and four valves. Deep vein valves are consistently located in the common femoral vein (within 5 cm of the
inguinal ligament), the femoral vein (within 3 cm of the deep femoral vein tributary) and in the popliteal vein
near the adductor hiatus.

34. The vessel at marker ‘D’ in the leg region is

a) Profunda femoris artery


b) Obturator artery
c) Posterior tibial artery
d) Peroneal artery
Ans. d) Peroneal artery.
Explanation: A: Politeal artery; B: Tibio-peroneal trunk; C: Anterior tibial artery; D: Fibular (peroneal) artery,
the shortest artery in the leg; E: Posterior tibial artery (Longest artery, which gives nutrient artery to tibia
and later gives plantar arteries in the foot).
PG-DIAMS ANATOMY 254

35. Tibialis posterior is inserted in all the tarsal bones EXCEPT


a) Calcaneus
b) Intermediate cuneiform
c) Cuboid
d) Talus
Ans. d) Talus.
Explanation: Tibialis posterior muscle has extensive attachments on the foot bones, but is not attached to
talus bone.Talus bone in the foot and Incus bone in the middle ear cavity has no has no muscle
attachments.
36. Which is NOT a part of medial longitudinal arch of foot
a) Third metatarsal
b) Cuboid
c) Calcaneum
d) Talus

Ans. b) Cuboid.
Explanation: Cuboid bone is present at the lateral aspect of the foot, articulates with calcaneum (CC joint
is saddle synovial) and both bones contributes to lateral longitudinal arch. Cuboid bone is the keystone
bone for the arch. Cuboid bone has a groove for the tendon of peroneus longus muscle.
 Lateral longitudinal arch is contributed by the calcaneus, the cuboid bone, and the lateral two
metatarsal bones. The keystone is the cuboid bone. It is supported by the peroneus longus tendon
and the long and short plantar ligaments.
 Medial longitudinal arch is contributed and maintained by the of the talus, calcaneus, navicular,
cuneiform, and three medial metatarsal bones. The keystone is the head of the talus, which is located
at the summit between the sustentaculum tali and the navicular bone. It is supported by the spring
ligament and the tendon of the flexor hallucis longus. Flat foot (pes planus or talipes planus) is a
condition of disappearance or collapse of the medial longitudinal arch with eversion and abduction of
the forefoot and leads to pain as a result of stretching of the plantar muscles and straining of the spring
ligament and the long and short plantar ligaments.
 Transverse arches: . 1. Proximal (metatarsal) arch is formed by the navicular bone, the three
cuneiform bones, the cuboid bone, and the bases of the five metatarsal bones of the foot. It is
supported by the tendon of the peroneus longus. 2. Distal arch is formed by the heads of five metatarsal
bones. It is maintained by the transverse head of the adductor hallucis.
37. Inversion & eversion mainly happen at which joint
a) Inferior tibio-fibular
b) Ankle
c) Subtalar
d) Calcaneo-cuboid
PG-DIAMS ANATOMY 255

Ans. c) Sub-talar.
Explanation: Subtalar (Talocalcaneal) joint It is a plane synovial joint (part of the talocalcaneonavicular
joint), and is formed between talus and calcaneus bones. Inversion and eversion of the foot occurs at this
joint.

38. Which tendon passes below sustentacula tali


a) Tibialis anterior
b) Tibialis posterior
c) Flexor hallucis longus
d) Flexor digitorum longus
Ans. c) Flexor hallucis longus.
Explanation: The tendon of flexor hallucis longus (calf muscle) passes deep to flexor retinaculum on the
mediaial side of the ankle and then underneath the sustentaculum talus of calcaneus bone, to pull on the
great toe for flexion.

39. All of the following pass under the flexor retinaculum EXCEPT
a) Tibialis anterior
b) Tibialis posterior
c) Posterior tibial artery
d) Deep peroneal nerve
e) Anterior tibial nerve
Ans. A) Tibilais anterior; d) Deep peroneal nerve; e) Anterior tibial nerve
Explanation: Tibialis anterior and deep peroneal (anterior tibial) nerve pass under the anteriorly placed
extensor retinaculum.

40. In foot pronation, the axis of which two joints become parallel
a) Talo-calcaneal and talo- navicular
b) Talo-calcaneal and calcaneo -cuboid
c) Subtalar and Lisfranc
d) Talo-navicular and calcaneo-cuboid

Ans. d) Talo-navicular and calcaneo-cuboid.


Explanation: Transverse tarsal (Midtarsal) joint is a collective term for the talonavicular (TN) part of the
talocalcaneonavicular joint and the calcaneocuboid (CC) joint. The two joints are separated anatomically
but act together functionally. During supination of foot, the soles face each other and there occurs
inversion, whereas, in pronation, soles move outwards and is accompanied by eversion (TN and CC joint
become parallel).

High Yield Facts


 In lower limb, gracilis is the most common muscle used for surgical grafting.
 Sartorius muscle causes flexion at both hip and knee joint (Sartor – Tailor).
 Tibialis anterior is the muscle, which works in stance as well as swing phase of walking cycle.
 Lateral femoral cutaneous nerve arises from the lumbar plexus (L2–L3), passes under the inguinal
ligament near the anterior-superior iliac spine and supply skin on the anterolateral aspect of thigh. It
might get involved in meralgia paraesthetica.
PG-DIAMS ANATOMY 256

 Saphenous Nerve is a branch of femoral nerve given in the femoral triangle and descends with the
femoral vessels through the femoral triangle and the adductor canal. Then it is accompanied by the
great saphenous vein to reach the medial margin of the foot. Innervates the skin on the medial side of
the leg and foot. It is vulnerable to injury during venesection at the medial malleolus.
 Pott’s (Dupuytren’s) fracture is caused by forced eversion of foot and involves the lower end of the
fibula, often accompanied by fracture of the medial malleolus (or rupture of the deltoid ligament).
 Most stable position for ankle joint is in dorsiflexion, when anterior wider part of talus(trochlear
surface) fits properly in tibio-fibular mortise.
 Obturator externus is supplied by lumbar plexus (obturator nerve).
 Tibio-femoral condyles are involved in weight transmission (pressure epiphysis) and are intracapsular.
PG-DIAMS ANATOMY 257

Q. A man comes with aphasia, is unable to name things and repetition is poor. However
comprehension, fluency and articulation is unaffected. He is probably suffering from (AIIMS)
a) Anomic aphasia
b) Transcortical sensory aphasia
c) Conduction aphasia
d) Broca's aphasia
PG-DIAMS ANATOMY 258
PG-DIAMS ANATOMY 259
PG-DIAMS ANATOMY 260
PG-DIAMS ANATOMY 261

 The liver is divided into four portal sectors by the four main branches of the portal vein. These are right
lateral, right medial, left medial and left lateral (sometimes the term posterior is used in place of lateral
and anterior in place of medial).
 The three main hepatic veins lie between these sectors as intersectorial veins. These intersectoral planes
are also called portal fissures (scissures). The fissures containing portal pedicles are called hepatic
fissures. Each sector is sub-divided into segments (usually two) based on their supply by tertiary divisions
of the vascular biliary sheaths.
 Three major fissures, not visible on the surface, run through the liver parenchyma and harbour the three
main hepatic veins (main, left and right portal fissures). Three minor fissures are visible as physical
clefts of the liver surface (umbilical, venous and fissure of Gans).
 Segment I (anatomical caudate lobe) lies posterior (dorsal) to segment IV with its left half directly
posterior to segments II and II and its medial half surrounded by major vascular branches. This segment
is a boundary line structure and receives dual artery, vein and duct supply. The Glissonian sheaths to
segment I arise from both right and left main sheaths: the segment therefore receives vessels
independently from the left and right portal veins and hepatic arteries. Caudate lobe is peculiar in the
finding that it drains independently into the inferior vena cava by multiple small branches and not into
major hepatic veins.. The bile ducts draining the segment are closely related to the confluence of the right
and left hepatic ducts.
Q. All is true about functional divisions of liver EXCEPT (AIIMS)
a) Based upon portal vein & hepatic vein
b) Divided into 8 segments
c) Three major & three minor fissures
d) 4 sectors
 Liver is divided into eight (functional (surgical) segments according to Couinaud’s classification,
following hepatic veins and portal veins, and is further enhanced by following bile duct distribution.
Recently there was addition of a ninth segment, but most of the surgeons do not accept the new
addition for operative procedures.
 The Bismuth system, which is a modified version of the Couinaud system, is the most commonly used
anatomic nomenclature system. In the Bismuth system, each segment has an independent vascular
supply, including arterial, portal, and venous supplies, as well as independent lymphatic and biliary
drainage.
PG-DIAMS ANATOMY 262
PG-DIAMS ANATOMY 263

High Yield Facts


 Peri-conceptional intake of 400 µg of folic acid per day by the pregnant women (starting from 1 month prior
to 3 months after conception) can prevent 75% of all neural tube defects. Intake of 4000 µg /d is
recommended to prevent recurrences.
 Cancellous bone is metabolically more active than the cortical bone. Endosteum is metabolically more
active than periosteum.
 Reticular fibres/collagen type III is absent in thymus. Basement membrane has type – IV collagen.
 Jersey finger (Rugby finger or Sweater finger) is a type of injury due to avulsion of the flexor digitorum
profundus (FDP) at the base of the distal interphalangeal joint.
 Cranial nerves carrying GVE (General Visceral Efferent) fibres come under autonomic nervous system like
3, 7, 9 and 10 (parasympathetic system). Cranial nerve 1, 2 and 8 belong to SSA (Special Somatic Afferent)
column.
 Glomerulus is a derivative of metanephric blastema (and not mesonephric duct).
 Lower two part of sternum fused by age of 14 years.
 Nerve beneath the facial colliculus is facial nerve, while the nucleus deep to that is abducent nucleus.
 SIT (Supraspinatus, Infraspinatus, Teres minor) muscles sit on greater tubercle. Subscapularis muscle
attaches to lesser tubercle of humerus.
 First part of subclavian artery give VIT (Vertebral, Internal thoracic, Thyrocervical) branches. T
(Thyrocervical trunk) itself give SIT (Suprascapular, Inferior thyroid artery, Transverse cervical) branches.
 Card test is done to check palmar interossei.
 Microvilli are present in collecting ducts, but not as markedly developed as in PCT, Gall bladder,
Duodenum.
 Independent assortment of chromosomes occur during meiosis-I, when primary spermatocyte converts
into secondary spermatocyte.
 Cornea is supplied by nasociliary nerve (Trigeminal, ophthalmic division).
 Trochlear nerve injury leads to paralysis of superior oblique muscle (an intortor) and presents with affected
eye in extortion leading to vertical diplopia, especially while reading a book (adduction and depression),
and the patient attains a contralateral head tilt to correct it.
 At birth, ovaries contains primary oocyte arrested in diplotene stage of prophase – I, due to OMI (Oocyte
Maturation Inhibitor).
 Pelvic viscera (Uterus) are supported by Urogenital diaphragm, Pelvic diaphragm, Perineal body (but not
Rectovaginal septum).
 Mesorectum do not contain inferior rectal vein
 Intact comprehension and motor articulation but poor repetition is a feature of conduction aphasia.
 Brown adipose tissue is present around scapula, adrenals, blood vessels. White fat is subcutaneous (not
brown?).

Vous aimerez peut-être aussi